Download as pdf or txt
Download as pdf or txt
You are on page 1of 89

TRAUMA / WOUND HEALING

RAFAEL H. DIZON, MD, FPCS, FPSPS, MBA

TRAUMA Endotracheal Intubation


 Nasotracheal intubation
Definition  Orotracheal intubation
 synonymous with injury  Operative
 cellular disruption caused by an exchange with  Cricothyroidotomy
environmental energy that is beyond the body’s  Tracheostomy
resilience which is compounded by cell death
due to ischemia/reperfusion Breathing
 supplemental oxygen
Statistics (U.S. Data)  pulse oximetry monitoring
 most common cause of death for all individuals
between age 1 and 44 years Rule out the following:
 third most common cause of death regardless of  tension pneumothorax
age  open pneumothorax
 injury-related medical expenses are estimated to  flail chest/pulmonary contusion
be 117 billion USD per year  massive air leak

 the American College of Surgeons Committee on Tension Pneumothorax


Trauma developed the Advanced Trauma Life  suspected when there is respiratory distress and
Support (ATLS) course hypotension
 premise is that appropriate and timely care can  Signs
significantly improve the outcome for the injured  tracheal deviation away from affected
patient sign
 lack of or decreased breath sounds
Phases  subcutaneous emphysema on the
 Primary survey/concurrent resuscitation affected side
 Secondary survey/diagnostic evaluation  Treatment
 Definitive care  immediate needle thoracostomy
 Tertiary survey decompression with a 14-gauge
needle (when in the field)
Primary Survey  tube thoracostomy (when in the
 identify and treat conditions that constitute an ED)
immediate threat to life
 “ABCs” Open Pneumothorax
 Airway  sucking chest wound
 Breathing  full-thickness loss of the chest wall permitting
 Circulation free communication between the pleural space
and the atmosphere
Airway Management  results in hypoxia and hypercarbia
 first priority in the primary survey
 futile to restore cardiovascular integrity if Flail Chest
oxygen in blood is inadequate  occurs when 3 or more ribs are fractures in at
 blunt trauma patients require cervical least two locations
immobilization until injury is excluded  paradoxical chest movement
 application of a herd collar or sand bags  with associated pulmonary contusion
on both sides of the head and taped to  may require intubation and mechanical
the forehead and the backboard ventilation
 not useful in penetrating neck injury
Massive Air Leaks
Airway  occurs from major tracheobronchial injuries
 “Are you okay?”  may present with pneumothorax
 Normal airway  bronchoscopy confirms the diagnosis and directs
 conscious management
 without tachypnea
 with a normal voice Circulation
 next priority once airway is secured and the
 patients with abnormal voice, abnormal ventilation is established
breathing, tachypnea, or altered mental status  carotid pulse: at least 60 mmHg
should have further airway evaluation  femoral pulse: at least 70 mmHg
 vomit, blood, the tongue, foreign objects, and  radial pulse: at least 80 mmHg
soft tissue swelling can cause airway obstruction  any episode of hypotension (SBP <90 mmHg) is
 suctioning affords immediate relief in many assumed to be caused by hemorrhage unless
cases proven otherwise
 in comatose patients the tongue can fall back
and obstruct the airway  Quick estimation of blood volume
 chin lift/jaw thrust can relieve obstruction  Adult: 70 ml/kg
UST FMS MEDICAL BOARD REVIEW 2019 1 | SURGERY
TRAUMA / WOUND HEALING
RAFAEL H. DIZON, MD, FPCS, FPSPS, MBA

 Pedia: 80 ml/kg
 Estimated blood loss, adults
 Each rib fracture: 100-200 ml
 Tibial fracture: 300-500 ml
 Femur fracture: 800-1000 ml
 Pelvic fractures: > 2000 ml

 In the ER
 establish IV access
 two peripheral catheters, 16-
gauge or larger
 intraosseus (IO) needles can be
rapidly placed in the proximal
tibia of the lower extremity
 venous cutdown
 Saphenous vein cutdown
 The saphenous vein is consistently
found 1 cm anterior and 1 cm superior
to the medial malleolus

 External control of any visible hemorrhage


 Manual compression with 4x4 gauze
using a gloved hand
 Blind clamping must not be done to
avoid injury to adjacent structures
 Splinting of open fractures
 Closure of profusely bleeding scalp
lacerations

 Four life threatening injuries that must be


identified promptly
 Massive hemothorax
 Cardiac tamponade
 Massive hemoperitoneum
 Mechanically unstable pelvic fractures
with bleeding
 Massive Hemothorax
 >1500 mL of blood in the thorax
 In the pediatric population, >25% of
patient’s blood volume in the thorax
 Tube thoracostomy
 Possible operative intervention
 Cardiac Tamponade
 Most commonly after penetrating
thoracic wounds
 >100 mL of pericardial blood may cause
tamponade
 Beck’s Triad: dilated neck veins, muffled
heart tones, hypotension
 Best diagnosed using bedside
ultrasound of the pericardium
 Ultrasound-guided pericardiocentesis
can improve hemodynamic status (as
little as 15-20 mL of blood removed)
 Allow safe transport to OR for
sternotomy

Disability
 Glasgow Coma Scale (GCS) should be
determined for all injured patients
 Quantifiable determination of neurologic
function
 Useful for triage, treatment and prognosis

UST FMS MEDICAL BOARD REVIEW 2019 2 | SURGERY


TRAUMA / WOUND HEALING
RAFAEL H. DIZON, MD, FPCS, FPSPS, MBA

Fluid Resuscitation  Blood tests


 Begin with 2 L IV bolus in adults
 20 mL/kg in children
 Isotonic crystalloid (Ringer’s lactate)
 PRBC and FFP are used when massive
transfusions are needed
 Urine output is a reliable indicator of organ
perfusion
 0.5 mL/kg/hr (adult)
 1 mL/kg/hr (child)
 2 mL/kg/hr (infant)

Secondary Survey
 Performed once immediate threats to life have
been addressed
 Thorough history and physical examination
 AMPLE (Allergies, Medication, Past Illnesses,
Last Meal, Events related to the injury)
 Head to toe, front and back physical
examination
 Vital signs and CVP monitoring
 ECG monitoring
 NGT placement
 Foley catheter placement
 X-rays

UST FMS MEDICAL BOARD REVIEW 2019 3 | SURGERY


REVIEW TEST
RAFAEL DIZON, MD

TRAUMA AND WOUND HEALING


REVIEW TEST

Choose the best answer:

_____1. A 25 year-old male crashed while riding his _____6. What cells enter the wound first during
motorcycle and was not wearing a helmet. At the ER acute injury?
he was in severe respiratory distress and A. Fibroblasts
hypotensive (BP 80/40 mm Hg) and was cyanotic. B. Lymphocytes
He was bleeding profusely from the nose and had a C. Macrophages
fracture on the right thigh with exposed bone. D. Neutrophils
Breath sounds were decreased on the right side of Ans: D
the chest. Initial management priority should be:
A. Control of hemorrhage with anterior and
posterior nasal packing. _____7. Which statement is true regarding the role
B. Endotracheal intubation with in-line cervical of neutrophils in wound healing?
traction. A. Peaks at 1 week post-injury
C. Obtain IV access and begin emergency type O B. Second population of inflammatory cells that
blood transfusions. invade the wound
D. Tube thoracostomy in the right hemithorax. C. Primary role is phagocytosis of bacteria and
Ans: B tissue debris
D. Derived from circulating monocytes
Ans: C
_____2. Which of the following steps is NOT part of
the primary survey in a trauma patient?
A. Insuring adequate ventilatory support _____8. What cells are responsible for synthesizing
B. Measurement of blood pressure and pulse collagen?
C. Neurologic evaluation of Glasgow Coma Scale A. Endothelial cells
(GCS) B. Lymphoblasts
D. Examination of the the cervical spine C. Fibroblasts
Ans: D D. Collagenocytes
Ans: C

_____3. Immediate life-threatening injuries that


preclude air exchange which can be treated in the _____9. What cells are responsible for angiogenesis
field include which of the following EXCEPT: during the proliferative phase of wound healing?
A. Massive open chest wounds A. Endothelial cells
B. Sucking chest wounds B. Lymphoblasts
C. Tension pneumothorax C. Fibroblasts
D. Tracheal disruption D. Collagenocytes
Ans: D Ans: A

_____4. Effective procedure to control life- _____10. Which layer of the intestines imparts the
threatening massive bleeding of a right thigh injury greatest tensile strength and greatest suture-holding
in the Emergency Department: capacity?
A. Application of an arterial tourniquet A. Mucosa
B. Clamping of the bleeding vessel with B. Submucosa
hemostats C. Muscularis
C. Direct pressure with gauze D. Serosa
D. Use of hemostatic agents Ans: B
Ans: C

_____5. A 20 year old college student was brought


to ER after throwing himself off his school building 4
stories high. He is opens his eyes to pain, speaks
with inappropriate words and exhibits abnormal
extension of his extremities. What is his GCS score?
A. 6
B. 7
C. 8
D. 9
Ans: B

UST FMS MEDICAL BOARD REVIEW 2019 1 | SURGERY


SHOCK
GERALD ALCID, MD

SHOCK
BREATHING
- Failure to meet the metabolic demands of cells - Respiratory muscles consume a significant
and tissues and the consequences that ensue amount of oxygen
- DECREASED TISSUE PERFUSION results in - Tachypnea can contribute to lactic acidosis
1. Lactic acidosis - Mechanical ventilation and sedation decrease
2. Cardiovascular insufficiency work of breathing
3. ↑ metabolic demands
CIRCULATION
COMPENSATORY MECHANISMS - Isotonic crystalloids titrated to CVP 8-12 mmHg,
- Inadequate systemic oxygen delivery activates Urine output 0.5ml/kg/hr, improving heart rate,
autonomic response to maintain systemic oxygen BP (MAP 65-90 mmHg)
delivery - No outcome benefits from colloids
- Sympathetic nervous system
• NE, epinephrine, dopamine and cortisol release OXYGEN DELIVERY
(causes vasoconstriction, increase HR, and - Decrease oxygen demand (analgesia, anxiolytics)
increase in cardiac contractility/cardiac output) - Maintain arterial oxygen saturation content
- Renin-Angiotensin axis - Give supplemental oxygen
• Water and sodium conservation and - Maintain hemoglobin > 10g/dL
vasoconstriction - Serial lactate levels or central venous 02
• Increase in blood volume and blood pressure saturation (>70%) to assess tissue oxygen
extraction
PATHOPHYSIOLOGY OF SHOCK
- TISSUE HYPOPERFUSION (main) and the ENDPOINTS IN RESUSCITATION
developing cellular energy deficit Systemic global
- Imbalance between cellular supply and demand Lactate
leads to NEUROENDOCRINE and INFLAMMATORY Base deficit
responses Cardiac output
- Magnitude of which is usually proportional to the Oxygen delivery and consumption
degree and duration of shock Tissue specific
- Goal: aimed at maintaining perfusion in the Gastric tonometry
Tissue pH, oxygen, carbon dioxidelevels
cerebral and coronary circulation
Near infrared spectroscopy
- Pathophysiology responses vary with time and in Cellular
response to resuscitation Membrane potential
Adenosine triphosphate
PHASES OF SHOCK
Goal-directed approach
COMPENSATED
- The body compensate for the initial loss of blood • UO >0.5 ml/kg/hr
volume primarily through the neuroendocrine • CVP 8-12 mmHg
response to maintain hemodynamics • MAP 65 to 90 mmHg
• Central venous oxygen concentration >70%
DECOMPENSATED
- Continued hypoperfusion, cellular death and CLASSIFICATION OF SHOCK
injury 1. Hypovolemic
2. Cardiogenic
IRREVERSIBLE 3. Septic (vasogenic)
- Persistent hypoperfusion leading to hemodynamic 4. Neurogenic
derangements and cardiovascular collapse leading 5. Traumatic
to DEATH 6. Obstructive (pulmonary embolism, tension
pneumothorax)
Goals of Treatment - ABCDE
1. HYPOVOLEMIC/HEMORRHAGIC
Airway - Most common cause of shock in the surgical or
Control work of Breathing trauma patient is loss of circulating volume from
Optimize Circulation hemorrhage
Assure adequate oxygen Delivery
Achieve Endpoints of resuscitation In acute blood loss (COMPENSATORY MECHANISMS)

AIRWAY • ↑ vasoconstriction and peripheral arterial


- Determine need for intubation resistance
- May need volume resuscitation prior to intubation • Induce sympathetic stimulation (epinephrine &
to avoid hemodynamic collapse norepinephrine release)
UST FMS MEDICAL BOARD REVIEW 2019 1 | SURGERY
SHOCK
GERALD ALCID, MD

• Activation of Renin-Angiotensis cascade b) Adequate volume resuscitation


c) Debridement of non-viable tissue
• ↑ Vasopressin release
d) Stabilization of bony injuries
e) Treatment of soft tissue injuries
PERIPHERAL VASOCONSTRICTION IS PROMINENT,
while lack of sympathetic effect on cerebral and
3. SEPTIC SHOCK (VASODILATORY SHOCK)
coronary vessels and auto regulation promote
maintenance of cardiac and CNS blood flow
- Result of dysfunction of the endothelium and
vasculature secondary to circulating inflammatory
Classification of hemorrhage mediators and cells or as a response to prolonged
and severe hypoperfusion
Class
HYPOTENSION (NOT due to volume depletion)
Parame I II III IV - Failure of the vascular smooth muscle to constrict
ter appropriately

Blood <750 750- 1500- >2000 CAUSES OF SEPTIC AND VASODILATORY SHOCK
loss 1500 2000
(ml) Systemic response to infection
Non-infectious systemic inflammation
Blood <15 15-30 30-40 >40 Pancreatitis
loss Burns
(%) Anaphylaxis
Acute adrenal insufficiency
Heart <100 >100 >120 >140
Prolonged severe hypotension
rate
Hemorrhagic shock
(bpm)
Cardiogenic shock
Blood Normal Orthost Hypote Severe Cardiopulmonary bypass
pressur atic nsion hypoten Metabolic
e sion Hypoxic lactic acidosis
Carbon monoxide poisoning
CNS Normal Anxious Confuse Obtund
sympto d ed
• Mortality rate for severe sepsis (30-50%)
ms
• Clinical manifestations: fever, leucocytes,
hyperglycemia and tachycardia
- Clinical manifestation: agitation, cool clammy
• The vasodilatory effects in septic shock is
extremities, tachycardia, weak or absent secondary to up regulation of the inducible form of
peripheral pulses and hypotension nitric oxide synthase (iNOS or NOS2) in the vessel
- Treatment: wall
a) Secure airway
• This potent vasodilator suppresses vascular tone
b) Control the source of blood loss and renders the vascular resistant to the effects of
c) IV volume resuscitation (crystalloids is the fluid vasoconstricting agents
of choice)
d) Blood replacement - target hemoglobin of 7 to 9 • Diagnosis:
g/dL • Identification of the offending organisms
e) FFO given in massive bleeding or PT > 1.5 INR (cultures)
f) Platelets to maintain > 50 x 109/L
g) Maintaining normothermia Sepsis - evidence of an infection, as well as systemic
signs of inflammation (fever, leucocytosis and
The presence of hypothermia is associated with: tachycardia)
a. Acidosis Severe sepsis - hypoperfusion with signs of organ
b. Hypotension dysfunction
c. Coagulopathy Septic shock - presence of the above with systemic
hypotension
2. TRAUMATIC SHOCK
- Small volume hemorrhage with soft tissue injury Treatment:
(femoral fracture, crush injury) • Airway and ventilation
- Combine effects of soft tissue injury, long bone • Fluid resuscitation
fractures, and blood loss • Empiric antibiotics (gram (-) rods, gram (+) cocci
- Higher incidence of multiple organ failure (ARDS) and anaerobes)
compared to purely hemorrhagic shock • Source control (focus of infection)
- Hypoperfusion deficit is magnified by • Vasopressor in patients with hypotension
PROINFLAMMATORY ACTIVATION
• Goal-directed therapy: (reduced 28 days mortality
- Treatment: in sepsis)
a) Prompt control of hemorrhage

UST FMS MEDICAL BOARD REVIEW 2019 2 | SURGERY


SHOCK
GERALD ALCID, MD

- Adjustment of cardiac preload. after load and - Deviation of mediastinal structure


contractility to balance O2 delivery with O2 - Depression of the hemidiaphragm
demand - Hypo-opacification with absent lung
- Higher mean venous O2 saturation markings
- Lower lactate levels - Treatment: Pleural decompression (needling) if
- Lower base deficit chest tube is not immediately available
- Higher pH
CARDIAC TAMPONADE
• Hyperglycemic control (80-110 mg/dL)
o Causes:
• Corticosteroids (50 mg IV every 6 ) - to address - Penetrating trauma
adrenal insufficiency which is present in some
patients with sepsis
- Heart failure, uremia
- Dyspnea, orthopnea, couch, peripheral edema,
4. CARDIOGENIC SHOCK chest pain, tachycardia, muffled heart tones
- Circulatory pump failure leading to diminished - Jugular venous distention and ↑ CVP
forward flow and subsequent tissue hypoxia BECK’S TRIAD: hypotension, muffled heart tones,
- Criteria: neck vein distention
• Sustained hypotension (SPP <90 mmHg for at
6. NEUROGENIC SHOCK
least 30 mins)
• Reduced cardiac index (<2.2 L/min per square Causes of neurogenic shock
meter) - Spinal cord trauma
• Elevated pulmonary artery wedge pressure (>15 - Spinal cord neoplasm
mmHg) - Spinal/epidural anesthesia
- Acute or extensive MI is the most common
- Diagnosis: - Diminished tissue perfusion as a result of lass of
• ECG vasomotor tone to peripheral arterial beds
• 2D Echo - Usually secondary to spinal cord injuries from
• Cardiac enzymes vertebral body fractures
- Treatment: - Diagnosis:
• Cardiotonics • Bradycardia
• Inotrophic support (to improve cardiac • Hypotension
contractility and CO) • Cardiac dysrhythmias
• ↓ CO
5. OBSTRUCTIVE SHOCK
- Mostly caused by mechanical obstruction of • ↓ peripheral vascular resistance
- Classic description:
venous return but in trauma patient it is
secondary to tension pneumothorax • ↓ BP associated with bradycardia (absence of
reflexive tachycardia due to disrupted
Causes of obstructive shock: sympathetic discharge)
- Pericardial tamponade • Warm extremities (loss of peripheral
- Pulmonary embolus vasoconstriction)
- Tension pneumothorax • Motor and sensory deficits
- IVC obstruction • Radiographic evidence of vertebral column
- Deep venous thrombosis fracture
- Gravid uterus on IVC - Treatment:
- Neoplasm • Airway/ventilation
- Increased intrathoracic pressure • Fluid resuscitation
- Excess positive end-expiratory pressure • Vasocontrictors (Dopamine)
- Neoplasm

TENSION PNEUMOTHORAX
- Respiratory distress
- Hypotension
- Diminished breath sounds
- Hyperresonance
- Jugular venous distention
- Shift of mediastinal structures to the unaffected
side with tracheal deviation
- Diagnosis
- Chest x-ray
UST FMS MEDICAL BOARD REVIEW 2019 3 | SURGERY
SHOCK
GERALD ALCID, MD

Hemodynamic Variables in Different Shock


States

Physiologic State Cardiac Index Systemic Pulmonary Prime Mover*


Vascular Capillary Wedge
Resistance Pressure

Normal 2.4-3.0 L/min/m2 800-1200 dyne-s or 8-12 mmHg Not applicable


dyne-s/cm

Distributive Elevated Decreased Low to normal Decreased vascular


(septic, (because of tone
neurogenic, decreased vascular
anaphylactic) tone)

Cardiogenic Decreased Increased Increased Decreased cardiac


(myocardial contractility
infarction,
cardiomyopathy)

Hypovolemic Decreased Increased (to Decreased Decreased pre-load


(hemorrhage, (because of attempt to
dehydration) decreased maintain blood
volume) pressure)

Obstructive Decreased Increased Normal to Obstruction of


(tamponade, increased flow blood flow
tension
pneumothorax,
pulmonary
embolus)

* The prime mover is the initial pathophysiologic


change; this change then causes compensatory
changes in other variables

UST FMS MEDICAL BOARD REVIEW 2019 4 | SURGERY


HEAD AND NECK
IDA MARIE T. LIM, MD

HEAD AND NECK Pyramidal lobe - 80 percent of individuals; usually


just to the left of the midline. It is a remnant of the
What are the important and relevant head and neck thyroglossal duct
diseases encountered by a surgeon?
The four parathyroid glands usually are found on the
1. Thyroid gland abnormalities in function and posterolateral surface of the lobes, within 1 cm of
in structure: the inferior thyroid artery in 80 percent of
a. Hyperthyroidism individuals.
b. Nodules- benign and malignant
Blood supply
2. Salivary gland tumors: A. Arterial supply:
a. According to involved gland: 1. superior thyroid arteries- first branch of the
i. Parotid gland external carotid artery,
ii. Submandibular gland
iii. Sublingual gland 2. inferior thyroid arteries- arise from the
thyrocervical trunk of the subclavian arteries
b. According to pathology and enter the gland from a posterolateral position
i. Benign tumors
ii. Malignant tumors 3. thyroidea ima, is occasionally present,
originating directly from the aortic arch or the
3. Head and neck mucosal squamous cell innominate artery and ascending in front of the
cancer trachea to enter the gland in the midline inferiorly.
a. Epidemiology -causes
b. Principles of management B. Venous drainage:
c. Management of lymph node 1. Superior thyroid vein and middle thyroid vein-
metastases drains to the internal jugular vein

THYROID GLAND 2. inferior thyroid veins –drain the inferior poles


bilaterally, usually forming a plexus that drains into
Review relevant anatomy and physiology the brachiocephalic vein.

The thyroid follicles:


-functional, secretory units of the thyroid gland that
produce thick, proteinaceous colloid

-Colloid is composed primarily of thyroglobulin


(Tg), a high-molecular weight glycoprotein that
- bilobed structure connected by an isthmus. facilitates the assembly of thyroid hormones within
- soft and brown-red; the colloid gives the cut the thyroid follicular lumen by providing the matrix
surfaces a glistening, translucent appearance for hormone synthesis and by storing iodine and
- has a thin capsule, usually incomplete, does not hormones which can be released at a steady rate or
strip easily, and contains sizable venous on demand.
channels that become strikingly prominent
when vascularity is increased

 N adult thyroid gland light brown in color;


firm, weighs 15 to 20 g.
 formed by two lateral lobes connected
centrally by an isthmus.
the lobes are 4 cm long, 2 cm wide, and 20 to 40
mm thick, with the isthmus 2 to 6 mm thick.

UST FMS MEDICAL BOARD REVIEW 2019 1 | SURGERY


HEAD AND NECK
IDA MARIE T. LIM, MD

 The formation of thyroid hormones is Hypothalamic-pituitary-thyroid axis (H_P_O)-


dependent on the availability of exogenous the principal homeostatic control of thyroid hormone
iodine. Ave daily requirement is 0.1 mg secretion.

 Iodine is rapidly converted to iodide in the TSH – produced by the basophil cells of the
stomach and jejunum and is absorbed into anterior pituitary;
the bloodstream within 1 h; and from there -directly regulates thyroid function by its
it is distributed uniformly throughout the action on the thyroid cell to promote thyroid
extracellular space hormone production at all levels, enhancing iodine
uptake, increasing synthesis, and raising secretion of
 Iodide is actively transported into the T4.
thyroid follicular cells by an ATP- dependent -also has a secondary action on thyroid
process. gland growth, increasing cellularity and
 thyroid-serum iodine ratio under normal vascularization of the gland
conditions is about 50:1, and most of the
body's store of iodine is found in the thyroid - Release of TRH from the hypothalamus is
gland (90 percent). suppressed by T 3, acting in a feedback loop. TRH
has been shown to be equipotent in stimulating
 Summary of steps in the synthesis of release of prolactin from the pituitary and TSH.
thyroid hormone are:
(1) active trapping and concentration of iodide I. FUNCTIONAL ABNORMALITIES OF THE
in the follicular cell; THYROID GLAND
(2) rapid oxidation of iodide to iodine;
(3) linkage of iodine with tyrosine residues in What is thyrotoxicosis/hyperthyroidism?
thyroglobulin; • “thyrotoxicosis” - clinical syndrome
(4) coupling of these iodotyrosines (monoiodo- caused by inappropriately high thyroid
and diiodotyrosine) to form the active thyroid hormone action in tissues generally due to
hormones T 4 and T3. excessive levels of active thyroid
hormone secreted into the circulation
• “hyperthyroidism ” - a form of
thyrotoxicosis due to inappropriately high
synthesis and secretion of thyroid
hormone(s) by the thyroid gland
• Rarely due to excess TRH or TSH production
• Great majority (>98%) are due to excess
hormones from the thyroid gland or
exogenous source
Causes:
1. Primary Thyroid Problem:
- Increased production of thyroid hormone from the
gland:
a. Graves' disease (diffuse toxic goiter)
b. toxic solitary or multinodular goiter
(Plummer's disease).
2. Extrathyroidal causes:
a. Leak of thyroid hormones
Acute stage of thyroiditis
b. Factitious hyperthyroidism
(exogenous thyroid hormone)
c. Struma ovarii
d. Secondary hyperthyroidism
TSH secreting pituitary tumor

UST FMS MEDICAL BOARD REVIEW 2019 2 | SURGERY


HEAD AND NECK
IDA MARIE T. LIM, MD

RAIU

Less Common Forms


Iodide-induced thyrotoxicosis Variable but usually low (< 25%)
Thyrotoxicosis factitia Decreased (very low: < 2%)

Uncommon Forms
Pituitary tumors producing TSH Increased (mild to moderate:
25-60%)
Excess human chorionic gonadotropin (molar pregnancy/
choriocarcinoma) Increased (variable: 25-100%)
Pituitary resistance to thyroid hormone Increased (mild to
moderate: 25-60%)
Metastatic thyroid carcinoma Decreased
Struma ovarii with thyrotoxicosis Decreased

TSH Graves’ Disease


 Single best screening test for
hyperthyroidism EPIDEMIOLOGY
 In most outpatient clinical situations is the • Women > Men
most sensitive test for detecting mild • 20 – 50 years old ; rare in young children
(subclinical) thyroid hormone excess or • Annual incidence: 1 – 10% / 100,000
deficiency. • Affects all races
• Incidence correlates directly with the
amount of iodine in diet

Graves’ disease
• Most common cause of hyperthyroidism
• An autoimmune disorder characterized by
clinical hyperthyroidism due to thyrotropin
receptor antibodies (TRAbs) stimulate the
TSH receptor, increasing thyroid hormone
production.
• TRIAD (GET ) :
Goiter, Exophthalmos and Thyrotoxicosis
• With dermopathy: Plummer’s nail
;Localized or generalized
hyperpigmentation;
RAIU ( N= 5 – 20% ) Dermopathy (**pretibial myxedema);
Acropachy (triad of digital clubbing, soft-
Common Forms (85-90% of Cases) 24-Hour RAIU Over Neck*

Diffuse toxic goiter (Graves disease) Increased (moderate to high:


tissue swelling of the hands and feet and
40-100%) periosteal new bone formation);
Toxic multinodular goiter (Plummer disease)
moderate: 25-60%)
Increased (mild to Hyperhidrosis; Diffuse, nonscarring alopecia

Thyrotoxic phase of subacute thyroiditis Decreased (very low: < 2%)


Diagnostic findings:
Toxic adenoma Increased (mild to moderate: 25-60%)
• TSH suppressed
• T3 is increased disproportionately more than
T4 (T3 divided by T4 >20)
• RAIU is high (or normal in some cases)
• Uptake on RAI scan is diffused and
homogenous

UST FMS MEDICAL BOARD REVIEW 2019 3 | SURGERY


HEAD AND NECK
IDA MARIE T. LIM, MD

• Antithyroid antibodies usually are not


present.

Treatment options for Toxic nodular goiter


• Surgery is the treatment of choice
• 131I ablation may be used in patients who
are unsuitable for surgery

If thyroidectomy is chosen for treatment of


GD, how should it be accomplished?

- patients with GD undergoing thyroidectomy


should be rendered euthyroid with
methimazole. Potassium iodide should be given
in the immediate preoperative period

- In exceptional circumstances, when it is not


possible to render a patient with GD euthyroid
prior to thyroidectomy, the need for
thyroidectomy is urgent, or when the patient is
allergic to antithyroid medication, the patient
should be adequately treated with beta-block-
ade and potassium iodide in the immediate
preoperative period.

• Potassium iodide can be given as 5–7 drops


(0.25–0.35 mL) Lugol’s solution (8 mg
iodide/ drop) or
Toxic Nodular Goiter or Plummer’s disease • 1–2 drops (0.05–0.1 mL) SSKI (50 mg
iodide/ drop) three times daily mixed in
• one or more thyroid nodules trap and water or juice for 10 days before surgery.
organify more iodine and secrete more
thyroid hormone independently of TSH If surgery is chosen FOR TOXIC NODULAR
control. Toxic nodular goiter occurs most GOITER, how should it be accomplished?
often in areas of endemic goiter. If surgery is chosen as treatment for TMNG or TA,
• It has been documented that most “hot” or patients with overt hyperthyroidism should be
“autonomous” thyroid nodules have TSH- rendered euthyroid prior to the procedure with
receptor (common) or gsp (less common) methimazole pretreatment (in the absence of allergy
mutations. to the medication), with or without beta-adrenergic
• Thyrorotoxicosis is milder than in patients blockade. Preoperative iodine should not be used in
with Graves' disease this setting
• not accompanied by the extrathyroidal
manifestations of ophthalmopathy, pretibial MEDICAL MANAGEMENT OF
myxedema, vitiligo, or thyroid acropathy HYPERTHYROIDISM
• Ingestion or administration of iodides, e.g.,
iodine supplements or intravenously
administered contrast agents, may
precipitate iodine-induced hyperthyroidism
(Jod-Basedow phenomenon)

Toxic Multinodular Goiter


• older at presentation than those with
Graves' disease.
• The thyroid-gland goiters characteristically
have one or more nodules on palpation.
• Symptoms such as dysphagia and dyspnea
may be present. Some goiters are
retrosternal.
• Symptoms are often mild, and atrial
fibrillation in the elderly is frequently the Medical
Management:
only clinical finding apart from the goiter.  Euthyroid state is achieved in 4-6 wk
• diagnosis is suggested by the history and  Relapse rate in 12-18 months = 50%
physical examination and confirmed by  Side effects especially in prolonged use -
documenting a suppressed serum TSH level rashes, fever, neuritis, agranulocytosis,
and raised thyroid hormone level.  Crosses the placenta - fetal goiter
 Hypothyroidism
UST FMS MEDICAL BOARD REVIEW 2019 4 | SURGERY
HEAD AND NECK
IDA MARIE T. LIM, MD

 Patient compliance  Hematoma, seroma


 No morbidity related to surgery  Anesthetic and medical complication
 Treatment of choice for small goiters &
pregnant patients (PTU) THYROID STORM

Pharmacologic agents:  Seen in unprepared surgical patients


 Anti-thyroid drugs- inhibit hormonogenesis  Hyperthyroid pts undergoing non-thyroidal
within the thyroid gland. operation
-propylthiouracil-PTU  Signs and symptoms of thyrotoxicosis are
- decreases the peripheral magnified
conversion of T4 to T3  Management
- methimazole-Tapazole o prevention
- carbimazole-Neo-Mercazole o anti-thyroid drugs
 Beta-blockers- Propranolol o beta-blockers
-block the beta-adrenergic o steroids
peripheral manifestations of the
hyperthyroid state and decreases the II. NODULAR NONTOXIC GOITER
peripheral conversion of T4 to T3
1.Familial Goiter
RADIOACTIVE IODINE TREATMENT  Inherited enzymatic defect
 Standard dose - 10 mCi = 8500 cGy  Impaired iodine metabolism
 Cure rate is dose dependent  Usually associated with
5 mCi - 70% hypothyroidism
10 mCi - 87%
15 mCi - 96% 2.Endemic Goiter
 Iodine deficiency in diet
Effects:  Mountainous regions
 Hypothyroidism  Iodized salt
15% within 1 year 3.Sporadic goiter
3% every year thereafter  Iodine deficiency in diet
 Mountainous regions
Permanent  Iodized salt
 May aggravate exophthalmos - 33%
 Exacerbate thyrotoxicosis/thyroid storm Indications for Surgery
 Crosses placenta - no pregnancy for 1 year  Huge goiter which is cosmetically
 May cause infertility in women unacceptable
 May increase risk of cancer in children  Compression symptoms
 Contraindicated in pregnant & breast-  Suspicion of malignancy
feeding patients
 Ease of treatment III. Dominant or solitary nodule
 Highly effective esp. in diffuse goiters  Most are benign - colloid goiter or adenoma
 No morbidity related to surgery  5-15% are malignant: 2-3-fold increase if
 Treatment of choice for failed surgical nodule is solid
management  40% incidence in the presence risk factors -
 The effect is seen in 1.5 to 4 months low dose neck irradiation, (+) family history
 FNAB - most accurate diagnostic tool
SURGERY  Clinical management of thyroid nodules
should be guided by the results of US
 Complete and permanent control of toxicity evaluation and FNA biopsy
 Rapid control of symptoms  FNA-Positive Thyroid Nodule
 Removal of mass For a thyroid nodule with positive (malignant)
 Treatment of choice for huge goiters FNA results, surgical treatment is recommended
 Needs pre-operative preparation
 Overall morbidity - 1-2%  Use of suppressive therapy with
Thyroidectomy Indications: levothyroxine (LT4) may be considered in
 If it is the choice of the patient the following:
 Second trimester of pregnancy  Patients from geographic areas with iodine
 Failure (resistance or intolerance) of drug deficiency
therapy  Young patients with small thyroid nodules
 Poor compliance to drug therapy  Nodular goiters with no evidence of
functional autonomy
Morbidity related to Surgery  combined with aspiration if nodule is cystic
1. Injury to the recurrent and superior laryngeal
nerves  Use of LT4 therapy should be avoided in
 Hypothyroidism most cases and especially in the following:
 Hypoparathyroidism
UST FMS MEDICAL BOARD REVIEW 2019 5 | SURGERY
HEAD AND NECK
IDA MARIE T. LIM, MD

 Large thyroid nodules and goiters, - Thyroid hormone replacement for the
particularly in the presence of symptoms or hypothyroidism
signs of functional autonomy - Steroids
 Clinically suspicious lesions or lesions with - Symptomatic relief of pain, fever, etc
an inadequate cytologic sample
 Postmenopausal women and men older than Indications of Surgery in thyroiditis
60 years
 Patients with cardiovascular disease  To relieve compression
 If malignancy cannot be ruled out
Surgical treatment  Cosmetic indication
 Incidental finding at surgery
1. Associated local symptoms e.g. compression
2. Hyperthyroidism from a large toxic nodule, V. Thyroid gland anomalies
or hyperthyroidism and concomitant MNG
3. Growth of the nodule Thyroglossal duct cyst
4. Suspicious or malignant FNA results. Patient  Mid-line mass that moves with protrusion of
belongs to high risk group. Nodule is solid. the tongue
5. Failure of thyroid suppression  May result to:
1. Compression
Radioiodine 2. Most common complication. Secondary
infection.
Indications: 3. Fistula
1. For small goiters (volume <100 mL) 4. Malignancy - 1% (25% with focus in the
2. In those without suspected malignant thyroid gland)
potential
3. In patients with a history of previous Procedure: Sistrunk procedure
thyroidectomy
4. In those at risk for surgical intervention VI. THYROID CANCER

IV. Thyroiditis Incidence: (2005 Philippine Cancer Facts and


 Hashimoto’s Estimates
 De Quervain’s
 Riedel’s  9th most common for both sexes combined
 Acute Suppurative (3.3%), 15th leading site in men (1.5%) and
6th among females (5%)
Hashimoto’s thyroiditis  In 2005, estimated 3,521 new cases, 755 in
males and 2,766 in females.
Most common form of thyroiditis  There will be 1,012 deaths, 245 in males
- Chronic lymphocytic thyroiditis and 767 in females.
- Autoimmune thyroiditis  Most common cancer of women at ages 15-
- Thyroid autoantibodies 24
1. anti-thyroid peroxidase  Incidence among female residents in the
2. antithyroglobulin Philippines is among the highest observed
- Genetic predisposition worldwide
- May co-exist w/ papillary CA
Well differentiated type
Clinical Manifestations:  Papillary 80 %
: Affects women commonly  Follicular cancer 5 – 10 %
: Most frequent complaint - enlargement of the  Medullary cancer 5 – 9 %
neck w/ pain & tenderness in the Anaplastic type
region of the neck  1–2%
: most individuals are initially hyperthyroid and
subsequently becomes euthyroid or Clinical evaluation
hypothyroid History:
- diffuse & symmetric enlargement (80%) Exposure to ionizing radiation
(dental x-rays)
Diagnostic findings: Family history
Thyroid antibodies Presence of difficulty swallowing, breathing,
- Treatment voice changes
1. Diffuse goiter- LT4 suppression
2. No goiter no therapy Structural assessment
3. Nodular goiter- suppression &/or surgery  PHYSICAL EXAMINATION
4. Surgery - pressure symptoms, suspicion of  IMAGING TESTS:
malignancy, cosmetic ULTRASOUND
CT SCAN
 Generally, management is non-surgical MRI
UST FMS MEDICAL BOARD REVIEW 2019 6 | SURGERY
HEAD AND NECK
IDA MARIE T. LIM, MD

Salivary gland tumors


Diagnostic work up:
Fine needle aspiration Cytology:
- hard mass
- mass associated with neck nodes
- suspicious for malignancy

Surgical management
 Total thyroidectomy – surgical procedure
that removes entire thyroid gland
 Near total thyroidectomy – Removal of
nearly all of each thyroid lobe leaving
unresected only a small portion of the gland
adjacent to the entrance of the recurrent
laryngeal nerve into the larynx.
 Subtotal thyroidectomy – removal of most
but not all of each lobe of the thyroid
 Hartley Dunhill operation- removal of 1
entire lateral lobe with isthmus and
partial/subtotal removal of opposite lateral
lobe. It is done in nontoxic MNG.

Total thyroidectomy is recommended:


 Primary tumor > 1cm
 With risk factors- contralateral
nodules, regional or distant
metastases
 History of radiation therapy to the
head and neck area Superficial lobe Deep lobe
 First degree relative with DTC -lateral to the facial -medial to the facial
 Age > 45 even if without factors nerve nerve
above
-Makes up 80% of -Makes up the
Less than total thyroidectomy
the gland) remaining 20%
 Tumors < 1 cm; micropapillary
carcinoma; unifocal, intrathyroidal, -Presents as a pre -retromandibular or
low risk auricular mass; may parapharyngeal
 No prior head and neck irradiation also be infra- or post masses, with
 No clinically or radiologically auricular displacement of the
involved lymph nodes tonsil or soft palate
appreciated in the
Management of regional nodes throat.
 “prophylactic”- removal of nodes considered
normal pre or intraoperatively (by palpation
or imaging)
 “therapeutic”- removal of nodes likely to Location % Malignant %Benign
contain metastatic disease based on Parotid 20 % 70 -80%
palpation, imaging studies or biopsy. Submandibular 40 % 60 %
Sublingual/Minor 60 % 40 %
 Prophylactic neck dissection not
recommended Incidence of salivary gland cancer
 If node positive, the most conservative neck
dissection is warranted  0.3% of all cancers
 Central compartment neck  1 – 3 % of head and neck cancers
dissection- for positive central lymph (Spitz MR, Batsakis JG, 1984)
nodes
 Modified Radical neck dissection- if  10- 15% of all salivary tumors
with lateral compartment nodes
(Spiro RH, 1986)

Categories of malignant neoplasms


 Tumors of epithelial origin
Mucoepidermoid carcinoma
Adenoid cystic carcinoma
Adenocarcinoma
Acinica cell carcinoma
UST FMS MEDICAL BOARD REVIEW 2019 7 | SURGERY
HEAD AND NECK
IDA MARIE T. LIM, MD

Malignant mixed tumor


Epidermoid carcinoma Treatment
Other anaplastic carcinomas Benign tumor- superficial or total parotidectomyd
epending on location of tumor
 Tumors of non-epithelial origin
Sarcoma Malignant tumor
Lymphoma Treatment approach modified according to clinical
 Secondary tumors stage, tumor grade and histological type
Metastatic tumors
Factors affecting extent of treatment
Diagnosis: Extent of the lesion at diagnosis = clinical
stage
Clinical evaluation  Small -surgery alone
Diagnostic Imaging  Extensive local disease - surgery +
Preoperative biopsy RT
 Disseminated disease - palliative
Clinical evaluation Tx
History Surgical management
- age A. T1 and T2 (N0)
- onset ( acute or chronic) 1. Low grade - mucoepidermoid, acinic cell,
- duration/rapidity of growth adenocarcinoma
- symptoms associated with malignancy: • Parotid gland - Superficial lobectomy or
- Pain 12 – 24 % Total parotidectomy (Complete resection)
- Facial nerve dysfunction • Submandibular gland - excision
8 – 26 %
- Complete facial n. paresis 7 – 9 % B. T1 and T2
- Fixation to the masseter 17 % 1. High grade - mucoepidermoid, adenocarcinoma,
- Skin ulceration 9% adenoid cystic,malignant mixed, squamous,
undifferentiated carcinoma
Physical examination • Parotid – Complete resection by
 Consistency parotidectomy
 Fixation Neck dissection (N+); PORT
 Induration of overlying skin • Submandibular gland – Wide excision
 Presence of cervical nodes Level I neck dissection (N0) or
comprehensive neck dissection (N+)
Imaging tests Post-op RT
Not routinely indicated but may be useful in various
situations: C. T3 and T4 (N0 and N+), any grade
 Lesions fixed to adjacent bone • Parotid - Total parotidectomy with node
 Lesions which appear to involve sampling (NO) or total parotidectomy with
parapharyngeal space comprehensive neck dissection (N+); post-
 Minor salivary gland cancers arising in op RT
the palate, nasal cavity,nasopharynx • Submandibular gland – Wide
and paranasal sinus excision(N0) with extension to clear margins
(selective levels I, II, III); comprehensive
Preoperative biopsy (Role of FNAC) neck dissection (N+); post-op RT
 Routine use is not absolutely necessary
 Well defined parotid masses are generally Treatment of the Neck
treated with surgical resection whether  Incidence of nodal metastases
FNAB indicates benign or malignant disease Clinical cervical metastasis
 The extent of surgery is based on 14 % to 20 %
intraoperative findings rather than FNAB (Armstrong JG, 1992, Rodriguez-Cuevas S, 1995
results Spiro RH, 1975)

FNAC may be done for the ff reasons:  Occult lymph node metastases:
 Evaluating poorly defined salivary gland 12 % to 48%
masses (Kelley and Spiro,1996, Armstrong, 1992)
 Confirming suspicion of malignant disease in
order to counsel patients before surgery Principles
 Diagnosing metastatic carcinoma especially  cN+ - perform therapeutic neck dissection
with submandibular gland masses  cN- - elective neck dissection is not routinely
 Distinguishing surgically treatable from recommended
nonsurgical pathologic conditions
(lymphoma)
 Evaluating salivary gland masses in patients
who are poor surgical candidates
UST FMS MEDICAL BOARD REVIEW 2019 8 | SURGERY
HEAD AND NECK
IDA MARIE T. LIM, MD

Impact of nodal metastases Etiology:


 Nodal status is an independent predictor for
overall (p=.03) and disease –free survival Chemical Infective Environment
(p=.001)

Management of cNo neck: Tobacco Epstein-barr UV rays


Based on the likelihood of occult metastases: alcohol virus Radiation
 Histologic type Human Wood dust
 Histologic grade papilloma nickel
 Size
virus
 Clinical predictors
HIV
Features associated with nodal metastases:
Histologic cell type – high grade
Squamous cell
Adenocarcinoma
Mucoepidermoid
Undifferentiated
Primary tumor size
>4 cm- T3 lesions
Extraglandular extension

Why is there a need to electively treat the N0 neck?


 Informative for nodal staging. Need for
post-operative radiation
 Regional control is improved when removing
occult nodal disease
 Neck disease progresses unpredictably.
Observed N0 may progress directly to N2 or
N3
 The outcome is poor after neck recurrence. Management of Nodal metastases
Salvage rate is 12-30%
 Low morbidity 1. Management of occult nodal metastases

Surgical options:  HNSCC are characterized mainly by loco-


 Staging supraomohyoid neck dissection for regional progression and low risk of
selected (N0) with high grade lesions distant metastases, priority is given to
 Sampling of level I and II with intra-op FS – loco-regional control
 If positive, full neck dissection + RT  Although the natural history of the “primary
tumor” is a major consideration, the extent
Role of radiation therapy of metastatic cancer in the cervical nodes
 T3- T4 tumors determines locoregional control and
 Close or positive margins/incomplete survival
resection
 Bone involvement
 Perineural tumor
 Documented lymph node metastases
 Recurrent disease

Head and Neck Squamous cell cancer

Definition of terms
 Clinically positive neck- node > 1 cm,
spherical rather than flat ovoid, and harder
than nonmetastatic lymph node
Macrometastases- node which can be
identified either on PE or by imaging tests
 Clinically occult metastases – undetected by
clinical or radiographic exam;

UST FMS MEDICAL BOARD REVIEW 2019 9 | SURGERY


HEAD AND NECK
IDA MARIE T. LIM, MD

Also called subclinical metastases: Type 1


- Established occult metastases- detected by Modified Levels IJV,
Selective neck Primary site First echelon Radical neck 1,2,3,4,5 submandibular
dissection dissection type gland
Supraomohyoid Oral cavity 1,2,3 2
AnteroLateral(Jugular) Larynx,pharynx 2,3,4 Modified Levels The three
Central compartment Thyroid 4, central radical neck 1,2,3,4,5 structures are
compartment dissection type preserved;SCM,
tracheoesophageal 3 SAN, IJV
groove Selective Neck See table
superior dissection below
mediastinal
Supraomohyoid Parotid Preauricular Types of selective neck dissection and primary
Periparotid, site
intraparotid
Level 2,3 Controversy regarding management of the cN0
supraomohyoid Submandibular, 1,2,3 neck
lublingual  Observe:
gland tumor “wait and see” approach results in a high
posterolateral Scalp 2,3,4,5 proportion of patients developing metastatic
disease
 Elective Neck Dissection
Associated morbidity
light microscopy
High proportion of unnecessary surgery
- Occult micrometastases- subpathological or
Even if occult metastases are removed, does it
submicroscopic smaller than 2 mm deposit of
confer any survival advantage?
malignant cell; detected by IHC and/or molecular
analysis
Patients at risk for metastases
High T stage
2. Management of clinically positive neck
High T grade
Depth of infiltration
Site/location

Treatment of Regional Lymphatics


 Historical development:
1.Radical Neck Dissection- Crile,1906
2. Modified Neck Dissection- Suarez (1963);
Bocca and Pignataro(1967)
3. Selective Neck Dissection, Jesse
(1978)

ELND:
- from generalized en bloc resection to
focused surgery

Types of Neck Dissection


Name Levels Structures
removed removed
Classical Levels SCM, SAN, IJV
Radical neck 1,2,3,4,5,
dissection
Modified Levels SCM,IJV,
Radical Neck 1,2,3,4,5 submandibular
Dissection gland

UST FMS MEDICAL BOARD REVIEW 2019 10 | SURGERY


REVIEW TEST
IDA MARIE LIM, MD

HEAD AND NECK SURGERY REVIEW TEST

Choose the best answer:


_____6. Which is an autoimmune disorder
_____1. Which of the following is true regarding the characterized by the presence of thyroid stimulating
anatomy of the thyroid gland? immunoglobulins directed to the TSH receptor on
A. There are two lateral lobes connected the thyroid follicles leading to excess thyroid
centrally by a pyramidal lobe hormone secretion?
B. The average weight of the gland is 30 to A. Hashimotos’s thyroiditis
50 g B. Grave’s Disease
C. It has an abundant blood supply from 4 C. Jod basedow hyperthyroidism
major arteries D. Riedel’s thyroiditis
D. The parathyroid glands are closely Ans: B
related to it and usually found on the
anterolateral surface of the lobes
Ans: C _____7. What are the components of the classic
triad of Grave’s disease?
A. goiter, thyrotoxicosis, pretibial myxedema
_____2. Which of the following hormone is produced B. Thyrotoxicosis, exophthalmos, pretibial
by the thyroid follicles: myxedema
A. Thyroid stimulating hormone (TSH) C. Goiter, thyrotoxicosis, exophthalmos
B. Thyroid- released hormone (TRH) D. Thyrotoxicosis, exophthalmos,
C. Triiodothyronine tachycardia
D. calcitonin Ans: C
Ans: C
A 20-year-old college student sought consult
because of easy fatigability. She also experiences
_____3. Which of the following is correct regarding palpitations, irritability, and progressive weight loss.
thyroid hormone production? Her friends called her attention because of
A. The formation of thyroid hormones prominence of her eyes. P.E. revealed PR-120/min,
depends on an endogenous source of RR-15/min & BP-110/70 mm hg. There is a bilateral
iodine moderate uniform enlargement of the thyroid gland
B. Iodine is converted to iodide in the with thrill & bruit.
duodenum
C. Iodide is passively transported into the _____8. Your clinical impression is:
follicle A. Grave’s disease
D. Iodine is linked with tyrosine residues in B. Hashimoto’s thyroiditis
thyroglobulin C. Plummer’s disease
Ans: D D. subacute thyroiditis
Ans: A

_____4. Which is correct regarding physical _____9. The best treatment at this moment is /are:
examination of the thyroid gland A. antithyroid drugs
A. The landmark used in locating the B. immediate surgery
isthmus is the thyroid notch C. radioactive iodine ablation
B. A normal thyroid gland could be D. iodone
appreciated as a thin, soft mass Ans: A
anterolateral to the trachea
C. A stony hard mass within a thyroid _____10. Which among the ff. would be safest to
gland is characteristic of follicular cancer give in pregnant hyperthyroid patients?
D. Conditions such as Grave’s disease or A. propranolol
Hashimoto’s thyroiditis would present as B. steroids
diffuse bilaterally enlarged thyroid C. propylthiouracil
Ans: D D. methimazole
Ans: C

_____5. An accurate, cost effective test to different _____11. The PE findings of thrill and bruit in the
a benign from malignant thyroid nodule thyroid gland is indicative of
A. ultrasound A. nodular nature of the gland
B. Iodine scan B. hypervascularity
C. Fine needle aspiration biopsy C. inflammatory nature of the disease
D. Thyroid function test D. malignancy
Ans: C Ans: B

UST FMS MEDICAL BOARD REVIEW 2019 | SURGERY


REVIEW TEST
IDA MARIE LIM, MD

_____12. The tachycardia, arrhythmia and _____17. A 32 years old female presented with a
cardiovascular effects of excessive T3 & T4 is best slowly growing anterior neck mass located to the
treated by right of the midline. No other symptoms were noted.
A. digitalis PE -3x2 cm solid movable well defined mass. No
B. sedative agent palpable nodes. Next step in the management.
C. propranolol A. Serum TSH
D. calcium channel blocker B. Thyroid ultrasound
Ans: C C. Thyroid scan
D. fine needle biopsy
Ans: A
_____13. Who among the following is a suitable
candidate for radioactive iodine ablation therapy?
A. 20-year-old with diffuse toxic goiter in _____18. A fine needle cytology result of follicular
the first trimester of pregnancy lesion warrants …
B. 40-year-old with multinodular toxic goiter A. serum TSH check
C. 38 y/o with severe exophthalmos B. Thyroid ultrasound
D. 70 y/o patient with toxic goiter who have C. Thyroid scan
relapsed after surgery D. Surgery to remove involved lobe
Ans: C Ans: D

A 45-year-old lady underwent subtotal _____19. 35-year-old female diagnosed to have


thyroidectomy for a huge nodular nontoxic goiter. papillary thyroid cancer,2 cm right lobe. The left
The surgery was otherwise unremarkable except for- lobe and isthmus are normal. Best treatment option?
significant intraoperative bleeding. While in the A. Subtotal thyroidectomy
recovery room the patient complained of dyspnea; B. Total lobectomy, right and subtotal
the nurses informed you that they have changed the lobectomy left
wound dressing 6 times because of very significant C. Total lobectomy right with
bloody drainage isthmusectomy
D. Total thyroidectomy
_____14. You should: Ans: D
A. request for CT, BT and PTTA
B. Open the wound _____20. After a total thyroidectomy, radioactive
C. give for parenteral vitamin K iodine ablation may be useful as an adjuvant
D. give parenteral epsilon amino caproic treatment in which type of thyroid cancer?
acid. A. Medullary
Ans: B B. Hurthle cell
C. Papillary
D. Anaplastic
A 30 y/o patient post total thyroidectomy Ans:P C
complained of circumoral numbness and carpopedal
spasm.

_____15. Which of the following should be done


first:
A. check serum ionized Ca levels
B. give oxygen per cannula
C. start IV calcium replacement
D. check serum K and Na
Ans: C

_____16. Which of the following types of thyroid


cancer has the best prognosis?
A. Papillary
B. Follicular cancer
C. Hurthle cell cancer
D. Medullary cancer
Ans: A

UST FMS MEDICAL BOARD REVIEW 2019 | SURGERY


GI, ACUTE ABDOMEN
CHRISTINE JOYCE SANTICRUZ, M.D.

ESOPHAGUS – Posterior Partial Fundoplication


• “Toupet Fundoplication”
Gastroesophageal Reflux Disease • Developed to minimize the
• Typical Symptoms risk of post fundoplication
– Heartburn side effects (dysphagia,
– Regurgitation inability to belch, and
– Dysphagia flatulence)
• Characteristics of Defective “LES” • Consists of a 270°gastric
– 1. Mean resting pressure of <6 fundoplication around the
mmHg distal 4 cm of esophagus
– 2. Overall sphincter length of <2 cm
– 3. Intra-abdominal sphincter length • Gastroesophageal Reflux Disease
of <1 cm (most common cause of a – Anterior Partial Fundoplication
defective sphincter) • Alternative approach to
• Diagnosis: partial
– 24-hour pH Monitoring (Gold fundoplication
standard) • Various degrees have been
– Endoscopy described (90, 120, 180°)
• Complications: • Anterior 180° is more
– Esophagitis, Stricture, Barrett’s robust and has excellent
metaplasia long-term outcome
• Metaplasia (Barrett's Esophagus) – Collis Gastroplasty
– Esophagus lined with columnar • Esophageal lengthening
epithelium procedure done before
– Hallmark - presence of intestinal fundoplication
goblet cells • Reduces tension on the
– Occurs in 10 to 15% of patients gastro-esophageal junction
with GERD • Believed to minimize risk of
– Earliest sign for malignant failure of repair
degeneration • Controversial and not all
– 1/3 of patients Barret’s Esophagus surgeons choose to
presents with malignancy; most undertake a Collis
cases of adenocarcinoma of the procedure
esophagus arise in Barrett’s
epithelium Giant Diaphragmatic (Hiatal) Hernia
• Medical Therapy • Type I – Sliding Hernia
– Uncomplicated GERD - 12 weeks of – Upward dislocation of the cardia in
empiric treatment of antacid the posterior mediastinum
– Lifestyle changes – The phrenoesophageal ligament is
• Elevate the head of the bed stretched but intact
during sleep • Type II – Rolling Hernia or PEH
• Avoid tight-fitting clothing – Upward dislocation of gastric fundus
• Eat small, frequent meals; along a normally positioned cardia,
avoid eating prior to thru a defect in the
bedtime phrenoesophageal membrane
• Avoid alcohol, coffee, • Type III – Combined
chocolate, and peppermint Sliding-Rolling Hernia
• Surgical Therapy – Upward dislocation of both cardia
– Nissen fundiplication and gastric fundus
• Most common antireflux • Type IV
procedure – An additional organ, usually colon, is
• Done thru open or herniated as well
laparoscopic approach • Intrathoracic Stomach
• Consists of a – End stage of type I and type II
360°circumferential wrap hernias
distal esophagus using the
gastric fundus

UST FMS MEDICAL BOARD REVIEW 2019 1 | SURGERY


GI, ACUTE ABDOMEN
CHRISTINE JOYCE SANTICRUZ, M.D.

– Whole stomach migrates up into the – Weight loss


chest by rotating 180° around its – Chronic aspiration
longitudinal axis – Repetitive respiratory infection
• Diagnosis: • Diagnosis:
– Upright Chest X-ray – Barium swallow
– Upper GI Barium Study • Treatment:
• More accurately diagnose – Pharyngomyotomy: 2 cm or less
PEH than sliding hernia – Diverticulectomy/Diverticuopexy:
because the latter can often >2cm
spontaneously reduce;
diagnosis almost all of PEH Achalasia
– Fiber-optic esophagoscopy • Complete absence of peristalsis in the
• Complications: esophageal body and failure of LES
– Occult GI bleeding and ulceration of relaxation
the herniated stomach • Triad of Symptoms
– Gastric Volvulus (surgical – Dysphagia
emergency) – Regurgitation
• Borchardt’s Triad – pain, – Weight loss
nausea with inability to • Pathogenesis
vomit, and inability to pass – Neurogenic degeneration which is
NGT idiopathic or due to infection
• Treatment – Surgical • Diagnosis:
• Open or Laparoscopic Repair – Barium Esophagogram
• Important Principles • Dilated esophagus with a
– Reduce the hernia contents tapering ("bird's beak“)
– Excise the sac – Manometric Studies
– Mesh reduces recurrence rates of • Failure of the LES to relax
hernia is > than 8 cm • Treatment:
– Heller’s Myotomy
Schatzki’s Ring • Relieve functional
• Thin submucosal circumferential ring in the obstruction at the LES
lower esophagus at the squamocolumnar
junction; Often associated with a hiatal Diffuse and Segmental Esophageal Spasm
hernia • Characterized by substernal chest pain and
• Probably an acquired lesion that can lead to dysphagia
stenosis from chemical-induced injury by pill • LES usually shows a normal resting pressure
lodgment in the distal esophagus, or from and relaxation on swallowing
reflux-induced injury to the lower • Diagnosis:
esophageal – Manometric Studies
mucosa • Frequent occurrence of
• Symptoms simultaneous waveforms
– Brief episodes of dysphagia during and multipeaked
hurried ingestion of solid foods esophageal contractions
• Treatment: – Esophagogram
– Dilation alone • Corkscrew esophagus or
– Dilation with antireflux measures pseudodiverticulosis
– Antireflux procedure alone
– Incision/Excision of the Ring Nutcracker Esophagus
• “Supersqueezer esophagus”
Zenker’s Diverticulum • Most common primary esophageal motility
• Most common esophageal diverticulum disorders
• False diverticulum; due to weakness of the • Characterized by peristaltic esophageal
cricopharyngeal muscle at the Killian’s area contractions with peak amplitudes up to 400
• Clinical Features: mmHg
– Dysphagia • Treatment – aimed at the treatment of
– Regurgitation GERD
– Halitosis • Esophageal Diverticulum

UST FMS MEDICAL BOARD REVIEW 2019 2 | SURGERY


GI, ACUTE ABDOMEN
CHRISTINE JOYCE SANTICRUZ, M.D.

• Location – Proximal, mid, distal – Barium swallow – perforation


• Pathology contained within the mediastinum
– Pulsion: motor disorder and drains back into the esophagus
– Traction: inflammatory disorder – Mild symptoms
– Minimal evidence of clinical sepsis
Epiphrenic Diverticula • Approach:
– Terminal 3rd & usually adjacent to – Hyperalimentation
the diaphragm – Antibiotics
– Considered as "pulsion" diverticula – Cimetidine – decrease acid secretion
– False diverticulum & diminish pepsin activity

Esophageal Diverticulum Mallory Weiss Syndrome


• Midesophageal or Traction Diverticula • Longitudinal tear in the mucosa of the GEJ
– True diverticulum • Acute upper GI bleeding caused by forceful
– Patients who had mediastinal LN vomiting and/or retching; commonly seen in
involvement with tuberculosis, alcoholics
fungal infections, lymphoma, • Massive arterial bleeding
orsarcoid • Mechanism
– An acute increase in intra-abdominal
Esophageal Perforation pressure against a closed glottis in a
• True emergency; most commonly occurs patient with a hiatal hernia
following diagnostic or therapeutic • Diagnosis – Requires high index of suspicion
procedures – Upper Endoscopy – longitudinal
• Boerhaave's Syndrome – Spontaneous fissures in the mucosa of the
perforation herniated stomach as the source of
• Most common location – left lateral wall, bleeding
just above the GEJ • Treatment
• Clinical Features: – Decompression
– Chest pain (very striking and – Antiemetics
consistent symptom), fever, – Bleeding will stop 90% of the time
tachycardia, subcutaneous spontaneously with nonoperative
emphysema, dysphagia, dyspnea management
• Diagnosis:
– Contrast Esophagogram Caustic Injury
• (+) extravasation • Alkalies dissolve tissue, penetrate more
(diagnostic) deeply
– Chest X-ray • Acids cause coagulative necrosis limits
• Air or effusion in pleural penetration
space • Phases of Injury
• Mediastinal or cervical – Acute necrotic phase
emphysema • Lasts 1 to 4 days after
• Treatment: injury
– Mortality related to time interval • Coagulation of intracellular
between perforation and treatment; proteins results in cell
key to optimum management is necrosis
early diagnosis. • Tissue around necrosis is
– Most favorable outcome – primary inflammed
closure of perforation within 24 – Ulceration and Granulation phase
hours results in 80 to 90% survival • 3 to 5 days after injury
• Non-operative Management • Quiescent period, symptoms
– Usually follows an injury occuring disappear
during dilation of esophageal • Necrotic tissue sloughs,
strictures or pneumatic dilations of leaving an ulcerated base
achalasia • Period esophagus is the
• Indication: weakest
– Cicatrization and Scarring

UST FMS MEDICAL BOARD REVIEW 2019 3 | SURGERY


GI, ACUTE ABDOMEN
CHRISTINE JOYCE SANTICRUZ, M.D.

• Begins 3rd week following • Cause: imbalance between mucosal


injury defenses and acid/peptic injury
• Connective tissue begins to • Curling’s Ulcer
contract, resulting in – Peptic ulcers formed after severe
narrowing of the esophagus burn injury
• Characterized by dysphagia • Cushing’s Ulcer
• Efforts must be made to – Peptic ulcers formed after severe
reduce stricture formation brain damage
• Diagnosis: • Risk Factors
– Early esophagoscopy is advocated – H. Pylori – both gastric and
• Treatment: duodenal (duodenal > gastric)
– Grade 1 – observation – NSAID
– Grade 2 and up – Surgical Therapy – Smoking
– Stress
Esophageal Carcinoma – Blood type
• Squamous carcinoma – majority of – Type A personality
esophageal cancers worldwide • Indications for Surgical Treatment
• Most common sx – dysphagia – Bleeding
• Risk factors: – Perforation
– Nitroso compounds – Obstruction
– Zinc & molybdenum deficiency – Intractability or non-healing ulcers
– Smoking (squamous CA) • Non-healing PUD
– Alcohol (squamous CA) – Rare indication
– Achalasia, Lye strictures – Consider possible differentials for
– Tylosis, HPV nonhealing PUD first
– Barrett’s esophagus (more
adenocarcinoma) Highly Selective Vagotomy or Proximal
• Staging: Gastric Vagotomy
– Stage 0: in situ, high-grade – mortality risk <0.5%
dysplasia, no LN mets – Minimal side effects
– Stage I: invaded lamina propria Taylor Procedure
– Stage IIA: invaded esophageal wall – Posterior truncal vagotomy and
but not surrounding structures anterior seromyotomy
– Stage IIB: LN (+); primary tumor Vagotomy + Drainage
has only invaded submucosa or – Truncal vagotomy + bypass or
muscularis propria ablation of the pylorus
– Stage III: invaded the adventitia • Pyroplasty, Truncal
and surrounding structures vagotomy and pyroplasty,
– Stage IV: (+) metastasis Truncal vagotomy and
• Diagnosis: gastrojejunostomy
– Barium esophagogram Vagotomy and Distal Gastrectomy
– Endoscopy & Endoscopic UTZ
– Chest/Abdomen CT Scan Zollinger – Ellison Syndrome
• Treatment: • Uncontrolled gastrin secretion; most
– Stereotactic radiation with common symptom – epigastric pain, GERD
concomitant chemotherapy and diarrhea
– Curative resection • Gastrinoma Triangle (Pasaro’s triangle)
• Ivor-Lewis Procedure – 90% of ZES tumors
• Left thoracoabdominal – Junction of cystic & CBD, confluence
approach of D2 & D3 junction of body and eck
• Transhiatal resection of pancreas
stomach • Diagnosis:
– Fasting gastrin of 1mg/L, BAO >15
Peptic Ulcer Disease mEq/h or >5 mEq/h
• Defects in gastric or duodenal mucosa – Secretin Stimulation Test
extending to submucosa or deeper • Treatment:
– Surgical resection of gastrinoma

UST FMS MEDICAL BOARD REVIEW 2019 4 | SURGERY


GI, ACUTE ABDOMEN
CHRISTINE JOYCE SANTICRUZ, M.D.

– In MEN 1, perform – Goal: negative margin at least 5 cm


parathyroidectomy before resection
of gastrinoma Gastrointestinal Stromal Tumor
– PPI for symptomatic relief • Submucosal solitary slow growing tumors
• Arises from interstitial cells of Cajal (ICC)
Malignant Neoplasms of The Stomach • 2/3 of all GISTs occur in the stomach
• Three most common gastric malignancy • Defining Feature
– Adenocarcinoma (95%) – Mutation of proto-oncogene KIT
– Lymphoma (4%) – Leads to persistence of cellular
– Malignant GIST (1%) growth
• Metastasis
Gastric Adenocarcinoma – Hematogenous route
• Risk Factors: – Most common liver & lung
– Pernicious anemia, blood group A, • Diagnosis:
(+) FH, smoked food, nitrates, H. – Endoscopy and biopsy
Pylori, smoking, EBV infections • Treatment:
• Alcohol has NO ROLE in gastric malignancy – Wedge resection with clear margins
• Protective – aspirin is adequate
• Premalignant Lesions – Imatinib
– Polyps • Chemotherapeutic agent
• Hyperplastic and adenomas that blocks c-kit
– Atrophic gastritis • Reserved for metastatic or
• Most common precancerous unresectable GIST
lesion
– Intestinal Metaplasia Benign Gastric Neoplasms
• Can be caused by H. pylori • Polyp
• Clinical Manifestations – Most common benign tumor of the
– Weight loss, anorexia and early stomach
satiety • Five Types
– Acute GI bleeding (unusual) – Adenomatous – malignant potential
– Dysphagia – if the tumor involves (10-15%)
the cardia – Hyperplastic – most common polyp;
• Paraneoplastic Syndromes low malignant potential
– Trousseau's syndrome – Hamartomatous
(thrombophlebitis), acanthosis – Inflammatory
nigricans (hyperpigmentation of the – Heterotopic
axilla and groin), or peripheral • Polyps that are symptomatic, >2 cm, large
neuropathy hyperplastic or adenomatous should be
• Virchow's Node removed, usually by endoscopic snare
– Enlarged left supraclavicular lymph polypectomy.
node
• Sister Joseph’s Nodule Gastric Volvulus
– Palpable umbilical nodule • Twist of stomach that usually occurs in
– Pathognomonic for advanced association with a large hiatal hernia or
disease unusually mobile stomach without hiatal
• Blumer’s Shelf Node hernia
– Palpable nodule in pouch of Douglas • Clinical Manifestation
– Evidence of drop metastasis – Abdominal pain and pressure
• Most important prognostic factor – Dyspnea
– LN involvement – Palpitations
– Depth of tumor invasion – Dysphagia
• Diagnosis: • Management – Vomiting and passage of a
– Endoscopy and biopsy NGT may relieve symptoms
– Abdominal CT Scan • Gastric Infarction – is a surgical emergency
• Treatment:
– Surgery is the only curative
treatment for gastric cancer

UST FMS MEDICAL BOARD REVIEW 2019 5 | SURGERY


GI, ACUTE ABDOMEN
CHRISTINE JOYCE SANTICRUZ, M.D.

SMALL INTESTINE • Due to a neurologic dysfunction, electrolyte


• Principle site of nutrient digestion and abnormality and advanced age
absorption. • Treatment:
• Layers – Fluid resuscitation, Antibiotics, NGT
– Mucosa decompression
– Submucosa – greatest suture – IV Neostigmine, IV Atropine
holding
– Muscularis propria Intestinal Fistulas
– Serosa – water tightness • Abnormal communication between two
• Peyer’s Patches epithelialized surfaces; (+) drainage of
– Commonly located in the ileum enteric material through abdominal wound
– Lymphoid follicles & local source of or through existing drains
IgA – Internal – within GI tract or
• Jejunum vs Ileum: jejunum has larger adjacent organs
circumference, thicker walls, less fatty – External – to external environment
mesentery, and longer vasa recta • 80% due to iatrogenic complications
• Classification
Small Intestinal Obstruction – Low Output Fistulas – <200 mL/day
• Most frequently encountered surgical – High Output Fistulas – <500 mL/day
disorder of the small intestine • Usually become clinically evident between
• Most common cause post-op adhesions the 5th & 10th postop
(75% of cases) • Diagnosis:
– Hernias, malignancy, Crohn's – CT scan – most useful initial test
disease – Small bowel series
• Congenital Abnormalities – Fistulogram – localize fistula origin
– Midgut volvulus and intestinal • Treatment:
malrotation – Maximize spontaneous closure
• Partial SBO • Usually do 2 to 3 months of
– Portion of the intestinal lumen is conservative therapy
occluded, allowing passage of some – Surgery – failure of spontaneous
gas and fluid. closure
• Closed Loop obstruction – Rehabilitation
– Segment obstructed proximally and
distally progression to strangulation. Small Bowel Neoplasms
• Diagnosis: • Adenomas – most common benign
– SFA neoplasm
• Sensitivity 70-80% • Most common location – duodenum
• Dilated small bowel loops • Primary Small Bowel Cancers
(>3 cm), air-fluid levels, – Rare (1.1 to 2.4% of GI
and a paucity of air in colon malignancies)
– CT Scan – Adenocarcinomas (35-50%)
• Sensitivity 80-90%; – Carcinoid tumors (20-40%)
Specificity 70-90% – Lymphomas (10-15 %)
• Treatment: • Clinical Presentation
– Fluid resuscitation, Antibiotics, NGT – Partial SBO – most common
decompression presentation
– Complete SBO – Surgery – Only becomes symptomatic when it
– Partial SBO with no fever, becomes large
tachycardia, tenderness, or incresed • Diagnosis – rarely are diagnosed
WBC, approach conservatively preoperatively
– No improvement within 48 hours – • Treatment: Surgical Resection
Surgery.
• Ogilvie’s Syndrome Meckel’s Diverticulum
– Massive idiopathic, non-obstructive • Most prevalent congenital anomaly of the GI
dilatation of the colon tract
– Acute colonic pseudo-obstruction • True diverticula; 60% contain heterotopic
mucosa

UST FMS MEDICAL BOARD REVIEW 2019 6 | SURGERY


GI, ACUTE ABDOMEN
CHRISTINE JOYCE SANTICRUZ, M.D.

– Gastric mucosa – is most common – (+) Complications – segmental


(60%) intestinal resection for diverticula
– Pancreatic tissue – 2nd most located in the jejunum or ileum
common
• “Rule of 2s” Intussuception
– Occurs in 2% of the population • Segment of intestine is drawn into the
– Has a 2:1 male:female ratio lumen of the proximal segment
– Discovered by 2 years of age • Presentation – intermittent abdominal pain
– Located 2 feet (60 cm) from the and signs of bowel obstruction
ileocecal valve • Diagnosis:
– Commonly 2 cm in diameter and 2 – CT Scan
inches (5 cm) long • Diagnostic of choice
– Can contain 2 types of heterotopic • “Target sign”
mucosa • Treatment:
• Littre’s hernia - Meckel's diverticula found in – Surgical resection
an inguinal or femoral hernial sac
• Clinical Presentation: LARGE INTESTINES
– Bleeding – pediatric
– Intestinal obstruction – adults Diverticular Disease
– Diverticulitis • Diverticulosis
• Diagnosis: – Diverticula without inflammation
– CT scan – low sensitivity and – Sigmoid colon is most common site
specificity – Common symptom is massive lower
– Enteroclysis – 75% accuracy; not GI bleed
applicable during acute • Diverticulitis
presentations – Inflammation and infection
– Radionuclide Scans (99mTc- associated with diverticula
pertechnetate) – positive only when – Uncomplicated diverticulitis – left-
the diverticulum contains associated sided abdominal pain, with or
ectopic gastric mucosa without fever, mass and
• Treatment: leukocytosis
– Diverticulectomy – Complicated diverticulitis – abscess,
– Segmental Ileal Resection – for obstruction, diffuse peritonitis, or
bleeding diverticula fistulas
• Hinchey Staging
Acquired Diverticulum 1. Stage I: colonic inflammation with
• False diverticula an associated pericolic abscess
• Common in the duodenum, near the 2. Stage II: colonic inflammation with
ampulla a retroperitoneal or pelvic abscess
• Pathophysiology – abnormalities of intestinal 3. Stage III: purulent peritonitis
smooth muscle motility, herniation of 4. Stage IV: fecal peritonitis
mucosa and submucosa through weakened • Diagnosis:
• Clinical Presentation: 1. CT Scan – pericolic soft tissue
– Asymptomatic unless associated stranding, colonic wall thickening,
complications arise and/or phlegmon
– Complications – intestinal • Treatment:
obstruction, diverticulitis, 1. Uncomplicated diverticulitis – broad-
hemorrhage, perforation, and spectrum oral antibiotics for 7-10
malabsorption. days & low-residue diet; failure to
• Diagnosis: improve within 48-72 hours
– Mostly discovered incidentally indicates abscess formation
– Upper GI Series – most sensitive 2. Small abscesses (<2 cm diameter) –
test treated with parenteral antibiotics
• Treatment: 3. Larger abscesses – best treated with
– Asymptomatic – observe CT-guided percutaneous drainage

UST FMS MEDICAL BOARD REVIEW 2019 7 | SURGERY


GI, ACUTE ABDOMEN
CHRISTINE JOYCE SANTICRUZ, M.D.

• If 2nd episode of uncomplicated diverticulitis – Rare autosomal dominant, errors in


or 1st episode of complicated diverticulitis: mismatch repair
elective sigmoid colectomy is recommended – Colorectal carcinoma at an early age
(40 to 45 years old)
Colorectal Adenocarcinoma – Synchronous or metachronous
• Most common malignancy of the GIT colorectal carcinoma – 40%
• Risk Factors: – Associated with endometrial (most
1. Aging – incidence increases after 50 common), ovarian, pancreas,
yo. stomach, small bowel, biliary, and
2. Known FH of cancer urinary tract carcinomas
3. Diet – high in animal fat and low in • Screening Colonoscopy
fiber – Annually for at-risk patients
4. Inflammatory bowel syndrome beginning at either age 20 to 25
5. Cigarette smoking years or 10 years younger than the
• Polyps youngest age at diagnosis in the
– Non-neoplastic polyps (no malignant family, whichever comes first
potential) • Familial Colorectal Cancer
– Hyperplastic polyp, Pseudopolyps, – 10 to 15% of patients with
– Hamartomas - not considered to be colorectal cancer
premalignant – Screening Colonoscopy every 5
• Familial Juvenile Polyposis years at age 40 or 10 years before
– Hundreds of polyps in the colon and the earliest diagnosed patient in the
rectum pedigree
– Degenerates into adenomas- • Routes of Spread & Natural History
carcinoma – Regional lymph node involvement is
• Peutz Jeghers Syndrome the most common form of spread
– Polyposis of the small intestine, – T stage (depth of invasion) is the
colon and rectum single most significant predictor of
– Melanin spots on the buccal mucosa lymph node spread
and lips – Number of lymph nodes metastases
• Cronkite-Canada Syndrome correlates with presence of distant
– GI polyposis + alopecia + cutaneous disease and inversely with survival
pigmentation + atrophy of the – 4 or more involved lymph nodes
fingernails and toenails predict a poor prognosis
• Cowden Syndrome – Most common distant metastasis is
– Hamartomas of all three embryonal the liver
cell layers; facial trichilemmomas, • Clinical Presentation – change in bowel
breast cancer, thyroid disease, and habits, rectal bleeding, melena, unexplained
GI polyps anemia, or weight loss
• Familial Adenomatosis Polyposis • Screening:
– ~1% of all colorectal – Annual DRE at age 40
adenocarcinomas – FOB at age 50
– Mutation in APC gene, develop – Flexible signoidoscopy every 5
hundreds to thousands of years at age 50
adenomatous polyps shortly after – Colonoscopy if with risk factors
puberty • Treatment – colonic resection
– Risk of colorectal cancer approaches – Colectomy
100% by age 50 years – LAR – tumors 5-10 cm from anal
– Flexible sigmoidoscopy of first- verge
degree relatives beginning at age 10 – APR – tumors <5 cm from ananl
to 15 years verge
– Associated with congenital
hypertrophy of the retinal Volvulus
pigmented epithelium, desmoid • Air-filled segment of colon twists about its
tumors, epidermoid cysts mesentery
• HNPCC or Lynch Syndrome • Involves sigmoid colon (90%), cecum
(20%), or transverse colon

UST FMS MEDICAL BOARD REVIEW 2019 8 | SURGERY


GI, ACUTE ABDOMEN
CHRISTINE JOYCE SANTICRUZ, M.D.

• May reduce spontaneously; More commonly – 2. Transsphincteric: extends


produces bowel obstruction through both the internal and
• Can progress to strangulation, gangrene, external sphincters
and perforation – 3. Suprasphincteric: tracks around
• Diagnosis: the entire external sphincter
– Bent inner tube or coffee bean sign – 4. Extrasphincteric: tracks around
– Sigmoid colon both sphincters to exit laterally
– Northern clearance sign • Goodsall’s Rule
– Transverse colon – If external opening is anterior
• Treatment: totransverse anal line, the fistula
– Resuscitation followed by runs directly into the anal canal. If
endoscopic detorsion the external opening is posterior,
– Clinical evidence of gangrene or the tract curves to the posterior
perforation mandates immediate midline
surgical exploration • Treatment:
– Fistulotomy
Hemorrhoids – Fistulectomy
• Cushions of submucosal tissue containing – LIFT
venules, arterioles, and smooth-muscle
fibers
• Internal Hemorrhoid Graded:
– 1st degree: bulge into the anal
canal on straining
– 2nd degree: prolapse through the
anus but reduce spontaneously
– 3rd degree: prolapse through the
anal canal and require manual
reduction
– 4th degree: prolapse cannot be
reduced
• Treatment:
– Dietary fiber, stool softeners,
increase fluid intake, avoid straining
– Rubber band ligation – 1st & 2nd
degree
– 4th degree and thrombosed
hemorrhoids – perform excision

Anal Fissure
• Tear in the anoderm distal to the dentate
line
• 90% - posterior midline
• Clinical Manifestation:
– Tearing pain with defecation and
hematochezia
• Treatment:
– Lubricants, warm sitz bath and bulk
laxatives
– Surgery – lateral internal
sphincterectomy

Fistula in Ano
• Classification:
– 1. Intersphincteric (most common):
tracks through the distal internal
sphincter and intersphincteric space

UST FMS MEDICAL BOARD REVIEW 2019 9 | SURGERY


BREAST
MICHAEL ALAY-AY, M.D.

Lymphatic drainage 1. 40 % reduction for Stage II disease


2. 30 % increase in survived in patient found
1. Level I- nodes located lateral to or below to have CA
the lower border of the p. minor
2. Level II- nodes located deep to or behind Ultrasound
the p. minor Value:
3. Level III- nodes located medial to or above 1. Resolution of equivocal mammography
the upper border of the p. minor 2. Diagnosis of cystic disease
3. Demonstration of solid abnormalities with
- Most women who present to a surgeon for specific echogenic features
evaluation of a breast problem do not have
cancer. For both patient and physician faced Magnetic Resonance Imaging
with any breast problem, differentiation - Major advantage: detects small tumors
between benign or malignant is the most without subjecting the patient to any
important issue. radiation
- Drawback: technique is expensive (P15000)
Gynecomastia compared with P1800 for conventional
- Presence of a female type mammary gland mammography.
in the male - More sensitive than mammography for
- Excess of estrogen in relation to circulatory discovery of small tumors (high risk
testosterone patients)
- Does not predispose the male breast to CA
Inflammatory and Infectious Disorders
Gynecomastia Bacterial Infectious
1. Pathophysiologic Mechanism of 1. Acute Mastitis
Gynecomastia - Puerperal or lactational mastitis
2. Estrogen excess states - Most common offending organisms: S.
3. Androgen deficiency states aureus
4. Drug-related conditions that initiate - Infection tends to be localized &
gynecomastia suppurative

Systemic disease with idiopathic mechanism Lactational mastitis


Treatment: Acute Mastitis
1. Medical therapy- Tamoxifen Treatment
2. Subcutaneous mastectomy- if refractory to 1. Allow lactation to continue to allow for the
medical tx stagnant milk to come out
2. Presuppurative - antibiotic
Mammography 3. Suppurative - Surgical drainage
Useful for:
1. Examinations of an indeterminate mass that Chronic Mastitis
presents as a solitary lesion suspicious of CA - Recurrent abscess formation
2. Examination of an indeterminate mass that - Multiple skin sinus
cannot be considered a dominant nodule - Simulate breast CA
3. Follow-up examination of breast cancer - Most common offending organisms: TB bacilli
treated by conservative surgery
4. Follow-up of examinations of contralateral Treatment:
breast following mastectomy for CA 1. Biopsy to r/o CA
5. Evaluation of the large fatty breast in the 2. Anti-koch’s regimen
symptomatic patient TB mastitis

Mammography Benign Lesions


Findings suggestive of CA 1. Fibrocystic change/disorders
1. Presence of a mass with spiculated or - Localized estrogen sensitivity
irregular margins - Diffuse, often bilateral breast pain
2. Presence of fine stippled calcification - Pain accentuated just before menstruation
- Not a risk factor for breast CA
Mammography - May present as a single dominant cyst
American Cancer Society recommendation
- Baseline mammography at age 55 and every Fibrocystic Change/Disorders
1-2 year and annual clinical BE Treatment
- Screening mammography for those at RISK 1. Analgesics
at Age 30-39 2. Danazol/Tamoxifen- anti-estrogen: 60% cure rate
3. Caffeine free diet
Mammography 4. Cessation of smoking
Prospective randomized studies of routine 5. Dominant cyst- aspiration
mammographic screening confirms:

UST FMS MEDICAL BOARD REVIEW 2019 1 | SURGERY


BREAST
MICHAEL ALAY-AY, M.D.

Benign Lesions Incidence starts rising steeply at age 30


2. Fibroadenoma
- 2nd-3rd decade Breast Cancer
- Relationship to estrogen sensitivity Etiology
- Well-defined, rubbery mass 1. Family history
- No premenstrual tenderness 2. High dietary fat intake/obesity
- Stop growing when they reach 2-3 cms. 3. Late menopause (> 45 y/o)
- No risk for malignancy 4. Infertility & nulliparity
5. History of primary breast CA
Fibroadenoma 6. Irradiation to chest wall
Treatment: 7. Germs- Line mutations
1. FNAB 1.) BRCA 1 Chrom. 17q21
2. Observation <25 y/o 2.) BRCA 2 Chrom. 13q12-13
3. Excision biopsy >25 y/o 8. Certain breast diseases
a. Atypical hyperplasia
Benign Lesions b. Lobular carcinoma in situ
3. Intraductal papilloma c. Ductal carcinoma in situ
- Most common cause of a bloody nipple
discharge (70%) WHO 2012
- Located beneath the areola Measures to reduce risk for breast cancer
- Usually solitary and <1 cm. in size Early pregnancy- 18-25 y/o
- Increased risk for CA for multiple Multiparity- 7 children
papilloma Breast feeding- 2 years
- Treatment: Excision BMI- 18-20

Benign Lesions Breast Cancer


4. Fat Necrosis Natural history
- Usual history of trauma to the chest or Untreated patients
breast 5 yr. SR- 18 %
- Affects women with pendulous breast 10 yr. SR- 3.6
- Mass and skin dimpling Metastatic disease- most common cause of death
- Mammography-Coarse calcification Affects the age group 45-65 y/o
- No risk for cancer 40-50 % are located in the upper outer quadrant
- Treatment: Excision Hard, non-tender mass on presentation
Skin dimpling, nipple retraction, bloody nipple
Phyllodes Tumor discharge, palpable axillary lymph nodes
- Present at a median age of 50
Mammographically indistinguishable from Breast Cancer
fibroadenoma
previously called Cystosarcoma Phylloides Axillary LN are involved progressively from Level I to
mesenchymal and epithelial components Level II to Level III
rapid growth Nodal status-most important prognostic correlate for
Phyllodes tumor recurrent disease & survival
Phyllodes tumor Number of positive axillary lymph nodes and location
Phyllodes tumor of the positive axillary lymph node
Benign-90%
Borderline BREAST CANCER
Malignant Breast Cancer
Sites of Distant Metastases
Treatment of Phyllodes Tumor 1. Bone
wide local Excision 2. Lung
3. Pleura
axillary dissection usually not necessary 4. Soft tissue
5. Liver
Adjuvant treatment
Benign- no adjuvant treatment Breast Cancer
Borderline- +/- radiotherapy Biopsy
Malignant-Radiotherapy 1. Palpable lesions
1. Incisional bx
Limited role of chemotherapy, hormonal therapy 2. Excisional bx
3. Tru-cut biopsy
BREAST CANCER FACTS AND FIGURES 4. Fine needle aspiration
Philippine figures – 2008* 2. Non palpable lesions
No. 1 cancer in the country - Localization followed by biopsy
Median survival among females is 60 months
Survival at the 5th year is 50.10% and 32.38% at
the 10th year
UST FMS MEDICAL BOARD REVIEW 2019 2 | SURGERY
BREAST
MICHAEL ALAY-AY, M.D.

- 25-70% risk for invasive CA ave. of 5 years


Stages are Defined by TNM Classification - Future cancers are observed in the ipsilateral
Primary Tumor (T) breast and in the same quadrant
Tx Primary tumors cannot be assessed
To No evidence of primary tumors Breast Cancer
TIS Carcinoma in situ Histopathology
T1 Tumor 2 cm.or less is greatest dimension B. Infiltrating Malignancies
T2 Tumor > 2 cm. but < 5 cm. 1. Paget’s Disease of the nipple
T3 Tumor > 5 cm. - Chronic eczematoid eruption of the nipple
T4 Tumor of any size with direct extension to - Good prognosis
chest wall of skin; includes
inflammatorycarcinoma Breast Cancer
Histopathology
TNM Staging 2. Infiltrating Ductal Carcinoma w/ productive
Regional lymph node (N) fibrosis
Nx Regional lymph nodes cannot be - Most common form of breast CA (75-80%)
assessed - Affects the 40-60 years old age group
N0 No regional lymph node metastasis - Commonly presents as a solitary,
N1 Metastasis to movable ipsilateral hard, non-tender, ill-defined mass
axillary lymph node (s) - Profound desmoplastic response- skin
N2 Metastasis to ipsilateral axillary lymph dimpling
nodes fixed to one another or to other - Important feature is multicentricity
structure Breast Cancer
N3 Metastasis to ipsilateral internal Histopathology
mammary lymph nodes; 3. Medullary carcinoma
supraclavicular lymph node - Soft, hemorrhagic bulky mass
- Better 5 yr. SR than invasive ductal or
TNM Staging lobular CA
Distant Metastases
Mx Presence of distant metastases cannot be Breast Cancer
assessed Histopathology
M0 No distant metastases 4. Mucinous Carcinoma (Colloid CA)
M1 Distant metastases - Presents as a bulky, mucinous tumor
- Cut surface glistening & gelatinous
TNM Staging
Stage 0 Tis No Mo Breast Cancer
Stage I T1 No Mo Histopathology
Stage 2A To N1 Mo 5. Tubular Carcinoma
T1 N1 Mo - Most differentiated variant of breast CA
T2 No Mo - 100 % long term survival rate
2B T2 N1 Mo
T3 No Mo Breast Cancer
Stage 3A To N2 Mo Histopathology
T1 N2 M1 6. Papillary Carcinoma
T2 N2 Mo - Lowest frequency of axillary nodal
T3 N1,N2 Mo involvement
3B T4 Any N Mo - 5 & 10 yr. SR approaches Tubular CA
3C Any T N3 Mo
Stage IV Any T Any N M1 Breast Cancer
Histopathology
Breast Cancer 7. Lobular Carcinoma
Histopathology - originate from terminal ductules of the
A. Non infiltrating (In Situ) Carcinoma of Ductal lobule
and Lobular Origin - high propensity for bilaterality,
1.Lobular CA in Situ multicentricity & multifocality
observed only in females
> 90 % are premenopausal Breast Cancer
90 % ER positive Histopathology
25-35 % will develop breast CA ave. of 25 yrs 8. Inflammatory Carcinoma
majority of future invasive CA are ductal - Carries the worst prognosis
important features : multicentricity & bilaterality - Features of erythema, peau d’ orange
& skin ridging w/ or w/o a palpable
mass
Breast Cancer - Subdermal lymphatics and vascular
Histopathology channels are permeated w/ foci of highly
2. Ductal Carcinoma in Situ undifferentiated tumor
- Among menopausal women
UST FMS MEDICAL BOARD REVIEW 2019 3 | SURGERY
BREAST
MICHAEL ALAY-AY, M.D.

Breast Cancer Breast Cancer


Treatment Treatment
Local Control A. In Situ Disease
1. Surgery 1. LCIS-Lobular carcinoma in situ
2. Radiation - Marker for increased risk rather than
Systemic Control an inevitable precursor of invasive
1. Chemotherapy disease
2. Hormonal therapy - Options:
3. Biologic therapy 1. Observation
2. Bilateral mastectomy with delayed or
BREAST CANCER immediate reconstruction
Historical Perspectives
Halsted (1882)- Classical Radical Breast Cancer
Mastectomy Treatment
Removes: 2. DCIS - Ductal carcinoma in situ
1. Whole breast - Precursor of invasive disease
2.Overlying skin - Options:
3. Axillary LN 1. Total mastectomy - gold standard
4. Pectoralis muscles 2. Conservative breast surgery -
- Gold standard for the next 50 years (lumpectomy, quadrandectomy,
segmental mastectomy) & irradiation
BREAST CANCER
Historical Perspectives Breast Cancer
Patey and Madden (1965) - Modified Radical Treatment
Mastectomy Stage I & Stage II
Components: Options:
1. Total mastectomy 1. Breast Conservative Surgery-
2. Axillary LN dissection (Lumpectomy/quadrandectomy /segmental
mastectomy) w/ separate axillary node dissection
-Most common operation for breast cancer in the and irradiation
Philippines 2. Modified radical mastectomy (Total mastectomy
w/ axillary dissection)
BREAST CANCER
Historical Perspectives Breast Cancer
Veronesi & Fischer (1980) - Conservative breast Treatment
surgery Stage 3 A
Components: Options:
1. Quadrantectomy/Lumpectomy 1. Modified Radical Mastectomy
2. Axillary sampling 2. Induction chemotherapy + MRM +
3. Radiation of the breast Radiation

Induction Chemotherapy
CONSERVATIVE BREAST SURGERY - Chemotherapy given before the initiation of local
therapy
SR - Also called neoadjuvant or preoperative
Study Stage Treatment No. 5 yr 10 chemotherapy
yr - Advantages:
Milan 1 QUART 352 92 79 1. Reduction of the initial tumor burden before
1 Rad. Mast. 349 90 78 surgery
WHO 1 QUART 88 95 2. Ability to treat the potential systemic disease w/o
1 MRM 91 91 delay
NSABP I,II Lumpty/Rtx 625 83 3. Ability to asses the response of the tumor to the
I,II MRM 586 79 treatment being rendered
NCI I,II Lumpty/Rtx 112 88
I,II MRM 103 84
Induction Chemotherapy

Contraindications to Breast Conservative Surgery Response Rate to Induction


1. Tumor greater than 5 cm Chemotherapy/Hormonal Therapy
2. Poorly defined tumor 1. Complete response- disappearance of the lesion.
3. Large tumor in a small breast a. Complete clinical response- mass
4. Pregnancy’ disappearance by PE/UTZ/Mammography
5. Previous irradiation in the same site b. Complete pathologic response- absence
6. Multiple tumors within the breast confirmed to be of cancer microscopically
malignant - Reliable surrogate of improved OS
7. Diffuse microcalcifications on mammography 2. Partial response- >50% disappearance of the
8. Tumor involves skin or chest wall lesion
UST FMS MEDICAL BOARD REVIEW 2019 4 | SURGERY
BREAST
MICHAEL ALAY-AY, M.D.

3. No response- < 50% disappearance of the lesion B. Hormonal therapy


4. Progression of the disease- growth or worsening C. Radiation
of the condition D. Biologic therapy

Chemotherapy
Multimodal Therapy Improves 5 yr. SR up to 30 %
- Tumor size > 1 cm., and/or (+)ALN metastasis
AUTHOR YR INST TREATMENT NO. -BONNADONA- CMF
COMPLETE + 5-YEAR - 12 cycle in 6 months
OF PARTIAL SWOG - FAC/AC
OVERALL - more effective than CMF but w/ more
PX’S RESPONSE toxicity
SURVIVAL - Complete alopecia, highly emetogenic
RATES (%) - most common regimen used in the Phil
(%)
Valagussa 1983 Instituto AVx3-4->S->AV Hormonal Therapy
65 79.6 49.4 - All invasive breast CA with ER/PR (+) tumors
et al Nazionale AVx3-4->XRT->AV 96 -Hormonal Receptors
35.7 - Specific proteins in the cytosol of breast
Tumori orAVx3-4->XRT 46 CA
Schwartz 1994 Thomas CMFx3->S->CMF - Estrogen and progesterone receptors
189 85 69 - ER and PR activity is a measure of
et al Jefferson or hormonal responsiveness of the index tumor
Univ. Hosp. CMFx3->A (if no resp) or metastatic foci of disease
>S>CMF - Degree of positivity is proportional to the
Buzdar 1995 M.D. Anderson differentiation and histologic subtype of the
Cancer Center FACx3->S->CTXx2yrs 174 lesion
88 54 (IIIA)
FACx3->XRT->CTXx2yrs Hormonal Manipulation
24 (IIIB) 1. Ablation - Oophorectomy
FACx3->S+XRT->CTXx2yrs - Adrenalectomy
or VACPx3->S->VACP 200 - Hypophysectomy
84 2. Additive - Parodoxical effects of high estrogen
Malilay 1994 USTH CMFx2->S->CMFx6->XRT dosage
16 75 3. Anti-estrogen –
Yap - Tamoxifen
Malilay 2002 USTH CMF3x-> or CAF 3x 30 - Diethylstilbestrol
50-80 - Aminogluthetimide
Narciso - Aromatase inhibitors- Letrozole,
______ Anastrazole, Exemestane

Breast Cancer
Treatment Hormonal Therapy
Stage 3 B - (LABC including Inflammatory CA) Tamoxifen
Induction Chemotherapy - Most common form of hormonal therapy
- Good response MRM Radiation - Absence of toxicity and profound side effects
- Poor response Radiation MRM - Adverse effect- Less than 5%
1. Endometrial CA
Induction hormonal treatment 2. Thromboembolic events
- Given at 20 mg. daily
For elderly, frail patients and for whom - Duration: 5 years
chemotherapy and or surgery cannot be given
Aromatase inhibitors better than Tamoxifen
Response rate 55%-83% Hormonal Therapy
Estrogen (+) positive tumors Markers:
1. Estrogen receptor
Breast Cancer 2. Progesterone receptor Response rate
Treatment
Stage IV ER + > 80%
Radiation &/or PR +
palliative (hygienic mastectomy ) +
chemotherapy &/or hormonal therapy ER + > 27%
PR -
Breast Cancer
Treatment ER - > 45%
Adjuvant Therapy PR +
A. Chemotherapy
UST FMS MEDICAL BOARD REVIEW 2019 5 | SURGERY
BREAST
MICHAEL ALAY-AY, M.D.

ER - > 10% 1. Thymidine labelling index


PR - 2. Ploidy pattern / S phase
Hormonal Therapy 7. Menopausal status
Aromatase Inhibitors 8.General health
Blocks conversion of steroids to estrogen
Treatment for postmenopausal women with Breast Cancer Survival Rate
Estrogen rich tumors should include AI’s with or
without Tamoxifen Breast Cancer
Letrozole, Anastrazole & Exemestane Follow-up
Given only to postmenopausal women - 80 % recurrence w/in the first 5 years
RCT’s better DFS compared to Tamoxifen - Evaluation should be individualized
Expensive (P360 vs P30) - 65-85% of recurrences detected by history & PE
Side effects: Bone fractures, muscle and joint pains, - Improved survival among breast cancer patients
cardiovascular who later become pregnant

Biologic Therapy Breast Cancer


Facts about the HER2 gene Follow-up
Recurrences:
- HER2 is a gene that helps control how 1. Local- 10-30 %
cells grow, divide, and repair 2. Distant- 60-70 %
themselves. 3. Local & distant- 10-30 %
- The test for the HER2 gene is called
FISH (Fluorescence in Situ Locally Recurrent Breast Cancer
Hybridization). A. Breast conservative surgery
- The gene directs the cancer cells to - rarely associated with distant
make extra HER2 protein. metastases
- Cancers with too much HER2 protein are - recurrence remains curable in the
called HER2-positive. majority of cases
- These cancers grow fast but respond - Treatment: Salvage MRM
well to HER2 antibody therapy - 60 - 70% 5 yr. disease free SR
(Herceptin).
Locally Recurrent Breast CA
B. Modified Radical Mastectomy
- Frequently associated w/ distant
metastases
- Treatment: Surgical excision +
Biologic Therapy radiotherapy
Trastuzumab (Herceptin) - Median survival - 2-3 yrs.
Tumor size > 1 cm., and/or (+)ALN metastasis -
A recombinant humanized monoclonal antibody Metastatic Breast Cancer
directed against the extracellular domain of the - Generally cannot be cured
human epidermal growth factor receptor 2 (HER2 - Median survival - 2 yrs.
neu or ErbB-2) protein. - Selection of initial treatment depends whether the
Human epidermal growth factor receptor 2 (HER2 patients tumor is hormonally sensitive or not
neu) is overexpressed in approximately 25% of
invasive breast cancers.
In combination with chemotherapy reduced risk of Metastatic Breast Cancer
recurrence by 50% and risk of death by 33% Metastatic breast cancer
Cardiac toxicity ER (+) ER (-)
Unknown
RADIOTHERAPY
Patients at risks for locoregional recurrence: Premenopausal Postmenopausal
Tumor size greater than 5 cm.
Patients with multiple (4 or more) involved axillary Tamoxifen
nodes
Patients with extracapsular extension of nodal tumor Nonresponders Responders
Patients with locally advanced breast CA 1. Oophorectomy
2. DES, Progesterone

Factors that Affect Prognosis and Selection of CHEMOTHERAPY


Therapy
1. Age of the patient Metastatic breast cancer
2. Stage of the disease
3. Pathologic characteristics of the primary tumor Results of Chemotherapy for metastases
4. ER & PR receptor level - Favorable response in 2/3 of patients
5. HER 2 neu - Complete response in 15 %
6. Measures of proliferative capacity
UST FMS MEDICAL BOARD REVIEW 2019 6 | SURGERY
BREAST
MICHAEL ALAY-AY, M.D.

Breast Cancer and Pregnancy


- Diagnosis typically more difficult
1. Low level of suspicion- young age
2. Nodular changes during pregnancy
3. Inc. density makes mammography less accurate
- Diagnosis frequently delayed
- Poor prognosis
- When matched for tumor stage, pregnant women
with breast cancer have the same prognosis as the
non- pregnant patient

Breast Cancer and Pregnancy


Treatment
- Treatment decisions are influenced by their
timing w/ respect to the specific trimester of
pregnancy
- Numerous studies have shown that terminations
of pregnancy in hopes of minimizing hormonal
stimulation of the tumor has shown no benefit to
maternal survival
- Goal should be curative treatment of the breast
cancer w/o injury to the fetus

Breast Cancer & Pregnancy


Treatment
- Mod. Radical mastectomy can be undertaken at
any point during pregnancy
- Breast conservative surgery complicated by the
fact that radiotherapy is contraindicated during
pregnancy. Option in the third trimester.
- Chemotherapy
1. No risk during the 2nd and 3rd trimester
2. During the Ist trimester asso. w/ an
increased
evidence of spontaneous abortion &
congenital malformation

Carcinoma of the Male Breast


- Less than 1% of all breast cancer
- Peak evidence 60 - 69 years of age
- Tumor commonly ER (+)
- Stage for stage have the same survival rate as
women
- Overall prognosis is poor because of the advanced
stage of the disease at diagnosis
- Treatment:
l. Modified Radical Mastectomy
2. Radiotherapy for ulcerative & high grade
tumor

UST FMS MEDICAL BOARD REVIEW 2019 7 | SURGERY


BREAST
MICHAEL ALAY-AY, M.D

BREAST REVIEW TEST

Choose the best answer:


______9. Characterized by chronic eczematoid
______1. True of breast cancer in the Philippines: eruption of the nipple:
A. The 2nd leading cause of cancer in women A. Paget’s disease
B. The leading cause of cancer in both sexes B. Tubular carcinoma
C. The leading case of cancer death in both C. Medullary carcinoma
sexes D. Mucinous carcinoma
D. 1 in 10 women will develop breast cancer
_____10. Therapy specifically directed at disrupting
_____ 2. True of breast cancer facts and figures in molecular genetic alteration promoting
the Philippines in the year 2005 cancer growth:
A. Median survival among females is 60 A. Hormonal therapy
months B. Immunotherapy
B. Survival at the 5th year is 70% C. Chemotherapy
C. 10-year survival rate is 50% D. Targeted therapy
D. Incidence starts rising steeply at age 50

______3. The most common operation for breast


cancer in the Philippines is:
A. Total mastectomy
B. Breast conservative surgery
C. Modified radical mastectomy
D. Halsted Mastectomy

______ 4. Most cause of death in breast cancer is:


A. Local recurrence
B. Disseminated metastasis
C. Chemotherapy induced toxicity
D. Hormonal therapy induced toxicity

______5. True of mammography:


A. Highly specific
B. 50% of nonpalpable lesions detected are
found to be malignant at biopsy
C. Sine qua non for breast cancer:
speculated density with ill-defined margins
D. Features that are diagnostic of cancer
include coarse calcifications

______ 6. The most important issue for both patient


and physician faced with any breast problem is:
A. Definition of the precise histology
B. Provision of symptomatic relief
C. Whether the patient wants the breast
preserve or not
D. Establish whether the patient does or
does not have cancer

______7. Known risks factors for breast cancers


EXCEPT:
A. Family history
B. High dietary fat intake, Obesity
C. Bilateral oophorectomy at age 25
D. Prolonged hormone replacement therapy

______8. Most important prognostic correlate for


recurrent disease & survival
A. Age
B. Axillary nodal status
C. Histologic grade
D. ER & PR status

UST FMS MEDICAL BOARD REVIEW 2019 | SURGERY


HERNIA
JAN ANDREW D. BUENO, MD

HERNIA

 abnormal protrusion of an organ or tissue


through a defect in its surrounding walls (fascia
or muscle)
 commonly involve the abdominal wall,
particularly the inguinal region
 occur at sites where the aponeurosis and fascia
are not covered by striated muscle

ETIOLOGY

 congenital: failure of fascial opening to close  Spermatic cord contents


(umbilical) or failure of obliteration (patent o Cremaster muscle fibers (from internal oblique
processus vaginalis) muscle)
 acquired: weakness/deterioration with age or o Testicular artery & accompanying veins
loss of tissue (injury, infection, wound healing) o Genital branch of genitofemoral nerve
o Vas deferens
RISK FACTORS o Cremasteric vessels
o Lymphatics
 Increase in abdominal pressure: heavy lifting, o Autonomic nerves
chronic cough/COPD/asthma, constipation,
straining on urination, ascites  Nerves in the inguinal region
 Obesity o Iliohypogastric (T12-L1): sensation to the
 Older age skin of lateral gluteal and above the pubis.
 Connective tissue disease o Ilioinguinal (L1): sensation to the skin of the
 Smoking upper and medial thigh, base of penis &
 Abdominal wall injury/previous surgery upper scrotum / mons pubis & labia majora.
Traverses the inguinal canal.
Primary Abdominal Wall Hernias o Genitofemoral nerve (L1-2):
 Genital branch: supplies cremaster and
Groin Pelvic scrotal skin / mons pubis & labia
Inguinal Obturator majora.
Indirect Sciatic  Femoral branch: skin of upper anterior
Direct Perineal thigh. Courses along the femoral
Combined (Pantaloon) sheath.
Femoral Posterior o Lateral femoral cutaneous (L2-3): supplies
Lumbar the lateral thigh
Anterior Superior triangle
Umbilical (Grynfeltt’s)
Epigastric Inferior triangle
Spigelian (Linea semilunaris) (Petit’s)

Inguinal Anatomy

 Inguinal canal
o contains the spermatic cord/round ligament
o begins at the internal inguinal ring (hiatus in
the transversalis fascia)
o ends at the external inguinal ring (defect in
the external oblique aponeurosis)
o Boundaries:
 Anterior: external oblique aponeurosis
 Posterior: transversalis fascia and
transversus abdominis muscle
 Superior (roof): internal oblique muscle
 Inferior (floor): inguinal/Poupart’s  Hesselbach triangle
ligament o refers to the margin of the floor of the
inguinal canal
o Boundaries:
 Superolateral: inferior epigastric vessels
 Medial: rectus sheath (linea semilunaris)
 Inferior: inguinal ligament

UST FMS MEDICAL BOARD REVIEW 2019 1 | SURGERY


HERNIA
JAN ANDREW D. BUENO, MD

Femoral Hernia
Inguinal Hernia
 located inferior to the inguinal ligament and
INDIRECT DIRECT protrude through the femoral ring
Lateral to inferior Medial to inferior  medial to the femoral sheath, lateral to lacunar
epigastric vessels epigastric vessels ligament
Pass through inguinal Bulge through  acquired muscle weakness of inguinal floor
canal posterior wall of  more common in females than males
*Complete/incomplete inguinal canal  high risk of incarceration
Congenital: patent Acquired: weakness of
processus vaginalis; muscular layer
canal of Nuck (females)
Common in children Common in old age
and young adults
Can descend into the Does not descend into
scrotum the scrotum*

 Pantaloon hernia: combined indirect and


indirect
 Incomplete: hernia confined to the inguinal
canal
 Complete: hernia descends to the scrotum
 Reducible: sac and/or contents can be
returned to abdominal cavity
 Irreducible/incarcerated: contents cannot be Umbilical Hernia
returned to the abdominal cavity
 10-30% in general population
 Strangulated: compromised blood supply
o Congenital: umbilical defect failed to close
 Richter: partial circumference of bowel wall
o Acquired: pregnancy, ascites
is incarcerated
 may close 12-18 months from birth
 Sliding: wall of viscus forms part of the
 repair is recommended when child is 3 years old
hernial sac (ie. R - Cecum, L- Sigmoid colon)
or older
 Littre: hernia contains Meckel’s diverticulum
o <2cm defect – primary closure
 Amyand: hernia contains appendix
(vest over pants technique)
o >2cm defect – mesh repair

Epigastric Hernia

 defects in abdominal midline between umbilicus


and xiphoid
 may be multiple (20%)

Spigelian Hernia

 bulge in semilunar line (lateral to rectus muscle)


 difficult to identify by physical exam 
ultrasound
 up to 20% are incarcerated
 repair once diagnosis is made

UST FMS MEDICAL BOARD REVIEW 2019 2 | SURGERY


HERNIA
JAN ANDREW D. BUENO, MD

Obturator Hernia  Techniques:


o Open repair: tissue vs mesh repair
 rare type (<1%) o Laparoscopic repair
 through the obturator ring  Use of hernia belt/truss
 right > left; female > male
 Howship-Romberg sign: pain along medial aspect
of thigh and hip. Relieved by flexion of hip;
exacerbated by abduction, extension, and medial
rotation

Lumbar Hernias

 bulge along lumber region


o Superior: 12th rib
o Medial: erector spinae muscle
o Lateral: external oblique muscle
o Inferior: iliac crest
 Grynfeltt – superior lumbar triangle
 Petit – inferior lumbar triangle

Incisional Hernia

 fascial defect at sites of previous incisions


 20% of patients following abdominal surgery
 contributory factors:
o obesity
o wound healing defects
o multiple surgical procedures
o prior incisional hernias
o technical error during repair

Diastasis Recti

 acquired widening of fascial ridge between rectus


muscles (not a hernia)
 no risk for incarceration or strangulation
 contributory factors: obesity and pregnancy
 treatment: weight loss and plication

Treatment of Hernias

 most should be treated surgically


 watchful waiting for:
o unfit for surgery: elderly, uncontrolled co-
morbid illness
o control of risk factors (COPD, ascites)
o child with umbilical hernia

UST FMS MEDICAL BOARD REVIEW 2019 3 | SURGERY


HERNIA
JAN ANDREW D. BUENO, MD

UST FMS MEDICAL BOARD REVIEW 2019 4 | SURGERY


REVIEW TEST
JAN ANDREW BUENO, MD

HERNIA BOARD REVIEW QUESTIONS:


_____7. A 50-year-old female presents with a right
_____1. Physical examination of an inguinal hernia inguinal mass. She also complains of pain along the
includes palpation of the external inguinal ring, medial aspect of the right hip, which is relieved by
which is a hiatus formed by which structure? flexion, and exacerbated by abduction, extension,
A. Rectus sheath and medial rotation. What do you call this physical
B. External oblique aponeurosis examination maneuver (sign)?
C. Internal oblique A. Obturator
D. Transversalis fascia B. Psoas
C. Fothergill
_____2. A 25-year-old female presents with an D. Howship-Romberg
inguinal bulge. What is the most common type of
hernia to be considered? _____8. What is the counterpart of the processus
A. Indirect inguinal vaginalis in females?
B. Direct inguinal A. Processus vaginalis
C. Femoral B. Foramen of winslow
D. Obturator C. Canal of Nuck
D. Internal inguinal ring
_____3. Which is NOT part of the Hesselbach
triangle? _____9. During a hernia surgery, the sigmoid colon
A. Inguinal ligament was noted to form part of the wall of the hernial sac.
B. Inferior epigastric vessels What do you call this kind of hernia?
C. Anterior superior iliac spine A. Incarcerated
D. Linea semilunaris B. Richter
C. Littre
_____4. A newborn infant was seen to have a bulge D. Sliding
at the umbilical region. No other problems were
noted. An umbilical hernia is considered. What is the _____10. A 40-year-old male was brought to the
best management? emergency room due to a right inguinal mass. He
A. Observation recalled that the mass used to be reducible but
B. Emergency repair of umbilical hernia became “stuck” 3 hours prior. No other symptoms
C. Repair of umbilical hernia after obtaining were noted. Physical exam was unremarkable other
pre-operative clearance than the irreducible inguinal bulge on the right.
D. Repair of umbilical hernia prior to What should be done to the patient?
discharge A. Send the patient home and follow-up at
the clinic after 1 week
_____5. A 30-year-old male consulted due to an B. Admit the patient for emergency
inguinoscrotal mass on the right. On physical herniorrhaphy
examination, the mass could be completely reduced, C. Perform taxis of the hernia
the right external inguinal ring was enlarged, and D. Advise the patient to wear hernia
both testis were unremarkable. Which the most belt/truss
appropriate diagnosis?
A. Hernia, inguinal, indirect, complete,
reducible, right
B. Hernia, inguinal, indirect, incomplete,
reducible, right
C. Hernia, inguinal, direct, reducible, right
D. Hernia, femoral, reducible, right

_____6. Which nerve is contained in the spermatic


cord?
A. Iliohypogastric
B. Ilioinguinal
C. Femoral branch of the genitofemoral
D. Genital branch of the genitofemoral

UST FMS MEDICAL BOARD REVIEW 2019 | SURGERY


ORTHOPEDIC SURGERY
ROBERT CHAN, MD/JOSE FERNANDO SYQUIA, MD

I. INTRODUCTION 2. T-score of -2.5 and below is


a. Orthopedics is a medical and surgical osteoporosis
specialty that deals with the prevention, 3. T-score between -1 and -2.5 is
diagnosis and treatment of conditions osteopenia or low bone mass
and injuries involving the musculoskeletal 4. T-score of -1 and above is
system. normal
b. Functions of the musculoskeletal system: b. Osteomalacia is different from osteoporosis
i. Provide support for the body i. Adult counterpart of rickets
ii. Protect the vital organs ii. Failure of proper mineralization of the
iii. Facilitate easy movement of normal organic matrix (osteoid)
joints
c. Bone III. BONE AND JOINT INFECTIONS
i. A hard connective tissue consisting of a. Osteomyelitis
cells embedded in a matrix of i. Bone infection
mineralized ground substance and ii. Modes of transmission:
collagen fibers. 1. Blood-borne
ii. Components: 2. By contiguity
1. Cellular components: 3. By direct invasion
a. Osteoprogenitor cells – iii. Type of osteomyelitis
undifferentiated 1. Acute hematogenous osteomyelitis
b. Osteoblasts – forms bone a. Blood-borne, usually in children
c. Osteoclasts – resorbs bone 2. Subacute osteomyelitis
d. Osteocytes – regulates a. Due to partially treated acute
calcium osteomyelitis
2. Matrix: b. Radiographic findings may mimic
a. Organic matrix tumors
i. Mainly type 1 collagen c. Example is Brodie’s abscess
b. Inorganic matrix 3. Chronic osteomyelitis
i. Mainlycalcium a. Presence of sequestrum (dead bone)
hydroxyapatite and involucrum (new bone)
ii. 99% of the calcium in the iv. Staphylococcus aureus – most common
body is stored in the pathogen
skeleton 1. H. influenzae may be more common for
iii. Types of bone formation: children less than or equal to four years
1. Intramembranous bone old (not including newborn)
formation v. Treatment involves antibiotics and, if
a. No cartilage model needed, surgery.
b. In facial and cranial bones 1. Antibiotic treatment usually lasts for 6
2. Enchondral bone formation: weeks
a. Cartilage model is formed b. Pyogenic arthritis
which is replaced with i. Joint infection
bone ii. Modes of transmission
1. Hematogenous
II. OSTEOPOROSIS 2. Local spread
a. Osteoporosis 3. Puncture wound involving the
i. Bone is normal in quality (unlike joint
osteomalacia) BUT it is diminished in 4. Open wound
quantity iii. Staphylococcus aureus – most common
ii. Types: pathogen
1. Postmenopausal or Type 1 1. Staphylococcus epidermidis –
a. Mainly in females after menopause common in joint replacement
(female to male ratio is 6:2) surgery
b. Mainly trabecular bone loss iv. Treatment involves surgical drainage
c. Accelerated bone loss and antibiotics
2. Involutional or Type 2 c. Tuberculous arthritis
a. Female to male ratio is 2:1; older i. Due to Mycobacterium tuberculosis
persons ii. Joint is involved by hematogenous
b. Involves both trabecular and spread from the lungs or intestines
cortical bone iii. Spine and lower extremities are usually
c. Bone loss is not accelerated involved.
iii. Based on the WHO classification: iv. Treatment may involve surgery.
1. T-score of -2.5 and below with a v. Anti-TB medications are usually given
history of a fragility fracture is severe for 6-12 months.
osteoporosis

UST FMS MEDICAL BOARD REVIEW 2019 1 | SURGERY


ORTHOPEDIC SURGERY
ROBERT CHAN, MD/JOSE FERNANDO SYQUIA, MD

d. Tuberculosis of the spine (Pott’s disease) 3. Classification of open fractures


i. Spine is the most common site of (Gustilo and Anderson)
skeletal TB
ii. A blood-borne infection that involves a. Type I
the vertebral body resulting in i. Low energy injury with a wound less than
destruction and caseation necrosis. 1cm
iii. Presence of cold abscess and vertebral b. Type II
collapse. i. Low energy injury with a wound more
iv. Features include back pain, kyphosis or than 1 cm but less than 10 cm
gibbus, muscle spasm and a possible c. Type III
neurologic deficit. i. High energy injury with extensive
wounds more than 10 cm
IV. TUMORS ii. Complete or incomplete
a. Primary bone lesions: 1. Complete fractures – cortex is completely
i. Sarcoma disrupted
1. Malignant neoplasm of 2. Incomplete fractures – only one side of the
mesenchymal origin bone is broken
ii. Benign bone tumors a. Greenstick fracture
iii. Reactive conditions b. Torus fracture
1. May mimic tumors but are not c. Compression fracture
true neoplasms, such as iii. Pattern of failure
certain bone infections 1. Transverse – due to tensile forces
b. Biopsy – performed to determine the 2. Oblique – due to compressive forces
nature of the tumor 3. 3-part with a butterfly fragment – due to
i. Needle biopsy – a fine needle is used bending forces
to obtain a specimen 4. Spiral – due to torsional forces
ii. Incisional biopsy – a small part of the 5. Comminuted fractures – 3 or more fragments
tumor is sampled
iii. Excisional biopsy – the tumor is d. Special types of fractures:
completely excised i. Stress fracture – normal bone is
c. Notes: continually stressed submaximally in a
i. Non-ossifying fibroma – most common cyclical fashion until failure occurs
benign tumor of bone ii. Pathologic fracture – weakened bone
ii. Osteogenic sarcoma (osteosarcoma) – fractures under normal stress
second most common primary bone e. Treatment:
malignancy; most common malignant i. Methods:
tumor of osteogenic origin 1. Reduce the fracture – align the
iii. Ewing’s sarcoma – malignant tumor, fragments
mainly diaphyseal, that may present a. Closed reduction – fracture
like low-grade osteomyelitis hematoma is not violated
iv. Multiple myeloma – most common b. Open reduction – fracture is
primary malignancy of bone directly visualized
v. Metastatic tumor – the most common 2. Maintain the reduction
malignant tumor of bone a. Casts, splints, traction
vi. Rhabdomyosarcoma – most common b. Internal fixation
sarcoma in young patients i. Rigid fixation with plates and
d. Treatment: screws
i. Surgery such as limb salvage or ii. Non-rigid fixation with nails
amputation and pins
ii. Chemotherapy c. External fixation
iii. Radiation therapy 3. Preserve or restore function
f. Fractures in children
V. FRACTURES i. Different from adults:
a. Definition: 1. Incomplete fractures are more
i. Break in the continuity of the bone common
ii. Soft tissue injury with a bony 2. Less displacement due to a
component thicker periosteum
b. Abnormal mobility (preternatural mobility) 3. Healing is faster
– most definitive sign of the presence of a 4. Lesser risk for contractures
fracture ii. Physeal fractures occur through the
c. Description: growth plate
i. Open or closed g. Some common eponyms for fractures:
1. Open fractures communicate i. Colles’ fracture - fracture of the
with the environment distal radius with dorsal
2. Surgical debridement should be displacement of the distal fragment
done within 8 hours
UST FMS MEDICAL BOARD REVIEW 2019 2 | SURGERY
ORTHOPEDIC SURGERY
ROBERT CHAN, MD/JOSE FERNANDO SYQUIA, MD

ii. Smith’s fracture – fracture of the 3. Valsalva maneuver worsens the


distal radius with volar pain
displacement of the distal fragment 4. Positive straight leg raising
iii. Monteggia’s fracture – fracture of (SLR) test
the proximal ulna with dislocation iii. Imaging:
of the radial head 1. MRI is the preferred imaging
iv. Galeazzi fracture – fracture of the modality
radius with dislocation of the distal d. Spondylolysis
radioulnar joint i. Defect in the pars interarticularis.
v. Jefferson fracture – burst fracture ii. Most common cause of low back pain
of the atlas in children and adolescents
vi. Hangman’s fracture – fracture of e. Spondylolisthesis
both pars interarticularis of the i. Vertebra or part of a vertebra slips
axis; also called traumatic anteriorly over the vertebra below it
spondylolisthesis of the axis ii. Most common is L5 slipping over S1
vii. Floating knee – fracture of the
femur and ipsilateral tibia VII. THE HIP
viii. March fracture – stress fracture of a. Avascular necrosis (osteonecrosis) of the
the 2nd metatarsal bone femoral head
ix. Lisfranc fracture dislocation – i. Due to impairment of the blood supply,
fracture with dislocation involving there is death of a part of the femoral
the tarsometatarsal joint head that eventually leads to its
x. Jones’ fracture – fracture of the collapse
base of the 5th metatarsal bone b. Legg-Calve-Perthes Disease (LCPD)
xi. Pseudo-Jones’ fracture – avulsion i. Osteonecrosis of the femoral epiphysis
fracture of the tuberosity of the 5th in children, usually boys, aged 3 – 12
metatarsal bone years
ii. Treatment is aimed at containing the
VI. THE SPINE head inside the acetabulum
a. Scoliosis (containment of the head) using braces
i. Defined as a lateral curvature of the or with surgery.
spine c. Coxa vara
ii. Types: i. Neck-shaft angle is less than 120
1. Postural – curve disappears degrees (normal is about 125 degrees
with a change in posture in an adult)
2. Structural – there is bony d. Coxa valga
abnormality, the deformity is i. Neck-shaft angle is more than 135
fixed and there is associated degrees
vertebral rotation ii. Normal in infants (as high as 140
iii. Structural scoliosis degrees), but the angle should
1. Adolescent idiopathic scoliosis decrease with age and with weight
a. Most common type bearing
b. Common in females e. Slipped capital femoral epiphysis (SCFE)
c. Risk of curve progression i. The epiphysis of the femoral head slips
is greater with: posteriorly and inferiorly at the physis
i. Greater curve (>20 degrees) or growth plate
ii. Younger age (< 12 years) ii. Shearing stress is greater than the
iii. Lower Risser stage (stage 0 or 1) resistance of the physis; slip usually
2. Congenital scoliosis occurs at the hypertrophic zone of the
a. Most common congenital spinal disorder physis
b. Acute lumbar strain f. Developmental dysplasia of the hip (DDH)
i. Non-radiating, low back pain due to i. Previously called congenital dysplasia
mechanical of the hip or congenital hip dislocation
ii. Normal neurologic examination ii. The acetabulum is shallow and the
c. Herniated disk disease of the lumbar spine femoral head is displaced laterally
i. L5-S1 disk is most commonly involved, iii. Special tests:
followed by L4-L5 1. Ortolani test (elevate and
1. Nerve root exiting the next abduct the femur to relocate a
lower level is often involved. dislocated hip)
For L5-S1, it would be S1. For 2. Barlow test (adduct and
L4-L5, it would be L5. depress the femur to dislocate
ii. Signs and Symptoms: a dislocatable hip)
1. Radicular leg pain along the 3. Galazzi sign (short knee on the
involved dermatome affected side)
2. There may be neurologic
symptoms
UST FMS MEDICAL BOARD REVIEW 2019 3 | SURGERY
ORTHOPEDIC SURGERY
ROBERT CHAN, MD/JOSE FERNANDO SYQUIA, MD

iv. On x-rays, the femoral epiphysis d. Anterior cruciate ligament injury


appears subluxated or dislocated and i. Due to:
Shenton’s line is disrupted 1. Extreme varus or valgus stress
2. Hyperextension
3. Rotation
VIII. ADULT RECONSTRUCTIVE SURGERY ii. PE includes positive Lachman (most
a. Arthrodesis sensitive, performed with the knee in
i. The joint is eliminated by fusing it. 20 deg flexion), anterior drawer test
ii. The pain is relieved at the expense of (knee in 90 deg flexion) and pivot shift
motion. tests
b. Resection Arthroplasty iii. Unhappy triad of O’Donaghue
i. The ends of the bones that form a joint 1. ACl tear
are removed. The resulting space is 2. MCL tear
eventually filled with scar tissue. 3. Medial meniscus tear
ii. The pain is relieved at the expense of iv. Treatment:
stability. 1. Conservative
c. Osteotomy a. The hamstrings are the
i. A procedure in which bone is surgically muscular counterpart of
cut and repositioned. The healthy part the ACL
of the joint is allowed to bear weight 2. Arthroscopic ACL
by shifting it into the optimal position. reconstruction
d. Arthroscopy e. Posterior cruciate ligament injury
i. The joint is viewed with a scope called i. Due to:
an arthroscope 1. Extreme varus or valgus
ii. Therapeutically, intraarticular problems stresses
may be treated with procedures such 2. Force displacing the tibia
as synovectomy or meniscectomy posteriorly
e. Arthroplasty is an operation designed to ii. Tests include: reverse Lachman,
restore motion to a joint and function to posterior drawer, and sag sign
the muscles, ligaments, and other soft f. Osgood-Schlatter Disease
tissues that control the joint. i. Partial separation of the epiphysis of
i. Absolute contraindications include: the tibial tuberosity
1. Recent or current joint sepsis ii. Common in active boys, 10-14 years
2. Neuropathic arthropathy old; usually due to constant, repetitive
ii. Materials used include metals forceful contraction of the quadriceps
(titanium, chromium alloy, stainless such as in jumping
steel), ceramics, and ultrahigh iii. There is pain and enlargement of the
molecular weight polyethylene tibial tuberosity
iii. Complications – several potential g. Recurrent Patellar Instability
complications, but: i. Either dislocation or subluxation
1. Implant loosening – most ii. Apprehension test is (+) in cases of
common long-term dislocation
complication h. Rupture of the Extensor Mechanism
2. Infection – most devastating i. Usually through the patella (Patellar
and dreaded complication fracture)
ii. May also be through the:
IX. THE KNEE 1. Rectus femoris
a. Meniscal tear 2. Quadriceps tendon
i. Medial meniscus is more commonly 3. Patellar tendon
injured; longitudinal split is the most i. Blount’s disease
common type of tear. i. Tibia vara due to growth retardation at
ii. Symptoms include pain, swelling, the medial proximal tibial physis; tibia
locking, clicking. vara extends beyond 3 years of age
iii. McMurray and Appley tests may be ii. Results in severe varus of the knee
positive j. Genu varus
iv. MRI is very sensitive in detecting tears. i. Bowlegs
b. Medial collateral ligament injury ii. May be physiologic in children up to 24
i. Due to valgus stress months
ii. Isolated injuries are treated k. Genu valgus
conservatively i. Knock-knees
c. Lateral collateral ligament injury ii. May be physiologic in children from 4-7
i. Due to varus stress years of age
ii. Common peroneal nerve may be l. Bursa around the knee
injured i. Prepatellar bursa (Housemaid’s knee -
when swollen)

UST FMS MEDICAL BOARD REVIEW 2019 4 | SURGERY


ORTHOPEDIC SURGERY
ROBERT CHAN, MD/JOSE FERNANDO SYQUIA, MD

ii. Deep infrapatellar bursa XI. THE NECK AND SHOULDER


iii. Superficial pretibial bursa a. Cervical spondylosis
iv. Popliteal bursa (Baker’s cyst) i. Cervical spondylosis is a progressive
v. Pes Anserine bursa degeneration of the intervertebral discs
of the neck
X. THE FOOT AND ANKLE ii. Signs and symptoms
a. Flatfoot 1. Headache that is mainly
i. Loss of the medial longitudinal arch of occipital with radiation into the
the foot frontal area
ii. Etiology: 2. Painful, stiff neck
1. Flexible flatfoot 3. Cervical radiculopathy
a. In children up to 5 years a. Pain that radiates in a
old dermatomal fashion to the
2. Peroneal tendon rupture upper extremity
a. Also called “Adult onset b. Weakness, numbness,
flatfoot” paresthesias
b. Clawfoot 4. Cervical stenosis with
i. Pes cavus with claw toes myelopathy
c. Hallux Valgus a. Paresthesias, dyskinesias,
i. Lateral angulation of the big toe at its or weakness of the hand
metatarsophalangeal joint b. Difficulty walking; broad
ii. First metatarsal deviates medially while based gait
big toe deviates laterally c. Loss of balance
iii. Commonly called “bunion”, although b. Ossification of the posterior longitudinal
this is incorrect. ligament (OPLL)
d. Bunion i. Relatively common cause of
i. Bony prominence over the medial spinal canal stenosis and
aspect of the 1st MTP joint myelopathy among Asians
ii. This may be associated with an ii. There is clumsiness, spastic
inflamed bursa gait, or spastic quadriparesis
e. Hallux Varus (most common neurologic
i. Medial angulation of the big toe at its presentation)
metatarsophalangeal joint iii. Levels most often involved are
f. Hallux Rigidus C4, C5, C6
i. Degenerative joint disease of the first iv. Decompression is the main form
metatarsophalangeal joint of surgical intervention
ii. Called hallux rigidus because there is c. Herniated nucleus pulposus
limited range of motion (especially i. There is a projection of the central disc
dorsiflexion) at the MTP joint, in material through the annulus fibrosus that
addition to pain may impinge on the spinal cord or on a
g. Lesser Toe Deformity nerve root.
i. Hammer toe – dorsiflexion of the MTP ii. Signs and Symptoms:
joint and plantar flexion of the PIP joint 1. Neck pain with radiation to the upper
ii. Clawtoe – hyperextension of the MTP extremity; however, it is possible that
joint and flexion of the IP joints neck pain may not be a feature
iii. Mallet toe – flexion contracture of the 2. Pain is worse with the Valsalva
DIP joint maneuver
h. Tendo-Achilles rupture 3. Nerve root often involved is the one
i. Thompson test is positive – squeezing exiting at the involved level (e.g., for
the calf muscle does not result in C5/C6, it would be C6)
plantarflexion of the foot d. Supraspinatus tendinitis
ii. Treatment is either conservative or i. Inflammatory and degenerative cuff
surgical changes due to impingement
i. Ankle sprain ii. There is a painful arc syndrome (60-120
i. Usually involves: degrees) and the impingement tests
1. Anterior talofibular ligament – (Neer and Hawkin) are positive
when the ankle is in e. Rotator cuff tears
plantarlexion; commonly i. Usually occurs near the insertion of the
injured cuff muscles
2. Calcaneofibular ligament – ii. Complete tears lead to difficulty in
when the ankle is plantigrade initiating shoulder abduction.
ii. Usually mechanism is inversion + f. Adhesive capsulitis
plantarflexion i. “Frozen shoulder”
ii. A chronic condition with stiffness for 3-12
months.

UST FMS MEDICAL BOARD REVIEW 2019 5 | SURGERY


ORTHOPEDIC SURGERY
ROBERT CHAN, MD/JOSE FERNANDO SYQUIA, MD

iii. Often women, 50-70 years old; risk XIII. THE WRIST
factor is diabetes a. Distal radius fractures
g. Bicipital tenosynovitis i. Colles’ fracture
i. Inflammation of the long head of the 1. An extraarticular fracture of the
biceps tendon or sheath distal radius with dorsal
ii. There is pain over the anterior and displacement or angulation
medial region of the shoulder with 2. Common in the elderly
tenderness over the intertubercular 3. With “dinner fork” or “silver
sulcus. spoon” deformity
iii. Speed’s and Yergason’s tests are positive ii. Smith’s fracture
h. Rupture of the biceps tendon 1. A reversed Colles’ fracture
i. “Popeye” deformity – due to a proximal with the distal fragment
rupture of the long head of the biceps volarly displaced or
ii. Weakness of flexion and supination angulated
(especially supination) iii. Barton’s fracture
i. Shoulder instability 1. Intraarticular fracture-
i. Anterior glenohumeral dislocation – dislocation of the distal radius
most common 2. Either volar or dorsal
ii. May be associated with: displacement
1. Bankart lesion – avulsion of the iv. Chauffeur’s fracture
anteroinferior labral complex from 1. Fracture of the radial styloid
the glenoid b. Scaphoid fracture
2. Hill-Sach’s lesion – fracture of a i. Most common carpal fracture in
part of the humeral head athletes
posteriorly ii. Risk of delayed or nonunion
iii. Axillary nerve – nerve commonly injured c. De Quervain’s tenosynovitis
i. Inflammation of the synovium of the
XII. THE ELBOW sheaths of these two tendons as they
a. Tennis elbow pass in a fibro-osseous canal in the first
i. Extensor tendinopathy dorsal wrist compartment (there are 6
ii. Also called lateral epicondylitis dorsal wrist compartments):
iii. Involves the extensor carpii radialis 1. Abductor pollicis longus
brevis tendon proximally 2. Extensor pollicis brevis
b. Golfer’s elbow ii. Finkelstein’s test (thumb in palm and
i. Flexor/pronator tendinopathy ulnarly deviate the wrist) is positive
ii. Also called medial epicondylitis d. Carpal tunnel syndrome
iii. Involves the tendinous origin of the i. Median nerve is compressed as it
forearm flexor/pronator muscles, passes underneath the transverse
especially the pronator teres carpal ligament
c. Elbow dislocation ii. Clinical tests include: Tinel’s test,
i. Usual direction is posterior Phalen’s wrist flexion test, carpal tunnel
ii. May be associated with a fracture of compression test (direct compression
the radial head and the coronoid of the nerve by the thumbs of the
process of the ulna (terrible triad) examiner)
d. Supracondylar fractures of the elbow in iii. Electrodiagnostic studies – considered
children to be the gold standard when testing
i. The most common fracture about the for this condition
elbow in children e. Ulnar nerve compression
ii. Anterior interosseous branch of the i. Occurs at Guyon’s canal (between the
median nerve is the most commonly pisiform on the ulnar side and the
injured nerve. hamate on the radial side
e. Ectopic Ossification about the Elbow f. Ganglion
i. Heterotopic ossification (ectopic bone i. Benign condition
formation) – the formation of mature ii. Most common tumor of the hand
lamellar bone in nonosseous tissue iii. A cystic swelling over the dorsal or
ii. Myositis ossificans – abnormal volar aspects of the wrist
formation of mature lamellar bone in
inflammatory muscle. XIV. THE HAND
iii. Periarticular calcification – calcification a. Fractures
around the elbow. Calcific deposits i. It is important to check for rotational
consist of calcium pyrophosphate and deformities with fractures involving the
do not contain mature bone. metacarpals and phalanges since these
are not evident on radiographs

UST FMS MEDICAL BOARD REVIEW 2019 6 | SURGERY


ORTHOPEDIC SURGERY
ROBERT CHAN, MD/JOSE FERNANDO SYQUIA, MD

b. Dislocations vessels which are also located in this


i. Most of the dislocations of the digits area
(MCP, IP) are dorsal. iv. Zone 4 – underneath the transverse
c. Skier’s thumb carpal ligament; tendon injury here
i. Injury to the ulnar collateral ligament should make one suspect of a possible
of the first MCP joint median nerve lesion also
ii. Also called gamekeeper’s thumb v. Zone 5 – proximal to the transverse
d. Mallet finger carpal ligament; composed of the
i. Flexion deformity of the DIP joint due muscles of the forearm
to an avulsion of the extensor
mechanism from the DIP joint
ii. May be treated conservatively with a
splint.
e. Boutonniere deformity
i. Disruption of the central slip of the
extensor digitorum communis tendon
over the PIP joint.
ii. There is hyperextension of the DIP
joint with a flexion deformity of the PIP
joint.
f. Jersey finger
i. Avulsion of the FDP tendon from the
distal phalanx
ii. Treatment is surgical
g. Swan-neck deformity
i. There is PIP hyperextension and DIP
joint flexion.
ii. Caused by dorsal subluxation of the
lateral bands following flexor digitorum
superficialis rupture.
h. Intrinsic minus hand
i. Claw hand
ii. Hyperextension of the MCP’s and
flexion of the PIP’s with a flattened
metacarpal arch.
iii. Due to ulnar and median nerve palsies
or a Volkmann’s ischemic contracture
i. Intrinsic plus hand
i. Tight hand
ii. Flexion of MCP’s and extension of PIP’s
iii. Due to ischemia or fibrosis of the
intrinsics
j. Lumbrical plus hand
i. Due to lumbricals being tighter than
the extrinsic hand muscles (e.g., FDP
laceration distal to the lumbrical origin)
ii. There is paradoxical extension – Active
flexion of the MCP joint causes
extension of the PIP joint.
k. Flexor zones of the hand have been defined
by Verdan since results of treatment after
tendon injury vary depending on where the
injury is:
i. Zone 1 – insertion of the FDP to the
insertion of the FDS
ii. Zone 2 – insertion of the FDS to the A1
pulley. Injuries are essentially on the
tendon within the fibro-osseous tunnel.
This is relatively avascular and there is
a risk of tendon adhesions. This is
called “no man’s land”.
iii. Zone 3 – proximal end of the A1 pulley
to the distal edge of the transverse
carpal ligament; injury in this area may
result in damage to nerves and blood

UST FMS MEDICAL BOARD REVIEW 2019 7 | SURGERY


REVIEW TEST
ROBERT CHAN, MD

REVIEW TEST: ORTHOPEDIC SURGERY

Choose the best answer:


______10. Osteoarthritis of the spine is
______1. This is a condition in which bone density generally called:
has a T-score of less than -2.5: A. Spondylosis
A. Osteoporosis B. Spondylolysis
B. Rickets C. Spondylolisthesis
C. Osteomalacia D. Spondylitis
D. Osteopenia
______11. Hip condition in infants in which the
______2. The most common cause of osteomyelitis acetabulum is shallow resulting in hip
is: subluxation or dislocation:
A. Staphylococcus aureus A. Legg-Calve-Perthes disease
B. H. influenzae B. Slipped Capital Femoral Epiphysis
C. S. epidermidis C. Developmental Dysplasia of the Hip
D. Mycobacterium tuberculosis D. Blount’s disease

______3. Osteomyelitis presenting at one month is: ______12. This is a test for meniscal tears:
A. Acute hematogenous osteomyelitis A. Dial test
B. Posttraumatic osteomyelitis B. Pivot shift
C. Subacute osteomyelitis C. Lachman test
D. Chronic osteomyelitis D. McMurray test

______4. The most common malignant tumor of ______13. The ligament commonly injured in an
bone: ankle sprain:
A. Osteosarcoma A. Anterior talofibular ligament (ATFL)
B. Osteochondroma B. Calcaneofibular ligament (CFL)
C. Multiple myeloma C. Deltoid ligament
D. Metastatic tumor D. Anterior Inferior Tibiofibular
ligament (AITFL)
______5. Injury to a ligament is called:
A. Strain ______14. Prominence over the medial aspect
B. Fracture. of the 1st metatarso-phalangeal joint:
C. Sprain A. Hallux valgus
D. Dislocation B. Hallux varus
C. Hallux rigidus
______6. A fracture of the proximal third of the ulna D. Bunion
with dislocation of the radial head:
A. Colles’ fracture ______15. An herniated disk involving the disk
B. Smith’s fracture between C5 and C6 will usually compress which
C. Monteggia fracture nerve root:
D. Galeazzi fracture A. C5
B. C6
______7. Fracture with four fragments: C. C7
A. Compound fracture D. C8
B. Comminuted fracture
C. Open fracture ______16. The nerve commonly involved in a
D. Closed fracture cubital tunnel syndrome:
A. Axillary nerve
______8. The most common type of scoliosis: B. Radial nerve
A. Congenital scoliosis C. Median nerve
B. Adolescent idiopathic scoliosis D. Ulnar nerve
C. Neuromuscular scoliosis
D. Congenital kyphosis ______17. The shoulder joint commonly
dislocates in what direction:
______9. Lumbar disk herniation is most common A. Anterior
at: B. Posterior
A. L3-L4 C. Superior
B. L4-L5 D. Inferior
C. L5-S1
D. S1-S2

UST FMS MEDICAL BOARD REVIEW 2019 | SURGERY


REVIEW TEST
ROBERT CHAN, MD

______18. Golfer’s elbow involves which


structure:
A. Extensor carpii radialis longus
tendon
B. Brachioradialis
C. Extensor carpii radialis brevis
tendon
D. Pronator teres

______19. The nerve involved in a tarsal tunnel


syndrome:
A. Common peroneal nerve
B. Superficial peroneal nerve
C. Sural nerve
D. Tibial nerve

______20. In the hand, where is “No Man’s


Land”?
A. Zone 1
B. Zone 2
C. Zone 3
D. Zone 4

UST FMS MEDICAL BOARD REVIEW 2019 | SURGERY


UROLOGIC SURGERY
DAVID T. BOLONG, MD

UROLOGY BOARD REVIEW plethora of causes makes the physician aspire to


 Microscopic Hematuria defined as 3 or more look for the etiology.
RBCs per high power field in a spun urine.
Dipstick screened urine with positive 2 major classifications:
findings should be confirmed with a spun 1. obstructive symptoms: hesitancy,
urine and microscopy. False positives occur intermittency/poor flow, and terminal
with myoglobinuria, bacterial peroxidase, dribbling (HIP)
povidone and hypochlorite. 2. storage symptoms: frequency, urgency and
 Gross hematuria. As the name implies is nocturia (FUN)
very visible, confirmed again by urinalysis
because false positives can occur in highly Nocturia and nocturnal polyuria
colored urine of jaundice, beeturia, Frequent urination at night defined at >/= 2.
phenazopridine intake.
Nocturnal polyuria
Transient Hematuria caused by - Measured by a bladder chart preferably 2 or
1. UTI more days showing a consistent >30% of
2. Exercise the total urine daily output produced on
3. Menstruation sleeping and that includes the first voided
4. Fever in cases of microscopic urine in the morning.
hematuria
Significant hematuria Urinary incontinence
1. one episode of gross hematuria Involuntary leakage
2. symptomatic microscopic hematuria Types:
withour UTI or transient cause 1. stress urinary incontinence- involuntary
3. persistent microscopic hematuria incontinence occurring on effort, exertion,
with proteinuria coughing/sneezing
HEMOSPERMIA 2. Urgency urinary incontinence- incontinence
 Blood in the semen preceded or accompanied by the extreme
 Mostly are aymptomatic and benign in desire to urinate
patients below 40 years old. This occurs 3. Mixed urinary incontinence is a mixture of 1
after ejaculation. and 2
 If persistent and accompanied with voiding 4. Enuresis- or bedwetting. It could be pure
symptoms, then a work-up is in order. nighttime wetting now called
 See causes in PowerPoint. monosymptomatic nocturnal enuresis or
accompanied by daytime wetting now called
Lower urinary tract symptoms (LUTS) non-monosymptomatic nocturnal enuresis
5. Continuous incontinence- persistent non-
stop leakage often found in vesicovaginal
fistula or ectopic ureters.
6. Overflow incontinence. Spillage from a
markedly distended bladder.

B. Priapism- painful, persistent, prolonged erection


not related to sexual stimulation
Infections:

It is a complex of symptoms. It was


originally attributed to prostate symptoms but the
realization that symptoms can be caused by a

UST FMS MEDICAL BOARD REVIEW 2019 1 | SURGERY


UROLOGIC SURGERY
DAVID T. BOLONG, MD

Asymptomatic Bacteiuria (ABU) urethra eg tamzulocin, alfuzocin,


Defined as an individual with no symptoms terazocin, etc. This is the choice for
but with a midstream urine bacterial growth of >/= small prostates,
105 cfu/ml in 2 consecutive samples in women and b. 5 alpha reductase enzyme inhibitors-
in one single sample in men. In a catherized urine, a stops the conversion of testosterone to
growth of 102/cfu. DHT, thereby causing epithelial and
Prevalence is 1-5% of reproductive age stromal cell atrophy.
group women, 4-19% in elderly people, 20% of c. For large prostates a combination therapy
diabetics, 2-10% of pregnant women, 40% of of alpha-adrenergic blockers and 5
elderly people confined in institutions, 23-89% in alpha reductace inhibitors is preferred.
those with neurogenic bladders (10)
Treatment is needed pregnancy, when Invasive management to remove to prostate ranges
urologic intervention is needed or in kidney from open removal, scrapping, evaporating, thermal
transplant recipients. ablation of prostate tissues is done. Standard to
which new modalities are compared is the
Transurethral resection of the prostate.

Prostate cancer Treatment:


Principles:
1. Most prostate cancer are slow to
progress, that is why 10 years survival
free is a basis of most treatment
options.
2. Aim to remove prostate for cure in
organ-confined cancers in strong
patients where 10 years is very likely.
Sexually transmitted Diseases: 3. When ten years is not likely more
Most common are: conservative management are opted.
1. Chlamydia tracomatis 4. Advanced prostate cancer responds to
2. Neisseria Gonorrhea testosterone deprivation. Androgen
3. Chlamydia genitalium deprivation comes either with surgical
- Chlamydia - Most common STD, the crvix in castration or medical castration. Medical
women is monly affected, it causes urethritis castration is either central LHRH
in women. targeted therapy or local with androgen
- Co existent with gonorrhea in 30% of cases. receptor competitive therapy. LHRH
*Nucleic Acid Amplification test (NAAT) agonists work on the principle to
using vaginal/cervical swabs or the first 10 eventually depriving the pituitary gland
cc of urine. of LHRH which has an initial surge of
- Antibody- antigen reaction tests LHRH release, surge of testosterone
**(Chlamydia Rapid Test) for quick results then eventually a decline. The LHRH
about 90% compared to NAAT antagonists has a direct effect on the
- Doxycycline 100mg BID/ 7 days or pituitary LHRH and therefore earlier
azithromycin 1000 mg single dose effect and with studies showing less
Treatment of GC if present cardiovascular events in the long term.
Gonorrhea
- Always consider co-infection with c. Renal malignancy:
trachomatis. Clinical Manifestation
- It can cause genital and extragenital  Asymptomatic (i.e. incidental)
infections  classic triad of flank pain, hematuria and
- A urethral or endocervical/vaginal swab palpable renal mass
gram staining will show gram negative  Signs of advanced disease – bone pains,
intracellular diplococci. This test can miss adenopathy, pulmonary symptoms, weight
about 10% especially in asymptomatic loss, anemia
patients.  Non-reducing varicocele is pathognomonic
- NAAT of swabs or first voided urine is 98% of vena cava or renal vein tumor thrombus
sensitivity and specific. The drug of choice is formation
ceftriaxone 250mg IM or cefixime 400mg  Lower extremity edema, ascites, hepatic
single oral dose dysfunction and pulmonary embolization are
Prostate Benign Prostatic Hyperplasia results of IVC thrombus invasion
Treatment:  Paraneoplastic syndrome -- symptoms
1. Medical Management that occur at sites distant from a tumor or
a. Alpha adrenergic blockers- relaxes the its metastasis
smooth muscles in the prostate and o Mechanisms: may be secondary to
therefore loosening its grip on the substances secreted by the tumor or

UST FMS MEDICAL BOARD REVIEW 2019 2 | SURGERY


UROLOGIC SURGERY
DAVID T. BOLONG, MD
may be a result of antibodies
directed against tumors that cross- 4. Altered cell cycle kinetics
react with other tissue a. Aberrant activation of additional
Stone prevalence radioopacity Formation signal transduction pathways in RCC
composition pH may also contribute to altered cell
Calcium 80-85% ++ Variable cycle kinetics
oxalate (6+/-/4) b. Mammalian Target of Rapamycin
Calcium 10% ++ variable (mTOR) is a pathway of signal
phosphate transduction involved in cell cycle
+ calcium regulation
oxalate
Uric acid 5-10% - Acidic
(5.5+/-.4)
Calcium rare +++ Alkaline
phosphate (>5,5)
Struvite 2-20% + Alkaline
infectious
stone
(Magnesium
Ammonium
Phosphate)
cystine ? +

common paraneoplastic symptoms include:

Trauma
Renal Trauma
When to suspect:
1. flank pains,
2. gross hematuria,
3. microscopic hematuria
the degree of hematuria is not predictive of
the degree of renal injury
Assessment:
1. hemodynamically stable: patients can have
contrast enhanced ct can of the whole
abdomen and pelvis.
Tumor Biology and Clinical Implications a. blunt trauma are mostly treated
Study of the tumor biology of RCC provides insight conservatively- supportive care,
into its refractory nature and, through elucidation of angioembolization of bleeders or
the regulatory pathways (e.g. VEGF, mTOR), has drainage with a ureteral
yielded agents with clinical benefit for advanced catheter. Only in expanding
disease. hematoma or worsening
1. Resistance to cytotoxic therapy urinoma is exploration
a. Expression of multidrug resistance warranted .
proteins (act as energy-dependent b. penetrating injuries are usually
efflux pumps for hydrophobic explored, and retroperitoneal
compounds like chemotherapy) exploration done when there is
2. Immune tolerance an expanding hematoma. In the
a. tumor interacts with the host past a one-shot IVP was done at
immune system 10 minutes post-injection of
b. Tumor-infiltrating immune cells can contrasts at 2ml/kg bolus.
be readily isolated from RCC (e.g. Nowadays, if not done palpation
cytotoxic T cells, dendritic cells, and of the contra leateral kidney and
helper T cells) palpation of good pulses in the
3. Angiogenesis renal pedicle is enough. For
a. Vascular Endothelial Growth Factor emergent cases where
(VEGF) is the primary angiogenesis retroperitoneal exploration was
inducer in clear cell RCC (see not done a post op ct urography
Diagram) can be done to make sure none
b. Increased expression of VEGF due was missed.
to mutation of VHL gene (produces
protein that suppresses VEGF)

UST FMS MEDICAL BOARD REVIEW 2019 3 | SURGERY


UROLOGIC SURGERY
DAVID T. BOLONG, MD

2. Hemodynamically unstable are brought to


the operating room for exploration if:
a. in blunt trauma there is an
expanding hematoma or cases
where embolization of bleeder
fails. This usually ends in
nephrectomy
b. in penetrating injuries. No
retroperitoneal exploration if no
retroperitoneal hematoma or
urinoma. Inspection of ureter
also at that time is the best way
to assess ureteral injuries.

Treatment:
Basic rule is to avoid retroperitoneal
exploration in penetrating or blunt trauma in the
absence of an expanding hematoma or expanding
urinoma as this usually leads to nephrectomy.

UST FMS MEDICAL BOARD REVIEW 2019 4 | SURGERY


REVIEW TEST
DAVID T. BOLONG, MD

UROLOGY REVIEW TEST _____6. Which statement about prostate cancer is


false?
_____1. 50-year-old man presents with microscopic A. Most of the time very aggressive, short
hematuria. Which will dictate that he needs progression time and there needs very
aggressive diagnostic procedures. aggressive management.
A. Microscopic hematuria with exercise B. Active surveillance is an option for a 70-
B. Microscopic hematuria after a bout of year-old man with multiple co
viral upper respiratory tract morbidities.
C. Microscopic hematuria with dysuria of C. Androgen deprivation alone is enough
one day duration as initial management for stage 4
D. Microscopic hematuria with proteinuria malignancies
D. The LHRH agonists for the treatment of
advance prostate cancer is to induce the
_____2. Which lower urinary tract symptoms tell you testis to produce more testosterone.
of bladder outlet obstruction?
A. Urgency and frequency _____7. Which of the following reason/s tell why
B. Hesitancy and feeling of incomplete renal malignancies are resistant to chemotherapy/
voiding radiotherapy:
C. Urgency incontinence A. Expression of multidrug resistance
D. Nocturia proteins (act as energy-dependent efflux
pumps for hydrophobic compounds like
chemotherapy)
_____3. A 21-year-old promiscuous male executive
B. Mutation of VHL gene, accumulation ho
comes with a slight dysuria and yellowish discharge.
He was given cefixime 200 mg capsule 2 capsules Hypoxic induction factor, causing VEGF /
one take. What will you to make a diagnosis? angiogenesis induction
A. Urethral swab and gram staining C. mToR (mammalian Target of
B. Urinalysis of first voided urine Rapamycin)
C. Urine nucleic acid amplification test of a signaling pathway is altered.
first voided urine for GC and CT
D. All of the above
D. Give him ceftriaxone 250 mg IM After
negative skin test
_____8. Inhibition of complexation and aggression
_____4. A 40-year-old asymptomatic woman is on
of crystals above the supersaturation level to form
her 7 weeks of gestation. She presents on routine
urolithiasis is best exemplified by this stone inhibitor:
urinalysis showing 15-20 wbc, 2 bacteria/hpf. You
A. Sodium
should:
B. Calcium
A. Give betalactamase inhibitors.
C. Potassium citrate
B. Give her nitrofurantoin.
D. Sodium citrate
C. Assure her she is alright.
D. Give her sulfonamides.
_____9. A 20-year-old women was in a vehicular
accident. She is stable. Ct program showed a 1 cm
_____5. A 60-year-old man presents with LUTS-
laceration of the left renal parenchyma. With a 30 cc
hesitancy, straining and urgency. DRE- showed a
perirenal hematoma. You should:
grade 1 prostate. Ultrasound KUB is normal. You will
A. Go for renal exploration
give:
B. Go for selective renal artery
A. Alpha adrenergic blocker
embolization
B. 5 alpha reductace inhibitor
C. Just monitor her.
C. Recommended TURP D. Transfuse immediately and prepare for
D. All of the above surgery electively.

_____10. BPH stands for:


A. Benign prostatic hypertrophy
B. Benign prostatic hyperplasia
C. Beginning prostatic hyperplasia
D. Beginning prostatic hypertrophy

UST FMS MEDICAL BOARD REVIEW 2019 1 | SURGERY


PLASTIC SURGERY
ROCELYN ANN P. GANNABAN, MD

FTSG STSG
PLASTIC SURGERY
Primary
+ -
 “plastikos” Contraction
 Has no organ system of its own Secondary
- +
 It is based on principles rather than specific Contraction
procedures
Hair
 Darling of the media because of aesthetic or + -
Growth
cosmetic surgery
Sensation + -
Will there be a scar?
 When a full thickness injury occurs or an Sebum + -
incision is made, there is ALWAYS a scar
 Will I have an inconspicuous fine line scar?
 Factors affecting scarring: OPTIONS: Allografts, Xenografts, Autografts,
o Age Amnion
o Oily vs. Dry skin
o Pigmented vs. Pale skin Skin Graft Survival
o Children  1st 24-48 hours: serum or plasmadic
o Local & Systemic factors imbibition
o Tension on closure  2-5 days: inosculation or alignment of
o Unpredictable differences between capillaries
individual patients  >5 days: revascularization
o Surgical technique
 Maximal contraction occurs when a scar Wounds containing >10 5 organisms per gram of
crosses the lines of minimal tension at a tissue will not support a skin graft
right angle
Skin Flaps
WOUND CLOSURE  Rotational Flaps- semicircular flap rotates to
Suturing Techniques cover an adjacent defect
 Simple interrupted- most common  Transposition flap- angular flap rotates to
 Vertical mattress & Horizontal mattress- cover an adjacent defect
evert edges and provide hemostasis  Interpolation flap- covers a nearby but not
 Continuous sutures- not as precise as adjacent defect
interrupted sutures  Advancement flap- no rotation or lateral
 Skin staples, tapes, adhesives movement
 DOG EARS DO NOT DISAPPEAR ON THEIR
OWN WOUND HEALING
A. Hemostasis & Inflammation
 Removing the sutures may be more  Platelet aggregation, degranulation, &
important than placing them activation
• Face: 3-5 days  Formation of fibrin clot
• Body: 7 days or less  PMN (persistence results in chronic wound)
• Buried dermal sutures  Monocytes & macrophages at 48-96 hours
 Evert wound edges- always goes away (absence results in abnormal healing)
 Lymphocytes at 7 days- bridge to next
Z plasty phase
 Limbs of the Z must be equal in length to
the central limb B. Proliferation
 Angles can vary from 30-90 degrees  Days 4-12
depending on desired length gain  Reestablish tissue continuity
 Involves transposition of 2 triangular flaps  Fibroblasts & endothelial cells
 Increases the length of a scar  Collagen type III
 Changes the direction of a scar
C. Maturation & Remodeling
Skin Grafts  Reorganization of previously synthesized
 Primary contraction is the immediate recoil collagen
of freshly harvested grafts  Collagen type I
 Secondary contraction is the contraction of  Remodeling continues for 6-12 months
healed grafts due to myofibroblast activity
The mechanical strength of the scar never achieves
that of uninjured tissue.

UST FMS MEDICAL BOARD REVIEW 2019 1 | SURGERY


PLASTIC SURGERY
ROCELYN ANN P. GANNABAN, MD

 Maxillary retrusion if too


Nutritional deficiencies affecting wound healing: early
 Arginine
 Vitamin A & C SKIN TUMORS
 Zinc  Actinic Keratosis- Pre SCC
 Protein-energy malnutrition  Bowen’s Disease- cutaneous SCC in situ
 Leukoplakia- mucosal pre SCC
 Wound healing represents the response of  Marjolin’s ulcer- SCC from chronic wound
an organism to reestablish homeostasis in
physical disruption  Indications for biopsy: ABCD
 Two processes Asymmetry
Border Irregularity
Scar formation: substitution with a different cellular Color variability
matrix Diameter> 8mm
• Nerves, skin, muscle, liver in cirrhosis  Basal cell Carcinoma

Regeneration: reactivating developmental pathways Most common malignancy worldwide


to recreate architecture Arise from the basal layer of the epidermis
• Liver, bone Raised borders with a pearly central area
Subtypes:
PEDIATRIC PLASTIC SURGERY • Nodular & superficial- least
Clefts aggressive
 Orofacial cleft is the most common • Micronodular
congenital anomaly • Cystic
 1 in 500 live births • pigmented
 Lower in African Americans • Infiltrating & morpheaform- most
 Higher in Asians aggressive

2 theories: Treatment: Wide excision, Moh’s surgery, RT, topical


 Failure of fusion of medial nasal and chemo
maxillary prominence  Squamous cell carcinoma
 Mesodermal penetration theory: Failure to Risk factors:
penetrate the epithelial bilayer of the face • UV damage
• Chemical carcinogens
• Risk factors • Chronic irritation
Increased parental age • Cigarette smoking
Drug use- phenyTEN, Vitamin A • HPV infection
Infections Arise from basal keratinocytes
Smoking Treatment : Surgical excision with wide margins,
Family history: 4% 1 parent LN dissection if indicated, RT
9% 1 parent, 1
child MAXILLOFACIAL TRAUMA
 The incisive foramen divides the primary
palate (lip, alveolus, nose, premaxilla) from  The first phase of care of a patient with
the secondary palate (hard & soft palate) maxillofacial trauma is the activation of
 Cleft Lip ATLS protocol
o Microforme/Forme Fruste- isolated Concomitant injuries are the rule rather than the
incomplete fusion of orbicularis oris exception
sphincter SCI in 10% of patients
o Incomplete cleft lip- connected by a  Normal findings:
bridge of tissue at the nasal sill • Symmetrical features
(Simonart’s band) • Equal palpebral features
o Complete cleft lip- extends into the • Present Nasolabial folds
nasal cavity and alveolar ridge • Normal Bite
o Tx: Cheiloplasty • Able to open mouth fully
 Rule of 10’s: 10 lbs, 10  Physical examination:
weeks, 10 g/dL Hgb • Lacerations
 Cleft Palate • Bony step offs
o Submucous cleft- Intact mucosa • Instability
 Zona pelucida • Tenderness
o Incomplete cleft palate- cleft of the • Ecchymosis
soft palate or uvula • Facial asymmetry
o Complete cleft palate- cleft of the • Deformity
soft and hard palate • Malocclusion
o Tx: Palatoplasty/Uranoplasty
 Before the child starts to
speak (by age 18 months)
UST FMS MEDICAL BOARD REVIEW 2019 2 | SURGERY
PLASTIC SURGERY
ROCELYN ANN P. GANNABAN, MD

 Diplopia on upward gaze


 Imaging studies

Xray
• Water’s view: single best image for
facial fractures
• Panoramic xray: single best image
for mandible
CT scan
Gold standard
 Treatment:
ORIF
External Fixation
Closed reduction
Conservative • Patients with acute burn injury should
NEVER receive prophylactic antibiotics
BURNS Promote fungal infection and resistant
 Initial evaluation: microorganisms
Airway management Tetanus vaccination should be given
Evaluation of other injuries
Estimation of burn size • Most formulas compute for fluid
Diagnosis of carbon monoxide or cyanide requirements using burn size
poisoning • Burn size is estimated by computing for
 First Degree Burns percentage of total body surface area
Epidermis involved (%TBSA)
Red and very painful • Superficial burns (1st degree) should not be
Superficial sunburn without blisters included in the calculation
Dry in appearance Common error
Healing occurs in 3-5 days, desquamates
Tx: Pain control and moisturizer

 Second degree superficial burns


Blisters can be present
Involve the entire epidermis and upper dermis
Pink, red in color, painful and wet
Blanch with pressure
Heal in 10-21 days without grafting
Scarring is usually minimal
Tx: Analgesics, moisturizer, fluid and electrolyte
management

 Second degree deep burns


Can be red or white in appearance
Involves the entire epidermis and most of the
dermis
Sensation can be demised, but present
Blanching is sluggish or absent
Tx: Full thickness will most likely need excision
& skin grafting to heal

 Third degree burns


All skin layers Pediatric patient can be calculated as adult size
Extend into the subcutaneous tissues by 15 years old
Areas can appear, black or white  Computations are at best an estimate
Can appear leathery in texture  Titrate fluid to urine output
Will not blanch when pressure is applied Adults: 30cc/hour
No pain – Children: 1 cc/kg/hour
Will require immediate coverage – In electrical burns: 100 cc/hour

 4th degree burns


Full thickness burns
Extend down to the muscle and bone
Will require immediate debridement of necrotic
tissue and coverage

UST FMS MEDICAL BOARD REVIEW 2019 3 | SURGERY


PLASTIC SURGERY
ROCELYN ANN P. GANNABAN, MD

AESTHETIC SURGERY
 Darling of the media
 Patients have no anatomic pathology
 Provides a patient a way to present
themselves to the world in how they see
themselves
 Improves wellbeing and increases
confidence

UST FMS MEDICAL BOARD REVIEW 2019 4 | SURGERY


REVIEW TEST
ROCELYN ANN P. GANNABAN, MD

PLASTIC SURGERY REVIEW TEST QUESTIONS:

_____ 1. True about scar formation after surgery: _____ 8. Which of the following are risk factors for
A. There will be no scar if the incision is placed development of orofacial clefts?
along Langer’s lines A. Parental age 18 years old and below
B. Oily or pigmented skin produces more B. Vitamin C
unsightly scars C. Seizure disorder
C. Wrinkled, pale, dry skin result in obvious D. Family history if strabismus
scars
D. Children scar better _____ 9. Which of the following structures serves as
a landmark to divide clefts of the primary from the
_____ 2. Elective incisions or excisions are planned secondary palate?
when possible so that the final scars will be parallel A. First premolar teeth
to the relaxed skin tension lines. Which of the B. Incisive foramen
following statements is correct? C. Uvula
A. Relaxed skin tension lines lie perpendicular D. Eustachian tube opening
to the underlying muscles
B. Incision lines should be made parallel to _____ 10. Cutaneous squamous cell carcinoma in
underlying muscle fibers situ
C. Incisions around the lips are best placed A. Marjolin’s ulcer
parallel to the vermillion border B. Basosquamous carcinoma
D. Incisions should be placed parallel to joints C. Bowen’s disease
when crossing flexion lines to prevent D. Actinic keratosis
contracture
_____ 11. Squamous cell carcinoma developing from
_____ 3. True regarding skin closure: chronic or nonhealing wounds:
A. Subcutaneous or dermal sutures will prevent A. Marjolin’s ulcer
the scar from widening B. Cushing ulcer
B. Dog ears will disappear after scar C. Curling’s ulcer
maturation D. Rodent ulcer
C. Skin adhesives can replace sutures for
wound closure and eversion _____ 12. The best coverage for third degree burn
D. Continuous sutures are not as accurate as wounds:
interrupted sutures A. Xenograft
B. Allograft
_____ 4. What is the most appropriate cleansing C. Autograft
solution for a pressure ulcer? D. Sterilized amnion
A. Hydrogen peroxide
B. Povidone iodine _____ 13. The initial fluid infused during burn
C. Normal saline solution resuscitation
A. Normal saline solution
_____ 5. Which of the following are associated with B. D5NSS
wound healing? C. Lactated Ringer’s solution
A. Alpha tocopherol D. D5LRS
B. Manganese
C. Retinol _____ 14. Which of the following topical agents is
D. Iron associated with metabolic acidosis?
A. Silver sulphadiazene
_____ 6. During the first phase of wound healing, B. Silver nitrate
which of the following is first noted at the site? C. Mafenide acetate
A. Neutrophils D. Fucidic acid
B. Platelets
C. Monocytes _____ 15. Skin graft survival during the first 24
D. Lymphocytes hours is through:
A. Inosculation
_____ 7. Which of the following tissues heal B. Plasmadic imbibition
primarily by regeneration? C. Capillary alignment
A. Muscle D. Angiogenesis
B. Skin
C. Bone _____ 16. Malocclusion can be seen in fractures of
D. Pancreas the following:
A. Mandible
B. Zygoma
C. Maxilla
D. All of the above

UST FMS MEDICAL BOARD REVIEW 2019 | SURGERY


REVIEW TEST
ROCELYN ANN P. GANNABAN, MD

_____ 17. Which of the following is likely to have


the most negative effect on wound healing?
A. Age 65 years and older
B. Hypoalbuminemia
C. Vegetarian diet
D. Diabetes mellitus, poorly controlled

_____ 18. True regarding the Rule of nines:


A. The perineum is equivalent to 1% of TBSA
B. A good approximate for 1% TBSA is the
palm of the patient’s hand
C. Superficial burns are not included in the
calculation
D. All of the above

_____ 19. A 3-year-old patient comes in with a scald


burn to the anterior upper chest approximately twice
the size of his palm. He is 15 kg. Which of the
following statements is appropriate?
A. Admit the patient and start on PLRS and D5
containing fluids
B. Refer to a plastic surgeon for skin grafting
C. Discharge with topical ointments
D. Call the police, this is clearly a case of child
abuse

_____ 20. True regarding burn fluid resuscitation:


A. Parkland’s formula is most commonly used
for the first 24 hours post injury
B. Saline solution should be used for
resuscitation because it most nearly
resembles the post burn plasma state
C. Serum hematocrit should be requested
every 8 hours to monitor hydration status
D. All of the above

UST FMS MEDICAL BOARD REVIEW 2019 | SURGERY


NEUROSURGERY
EDGARDO T. TAN, MD
MARIA LOURDES D. MAGLINAO, MD
RICARDO C. ENRILE, MD & GILBERT J. RAÑAO, MD

NEUROSURGERY
 RAISED INTRACRANIAL PRESSURE
RAISED INTRACRANIAL PRESSURE  Monro-Kellie Doctrine of Intracranial
Monro-Kellie Doctrine of Intracranial Pressure Hypertension (ICP)
Herniation Syndromes  Cranial vault is a rigid structure and
therefore volume inside the cranium is fixed
Management of Increased ICP
 Total volume of brain (80%), blood (10%),
TRAUMA (BRAIN & SPINAL CORD) and CSF (10%) determines ICP usually in
Glasgow Coma Scale (GCS) the range of 5-15 mmHg
Skull Fractures  Any increase in volume of one of the cranial
Brain Injury constituents must be compensated by a
Intracranial Bleeding decrease in volume of another.
Spinal Cord Injury
Peripheral Nerve Injury Compensation is done by extrusion of CSF
from the intracranial cavity into the thecal
CEREBROVASCULAR DISEASE sac of the spine and by extrusion of venous
Subarachnoid Hemorrhage (SAH) blood from the cranium.
Intracranial Aneurysm  Relationship between ICP and intracranial
Arteriovenous Malformations (AVM‟s)
Hypertensive Intracerebral Hemorrhage (HIH) volume is described by a sigmoidal
Cerebral Amyloid Angiopathy (CAA) pressure-volume curve. Increased ICP can
Cavernous Malformations injure the brain by either focal mass lesions
cause by pressure and shifting, and by brain
NEOPLASM herniation
BRAIN – Metastasis, Astrocytoma, & Meningioma
Vestibular Schwannoma, Pituitary Adenoma,
Lymphoma, Craniopharyngioma
SPINAL CORD –
Extradural Tumors
Intradural – Extramedullary Tumors
Intradural – Intramedullary Tumors

INFECTIONS
Brain Abscess
Pott‟s Disease

DEGENERATIVE SPINE
Cervical Disc Syndrome
Lumbar Disc Syndrome
 Consider therapy for high ICP when ICP is
Cauda Equina Syndrome
>/= 20-25 mm Hg
Lumbar Spinal Stenosis

CONGENITAL
Hydrocephalus
Craniosynostosis
Spinal Dysraphism
Chiari Malformation
Tethered Cord Syndrome

PERIPHERAL NERVE
Tumor
Entrapment Neuropathy (Carpal Tunnel
Syndrome)
Brachial Plexus
Thoracic Outlet Syndrome

FUNCTIONAL NEUROSURGERY
Deep Brain Stimulation – Parkinson Disease
Epilepsy Surgery
Trigeminal Neuralgia
Stereotactic Radiosurgery (SRS)
UST FMS MEDICAL BOARD REVIEW 2019 1 | SURGERY
NEUROSURGERY
EDGARDO T. TAN, MD
MARIA LOURDES D. MAGLINAO, MD
RICARDO C. ENRILE, MD & GILBERT J. RAÑAO, MD

 Herniation Syndrome - 100% oxygen, Plain NSS, CPP =/>


1. Subfalcine (Cingulate) Herniation, most 90 mm Hg (Cerebral Perfusion
common Pressure)
- Cingulate gyrus herniates under the - 20% Mannitol, Hypertonic Saline
falx
Solution, Steroids, Diuretics
- Due to lateral supratentorial lesion
- Clinical presentation is usually  Surgical: Ventriculostomy,
asymptomatic but warns of impending Craniotomy, Decompressive
transtentorial herniation Craniectomy
- May cause frontal lobe infarction  Intensive Care: ICP Monitoring,
when the Anterior Cerebral Artery Barbiturate Coma, Hypothermia
(ACA) is kinked
2. Trans-tentorial Herniation: Uncal-  TRAUMA:
herniation
 Glasgow Coma Scale
- most dramatic & second most
common
- herniation syndrome
- herniation of the inferomedial most
part of the temporal lobe between
the rostral brain stem & tentorial
edge into posterior fossa;
compression of the diencephalon and
midbrain as well as the posterior
cerebral artery (PCA) resulting to
occipital infarction.
- syndrome consists of progressively
impaired consciousness, dilated
ipsilateral
o pupil (CN III), and contralateral
hemiplegia
- Kernohan‟s phenomenon (ipsilateral
hemiplegia to the side of dilated pupil)
 Skull Fractures:
- Cushing‟s Triad: widening pulse
 Depressed fractures - double density on
pressure (rising systolic, declining
skull x-ray
diastolic), irregular respirations, and
 Simpl e/Linear fractures or compound
bradycardia ~ signifies severe
fractures -may increase risk of hematoma &
increased ICP
infection
- Brainstem hemorrhage “Duret‟s”, due
 Basal skull fractures – may cross sinuses
to a shearing of basilar artery‟s
that may increase the chance of infections,
perforating vessels.
& pneumocranium
3. Tonsillar Herniation:
• retroauricular ecchymoses (Battle‟s sign)
- tonsils of the cerebellum herniates
• periorbital ecchymoses (raccoon eyes)
thru foramen magnum into the upper
• hemotympanum
spinal canal, compressing the medulla
• CSF rhinorrhea, otorrhea
& blocking CSF flow (obstructive
• suspect with Lefort II or III midface
hydrocephalus)
fracture
- cardiorespiratory impairment,
hypertension, high pulse pressure,
Cheyne-Stokes respirations, Brain Injury:
neurogenic Traumatic Brain Injury (TBI)
hyperventilation, & impaired LOC  Primary damage (PI): Biomechanical
- posturing: opisthotonus, decorticate, injury, instantaneous onset, clinical effects
decerebrate rigidity. are immediate, maximal, and irreversible
e.g.: Concussion, Contusion, Acute
 Management of Increased ICP Subdural Hematoma, DAI, tSAH
 Mechanical:  Secondary damage: Complication of PI,
- Head elevation at 30-40 degree delayed in onset, clinical effects are
- Keep head & neck in neutral position delayed, progressive, preventable, and
- Hyperventilation to pCO2 30-40 mm treatable
Hg e.g.: Hypotension, Hypoxia, Ischemia,
 Medical: Brain swelling, Infection, Hematoma
- Sedation, Normothermia, formation, Herniation syndrome.
Anticonvulsant

UST FMS MEDICAL BOARD REVIEW 2019 2 | SURGERY


NEUROSURGERY
EDGARDO T. TAN, MD
MARIA LOURDES D. MAGLINAO, MD
RICARDO C. ENRILE, MD & GILBERT J. RAÑAO, MD

 Coup – damage at site of blow thick, friable, vascularized outer


 Contre-coup – damage at opposite site of membrane and a thin, lucent inner
blow membrane. Risk factors: elderly, alcoholic,
 Contussion – hemorrhagic bruising of the patients with CSF shunt or on
brain on bony prominences (falx, sphenoid anticoagulants. CT scan shows low density
wing, roof of orbit, temporal fossa, concave mass (due to liquefied clot).
petrous bone, etc.) Management is burr hole craniostomy for
 Diffuse Axonal Injury – due to diffuse evacuation and drainage of CSH.
axonal shearing; may cause injury to  Subacute Subdural Hematoma: usually in
blood vessels resulting to hemorrhagic between solid to liquefied clot. An attempt
foci, often the cause of decreased LOC if for burr hole craniostomy can be done
no space occupying lesion is seen on CT before a final bone flap should clot failed
scan to be evacuated.
 Traumatic ICH: frontal and temporal lobes
 Intracranial Bleeding: most commonly injured. Indication for
 Acute Epidural Hematoma (AEH): evacuation –
 Usually arterial in origin esp. from a if hematoma is >/= 50 ml and superficial.
laceration of the middle meningeal artery
(85%) associated with a fracture in the
squamous portion of the temporal bone
 Classic story (10%) is triphasic with a mild
head injury leading to a transient
alteration of consciousness followed by a
“lucid interval” where patient regained
consciousness or is relatively symptom
free, followed by obtundation, focal
deficit, and ipsilateral pupillary dilatation.
 Plain cranial CT scan – high density
biconvex mass with a sharp margin “lens-
shaped”
 Management usually craniotomy for
evacuation of AEH for thickness >/= 1.5
cm and symptomatic; volume of > 30 ml
and a shift of >/= 5 mm midline shift  Spinal Cord Injury
 Prognosis: good if managed promptly  Complete SCI:
 Subdural Hematoma: arise from rupture a Transection of the Spinal Cord -
bridging vessel between the surface of the lowest level is S4-5 (anal sphincter &
brain and the skull (e.g. cortical artery, perianal sensation); Flaccid paralysis + total
bridging vein, or venous sinus) as a result loss of sensory & motor functions
of a brain trauma.  Incomplete SCI:
 CT scan – typically a high density crescent  Central Cord Syndrome: most common
shaped mass with concave medial and with weakness upper > lower extremities,
convex lateral border that may extend a “dissociated” sensory loss, a sacral
diffusely; may cross suture lines but no sparing and sphincteric dysfunction
dural attachments.  Posterior Cord Syndrome: due to posterior
 Management – craniotomy for hematoma trauma or involvement of posterior spinal
>/= to 1 cm thick and >/= 5 mm midline artery; have joint position and vibration
shift, whereas for CSH, a burr hole sensation loss.
craniostomy is done for evacuation and  Anterior Spinal Artery Syndrome: due to
drainage of CSH anterior cord compression or compression
 Acute (up to 48 hours) of the anterior spinal artery; manifested
 Subacute (after 48 hrs to 3 weeks) by dissociated sensory loss of both pain
 Chronic (>/= 3 weeks) and temperature with sparing of touch,
 Acute Subdural Hematoma: overall joint position and vibratory sensation.
prognosis is poor because of concomitant Bilateral paraplegia and sphincter
brain injury. CT scan shows a high-density dysfunction. Worst prognosis with only-
concave mass. Management is 10%-20% recover functional motor
craniotomy. control.
 Chronic Subdural Hematoma: usually the  Brown Sequard‟s Syndrome: ipsilateral
clot liquefies and becomes bounded by a weakness and loss of proprioception and
UST FMS MEDICAL BOARD REVIEW 2019 3 | SURGERY
NEUROSURGERY
EDGARDO T. TAN, MD
MARIA LOURDES D. MAGLINAO, MD
RICARDO C. ENRILE, MD & GILBERT J. RAÑAO, MD

vibratory sense below lesion usually due


to a penetrating trauma such as stab  CEREBROVASCULAR DISEASES:
wound. HEMORRHAGIC: Subarachnoid
Hemorrhage
 ASIA Impairment Scale - Most common cause of SAH – trauma
A = Complete, no sacral sparing
B = Incomplete, only sensory preserved
C = Incomplete, only motor preserved; key
muscles < grade 3
D = Incomplete, only motor preserved; key
muscles > grade 3
- Most common cause of spontaneous SAH:
E = Normal sensory & motor
o Aneurysm (75%-80%)
o Arteriovenous Malformation (4%-
 Peripheral Nerve Injury:
5%)
 Classification:
- Intraparenchymatous Hematoma:
- Seddon Grade:
o Hypertensive Intracerebral
o Neurapraxia – nerve intact but fails to
Hemorrhage (HIH) Cerebral Amyloid
function, recovery within hours to days
Angiopathy (CAA)
o Axonotmesis – axon disrupted but nerve
sheath intact with Wallerian
 Aneurysm
degeneration of axon segment distal to
- Sudden severe headache ~ “worst headache
injury; recovery
of my life”; peak age 35 – 65 years old
1 mm/day
- 1%-3% annual risk of rupture
o Neurotmesis – nerve completely
- Risk factors: HPN, smoking, alcohol or
severed; need surgical repair for
sympathomimetic drug abuse, family history
recovery (polycystic kidneys), & collagen vascular
- Sunderland Grade: disease (Ehlers Danlos Type 4, Marfan
o Grade 1 - Local myelin damage. Axons syndrome)
preserved. No degeneration - Clinical features: sudden severe headache,
o Grade 2 - Endoneural tube preserved. vomiting, meningismus, photophobia,
Axon degeneration decreased level of consciousness, cranial
o Grade 3 - Loss of endoneural tube nerve palsy, hemiparesis, etc.
continuity. Perineurium - Site: 85% in the anterior circulation
intact. Axon degeneration (AComA, PComA aneurysms) and 15%
o Grade 4 - Endoneural tube and found in the posterior circulation (Basilar &
Vertebral Artery Aneurysms)
perineurium disrupted. Epineurium
- Overall mortality 50%-60% in the first 30
intact. Axon degeneration
days;
o Grade 5 - Complete loss of neural - Risk of rebleeding is 50% w/in 6 weeks,
continuity. 25% within the first 2 weeks after the ictus,
50% within 6 months; then 2-3% per year
thereafter.
- Vasospasm rare before day 3 & peak at 6-8
- days post-SAH, typical risk period is 3-10
days, is a common complication (25%-30%
symptomatic); causes death in 14-20%
- Multiple aneurysms – 20% - 25%
- Modified Fisher Scale

- Management: EMG-NCV may be helpful in


assessing nerve integrity. Microsurgical repair
unless nerve is known to be intact (suture
nerve sheaths =/- nerve graft
UST FMS MEDICAL BOARD REVIEW 2019 4 | SURGERY
NEUROSURGERY
EDGARDO T. TAN, MD
MARIA LOURDES D. MAGLINAO, MD
RICARDO C. ENRILE, MD & GILBERT J. RAÑAO, MD

Modified Fisher Scale younger person or a patient without clear


risk factors for hemorrhage; MRI/MRA
essential for the initial diagnosis of AVMs.
Digital Subtraction Angiography (DSA) is the
definitive diagnostic procedure to require
hemodynamic assessment.

 Hypertensive Intracerebral Hemorrhage


(HIH)
- accounts for 10%-15% of all cases of
stroke and is associated with the highest
- Diagnosis: CT scan without contrast, Lumbar mortality rate (only 38% surviving the
o Puncture, CTA, MRA, DSA 1st year)
- Complications: Rebleeding, Vasospasm, - Most common site is putamen
o Hydrocephalus (thalamus, cerebellum, pons, &
- Vasospasm: Higher risk in poor grade SAH, subcortical white matter)
o large subarachnoid blood load, - Risk factors: hypertension, smokers,
intraventricular hemorrhage, heavy alcohol user, non-compliant with
smokers; treatment – Nimodipine, anti-HPN regimens, low serum
Triple H therapy (hypertension, cholesterol, high intensity statin therapy,
hypervolemia, hemodilution) genetic factors.
- Neurological findings depend on site and
size of bleeding; vomiting, altered
sensorium & high BP
- CT scan is the choice of imaging
procedure
- 30-day mortality rate for hemorrhagic
stroke is 40%-80%. Approximately 50%
of all deaths occur within the first 48
hours.
- Clinical predictors of mortality: Age,
LOC, size & location of hematoma on CT
and presence of intraventricular blood.
- Treatment: Endovascular coiling or surgical
o Clipping; unruptured aneurysm best
treated with endovascular coiling

 ArterioVenous Malformation
- tangle of abnormal vessels with no
intervening capillary beds or brain
parenchyma (nidus)
- present in younger age group 10-30 years
old
- Presentation: ICH (40%-60%), seizures
(50%), focal neurological deficit, bruit
- Diagnosis: high density blood on CT without
contrast raising suspicion of AVM in a-
UST FMS MEDICAL BOARD REVIEW 2019 5 | SURGERY
NEUROSURGERY
EDGARDO T. TAN, MD
MARIA LOURDES D. MAGLINAO, MD
RICARDO C. ENRILE, MD & GILBERT J. RAÑAO, MD

- Treatment: Medical and Surgical* - Diagnosis: CT& MRI scan with


* patients with cerebellar hemorrhage > 3 cm who contrast, stereotactic biopsy (tissue
are neurologically deteriorating or with diagnosis)
radiographic evidence of symptomatic brainstem - Management
compression or hydrocephalus; young patients - Conservative management
- serial imaging for slow
with moderate to large lobar hemorrhage who are growing/benign lesions
clinically deteriorating. - Medical management
Best therapy still unclear for all other patients.

 corticosteroids to reduce vasogenic


cerebral edema
 pharmacological treatment for pituitary
tumours (see Pituitary Adenoma
section) surgical
 excisional: total, partial,
decompressive, palliative
 shunt if CSF flow is blocked
 Cerebral Amyloid Angiopathy (CAA)  radiotherapy - stereotactic radiosurgery
- Presence of pathologic amyloid deposition in (Gamma-knife, Linear Accelerator)
 chemotherapy – e.g. alkylating agents
the media of small cortical vessels; tends to (temozolomide)
be more superficial (lobar). They can
hemorrhage multiple times. Metastasis:
- Treatment is surgical if superficial; usually
medical management & family counselling
 Cavernous Malformations
 benign vascular hamartoma consisting of
irregular thick and thin walled sinusoidal
vascular channels located within the brain.
 symptoms: H/A, seizure, neurological deficit,
ICH
- prevalence: 0.3-0.5%
- hemorrhage risk may be up to 3.6% per year
- diagnosis: MRI is the choice; usually not seen
with 4-vessel angiography
- treatment: surgical excision - depending on
presentation and location (most are
observed)

 NEOPLASM:
 BRAIN:
- Primary vs Metastatic
- Primary tumors: Astrocytomas, GBM,
Meningiomas, Pituitary Adenoma,
Vestibular Schwannoma, Lymphoma,
Craniopharyngioma
- Clinical presentation: increased ICP
due to the size of the tumor as well as
brain edema and obstruction of CSF
flow; Local effects resulting to focal
neurological defects, lobe syndromes,
cognitive decline, seizures and
headaches
UST FMS MEDICAL BOARD REVIEW 2019 6 | SURGERY
NEUROSURGERY
EDGARDO T. TAN, MD
MARIA LOURDES D. MAGLINAO, MD
RICARDO C. ENRILE, MD & GILBERT J. RAÑAO, MD

 most common brain tumour survival


seen clinically
 15% of cancer patients present with
cerebral metastasis
 Source
 Lung 44% (esp.
Bronchogenic CA)
 Breast 10% (Adenocarcinoma)
 Kidney 7% (Renal Cell CA (RCC)
 GI 6% (Colon CA)
 Melanoma 3%
 route of spread – hematogenous
 location – 3/4 are supratentorial, often
at grey-white matter junction
 diagnosis: metastatic work-up (CXR, CT
chest/abdominal, abdominal U/S)
 CT with contrast (round, well-
circumscribed uniformly lesion)
 consider biopsy (as up to 10% may not
be cerebral met in patient with cancer  radiotherapy prolongs survival
history) and patient may not have a (retrospective evidence)
cancer history  chemotherapy (alkylating agents)
 prognosis: median survival with optimal Rx Meningioma:
26-32 weeks but varies depending on  mostly benign (1% malignant), slow-
primary growing, non-infiltrative
 treatment: palliative  common locations: parasagittal convexity,
 single accessible lesion ––> surgical sphenoid wing, falx
excision + radiation  presentation: middle aged, symptoms of
 multiple lesions ––> whole brain increased ICP, focal symptoms depend on
radiation location
 diagnosis: MRI, CT with contrast
 therapy

 conservative
management for slow-growing lesions
 surgery is treatment of choice (curative
if complete resection)
 radiotherapy – ineffective
 prognosis: > 90% 5-yr survival

Vestibular Schwannoma (“Acoustic Neuroma”)


 progressive unilateral deafness = acoustic
neuroma until proven otherwise
 slow-growing (average of 1-10 mm/yr),
benign posterior fossa tumour
 arises from vestibular component of CN VIII
at cerebello-pontine angle (CPA)
 clinical presentation: compression of
structures in CPA
 often CN VIII symptoms, then V, then
VII
 CN VIII: unilateral sensorineural
deafness, tinnitus, dysequilibrium
 CN V: facial numbness, loss of corneal
reflex
- Astrocytoma: most common primary brain  CN VII: facial weakness (uncommon
tumour pre-operatively)
(45%- 50%)  cerebellum: ataxia, nystagmus
 Clinical presentation: middle aged, recent
onset of new, worsening H/A, N/V, +/–  diagnosis
focal deficits or symptoms of increased  MRI, CT (contrast enhancing mass in
ICP CPA)
 diagnosis: CT (see Figure 4), MRI with  audiogram, BAEP (brainstem auditory
contrast evoked potentials), caloric tests
+/– biopsy  if bilateral: neurofibromatosis type II
 therapy:  management
 surgery: not curative, aim to prolong  conservative: serial imaging
“quality”
UST FMS MEDICAL BOARD REVIEW 2019 7 | SURGERY
NEUROSURGERY
EDGARDO T. TAN, MD
MARIA LOURDES D. MAGLINAO, MD
RICARDO C. ENRILE, MD & GILBERT J. RAÑAO, MD

 surgery: several routes, curable if


complete resection (almost always
possible)
 stereotactic radiosurgery: gamma-
knife, linear accelerator
 significant post-therapy

• Extradural Tumors
• Intradural–Extra-
morbidity: CN VII, VIII medullary Tumors
dysfunction (only significant • Intradural–Intra-
disability if bilateral), CSF leak
medullary Tumors
- Pituitary Adenomas
 primarily from anterior pituitary, 3rd-4th
decade, M=F
 may be functional (secretory) or non- - CNS Lymphoma
functional  comprising 0.85% - 2% of all primary brain
 clinical presentation tumors and 0.2% - 2% of malignant
a) mass effects lymphomas; maybe increasing in incidence
 H/A due to AIDS and
 bitemporal hemianopsia (compression transplant patients. Maybe primary
of optic chiasm) (B lymphocytes) or secondary
 CN III, IV, V1, V2, VI palsy  Median age at diagnosis is 52 years
(compression of cavernous sinus) (younger among immune -compromised
b) endocrine effects patients)
 hyperprolactinemia ––> infertility,  Increase risk: Collagen vascular disease,
amenorrhea, galactorrhea, impotence immunosuppression, Epstein-Barr virus
 ACTH production ––> Cushing‟s  Clinical presentation: mental changes,
disease increased ICP, seizures (9%) CNs palsies,
 GH production ––> acromegaly FND, or combination
 panhypopituitarism (hypothyroidism,  Most common supratentorial locations –
hypoadrenalism, hypogonadism) frontal lobes, then deep nuclei;
c) apoplexy (abrupt onset H/A, periventricular also common;
visual disturbances, Infratentorially cerebellum is the most
ophthalmoplegia, and reduced common location; Diagnosis by MRI scan
mental status) and CSF with contrast
rhinorrhea (rare presenting  Treatment: Very responsive initially to
signs of pituitary tumour) steroids (may produce “ghost tumors”). Tx
 diagnosis: formal visual fields, usually XRT +/- chemotherapy. Role of
endocrine tests (PRL level, TSH,
cortisol, fasting glucose, FSH/LH, IGF-
1), imaging (MRI)
 differential: parasellar tumours (e.g.
craniopharyngioma, tuberculum sellae
meningioma), carotid aneurysm
 treatment: medical
 dopamine agonists (e.g.
bromocriptine) for prolactinoma
 serotonin antagonist
(cyproheptadine), neurosurgery usually limited to biopsy
inhibition of cortisol and/or placement of ventricular access
production reservoir for chemotherapy
(ketoconazole) for  Prognosis: with no treatment, median
Cushing‟s survival is 1.8 - 3.3 months following
 somatostatin analogue diagnosis; with radiotherapy median
(octreotide) +/– bromocriptine survival is 10 months with 5-year survival
for acromegaly of 3% - 4%; 78% will recur usually 15
 endocrine replacement therapy months after treatment. In AIDS related
 surgical: trans-sphenoidal, cases, the prognosis appears worse
transethmoidal, transcranial - Craniopharyngioma
approaches  Incidence: 2.5-4% of all brain tumors
 Age of predilection: 50% occur in childhood,
peak incidence age 5-10 years; 2nd peak
 Location: occur in the region of pituitary
fossa extending to suprasellar cisterns &
hypothalamus
 Cardinal clinical symptoms:
- raised ICP
- visual impairment
- endocrine dysfunction around 50-years-
old.

UST FMS MEDICAL BOARD REVIEW 2019 8 | SURGERY


NEUROSURGERY
EDGARDO T. TAN, MD
MARIA LOURDES D. MAGLINAO, MD
RICARDO C. ENRILE, MD & GILBERT J. RAÑAO, MD

 Diagnosis: plain x-ray Calcification 85% in  Diagnosis: MRI scan with contrast, CT-
childhood, 40% in adults, CT, MRI scan with Myelography
contrast  Treatment:
 Histology: arise from epithelial remnants Surgery: Laminectomy for excision of SCT;
of Rathke‟s pouch; majority cystic & urgent
fluid is yellow with cholesterol crystal; - Indications for surgery: Cord
calcification 50% compression
With progressive neurological deficit and
Sphincter disturbance
High dose of IV Dexamethasone
(Corticosteroid)
Radiation therapy
 Extradural Tumors: lymphoma, metastases
from prostate, lung, breast, kidney
 Intradural – Extramedullary Tumors:
schwannoma, meningioma, neurofibroma
 Intradural – Intramedullary Tumors:
ependymoma, astrocytoma,
hemagioblastoma

 CNS INFECTIONS
 Brain Abscess:
 Treatment: Sx, XRT  Etiology
 Prognosis: 5-10% mortality; 5 yr survival  local spread (adjacent infection)
is 55-85% otitis media, mastoiditis, sinusitis, dental
 SPINAL CORD
abscess, osteomyelitis
 Clinical presentations:  hematogenous spread
- Pain – most common complaint  adults: lung abscess, bronchiectasis,
Local, Radicular or „Central‟ pain;
Lhermitte‟s empyema
sign  children: cyanotic heart disease with R
- Neurologic dysfunction distal to the cord
compression (progressive) to L shunt (blood is shunted away
- Sensory dysesthesia (with dermatome from lungs preventing filtration of
level) bacteria)
and muscular weakness (2nd or 3rd
complaint)  immunosuppression (AIDS -
- Sphincter dysfunction toxoplasmosis)
# Radiculopathy
- Motor: weakness, wasting, decreased
deep
tendon reflex in root distribution
- Sensory: dermatomal decreased
pinprick
sensation, numbness, paresthesiae,
pain
- Trophic changes: eg. dry skin (if long-
standing radiculopathy)
# Myelopathy
- LMN signs/symptoms at level of lesion
- UMN signs/symptoms below lesion
 motor: proximal weakness and
spasticity of lower extremities,
increased reflexes, clonus,
Babinski sign (extenser plantar
response), sphincter disturbance  dural disruption: surgery, trauma,
 sensory: findings may be minimal congenital defect, e.g. dermal sinus
(reduced vibration,
proprioception), +/– Lhermitte  pathogens
sign - Streptococci (most common),
often
anaerobic or microaerophillic
- Staphylococci (penetrating injury)
- Gram negatives, anaerobes
- Toxoplasmosis and Nocardia in
immunocompromised hosts
 Diagnosis:
- focal neurological signs and
symptoms
- mass effect, increased ICP and
sequelae
- seizures

UST FMS MEDICAL BOARD REVIEW 2019 9 | SURGERY


NEUROSURGERY
EDGARDO T. TAN, MD
MARIA LOURDES D. MAGLINAO, MD
RICARDO C. ENRILE, MD & GILBERT J. RAÑAO, MD

- +/– signs of systemic infection (mild - Fever, weight loss


fever, - Duration of symptoms at
leukocytosis) diagnosis is 4 months or longer
- blood cultures rarely helpful, LP not - Chronic back pains (spinal or
helpful and contraindicated radicular pain), gibbus, impaired
- CT scan with contrast sensation, nerve root pain,
neurological deficit (paraplegia) in
50% of cases
 Physical examination
- Careful assessment of spinal alignment
(kyphosis)

- Inspection of skin, with attention to detection


of sinuses (cold abscess/tract)
- Abdominal evaluation for subcutaneous flank
mass (psoas muscle)
- Meticulous neurologic examination

 Management:  Diagnosis: Plain spine X-rays, CT/MRI scan


- multiple aspiration of abscess and/or of spine, positive tuberculin skin test
excision, and send for C&S (purified protein derivative)‟ presence of
- excision is preferable if location suitable risk factors for tuberculosis; ESR;
- antibiotics microbiologic study (AFB) to confirm the
- empirically: vancomycin plus diagnosis
ceftazidime plus metronidazole or  Treatment: Usually medical (Quadruple
chloramphenicol or rifampin therapy; Isoniazid & Rifampicin should be
- after sensitivity results return, revise administered during the whole course of
antibiotics therapy for 9-12 months; additional drugs
- anti-convulsants x 1-2 years such as pyrazinamide, ethambutol, &
 Pott’s Disease streptomycin are given during the first 2
 A very common spine disease in the months of therapy).
Philippines Second line drugs used for drug resistance.
 Can affect any age & both sexes
 Usually secondary to a primary
focus and spread by direct
extension or hematogenous spread
 Anterior aspect of vertebral body
adjacent to the subchondral plate is
usually affected
 Thoracic vertebrae are affected
most (40-50%) especially the lower
region; lumbar spine (35-45%),
cervical spine (10%)
 Presentation depends on the
following:
- stage of disease  DEGENERATIVE SPINE:
- affected site  CERVICAL DISC SYNDROME
- presence of complications such  Etiology:
as neurologic deficits, abscesses, or - most common levels - C5-C6 (C6 root) more
sinus tracts common vs. C6-C7 (C7 root)
 Presentation - less common, but important with respect to

UST FMS MEDICAL BOARD REVIEW 2019 10 | SURGERY


NEUROSURGERY
EDGARDO T. TAN, MD
MARIA LOURDES D. MAGLINAO, MD
RICARDO C. ENRILE, MD & GILBERT J. RAÑAO, MD

activities of daily living - C4-5 (C5 root), C7-  Clinical Features


T1 (C8 root) - leg pain > back pain
 Clinical Features - limited back movement (esp. forward
- lateral disc protrusion compresses nerveroot flexion)
 pain down arm in nerve root - symptoms and signs of radiculopathy
distribution, worse with neck extension  pain in root distribution (worse with
 referred parascapular pain movement, valsalva)
 +/– nerve conduction velocity  dermatomal sensory deficit
abnormalities  LMN weakness
- central cervical disc protrusion compresses  reduced deep tendon reflex
spinal cord as well as nerve roots  +/– reflex paravertebral muscle
 Differential Diagnosis spasm (functional scoliosis, loss of lordosis)
 shoulder lesion - nerve root tension signs: straight leg raises
 thoracic outlet syndrome (including (SLR: Lasegue‟s test), crossed SLR ––>
Pancoast tumour) L5, S1 roots; femoral stretch ––> L4
 cervical spine tumour root
 peripheral nerve lesion (e.g. carpal  Differential Diagnosis
tunnel) - spinal: stenosis, tumour, spondylolisthesis
 acute brachial neuritis - leg: spinal stenosis, arthritic hip, sciatic
 myocardial infarction (MI) (left C6 nerve lesion (e.g. tumour)
radiculopathy) - pelvic bones: tumour
 Investigations: C-spine x-ray, CT, MRI - functional /nonorganic
(procedure of choice), EMG-NCV studies  Investigations: X-ray spine, CT, MRI,
 Management myelogram and post-myelogram CT (if surgery
 conservative (recovery in 95%): contemplated and plain CT not conclusive)
NSAIDs, collar, traction may help; most  Management
patients get better spontaneously in 4 to - conservative
8 weeks  no bedrest unless severe radicular
 surgical indications symptoms
o intractable pain despite adequate  activity modification (reduce
conservative treatment for > 3 sitting, lifting)
months  physiotherapy (PT), exercise
programs
o progressive neurological deficit  analgesics (acetominophen,
o anterior cervical discectomy is usual NSAIDs)
surgical choice  patient education
 LUMBAR DISC SYNDROME  95% improve spontaneously
 Etiology: within 4 to 8 weeks
- protrusion/herniation of nucleus pulposus: - surgical indications
laterally: compressing nerve root  intractable leg pain despite
centrally: compressing cauda equina adequate conservative treatment
- common: (>95% of herniated lumbar for > 3 months
disks):  disabling neurological deficit
L4-5 (L5 root), L5-S1 (S1 root) ❏  progressive neurological deficit
uncommon:  cauda equina syndrome
L3-4 (L4 root)
 CAUDA EQUINA SYNDROME
 Etiology
- secondary to compression of lumbosacral
nerve roots below conus medullaris
- extrinsic tumour, carcinomatous meningitis,
arachnoiditis, spinal stenosis, central
posterior lumbar disc herniation
 Clinical Features
- motor
 weakness/paraparesis in multiple root
distribution
 reduced reflexes (usually achilles
reflex)

UST FMS MEDICAL BOARD REVIEW 2019 11 | SURGERY


NEUROSURGERY
EDGARDO T. TAN, MD
MARIA LOURDES D. MAGLINAO, MD
RICARDO C. ENRILE, MD & GILBERT J. RAÑAO, MD

 sphincter disturbace (urinary retention  CLINICAL MANIFESTATIONS:


which can lead to overflow - Abnormal head enlargement
incontinence, and fecal incontinence - “Sun setting” eye phenomenon
due to loss of anal sphincter tone) - Headache, irritability, nausea/vomiting,
- sensory drowsiness
 multiple dermatome, bilateral sensory - Prominent scalp veins
loss or pain - Skull bones separated
 saddle anesthesia (most common
- Macewen sign
sensory deficit)
- Tense, bulging fontanel
- other
- CNs VI & III palsy
 sexual dysfunction (late finding)
- Irregular respirations with apneic spells
 Management: requires urgent investigation and
decompression to preserve bowel and bladder - Alteration in development of intelligence
function
 LUMBAR SPINAL STENOSIS
 Etiology: congenital narrowing of spinal canal
combined with degenerative changes
(herniated disk, hypertrophied facet joints and
ligaments)
 Clinical presentation
 neurogenic claudication
(“pseudoclaudication”)
 neurologic exam may be normal,
including straight leg raise test
 symptoms relieved only by changing
position (leaning forward, sitting down)
 Investigations: MRI, CT  CLASSIFICATION
 Treatment - Obstructive (Non-Communicating)
 conservative (NSAIDS, PT) Hydrocephalus
 surgical (laminectomy with root - absorption is blocked within ventricular
decompression) system
proximal to the arachnoid granulations
 CONGENITAL: - causes/location of block:
 intraventricular hemorrhage
 ventricular tumours (e.g. 3rd
ventricle colloid cyst)
 supratentorial mass causing tentorial
herniation and aqueduct compression
 infratentorial mass causing 4th
ventricle or aqueduct obstruction
 congenital e.g. aqueductal stenosis,
Dandy-Walker malformation, or
Chiari malformation
- CT findings
 lateral and 3rd ventricles dilated
 normal 4th ventricle (e.g. aqueduct
 Hydrocephalus: stenosis) or deviated/absent 4th
 DEFINITION: increased CSF volume, decreased ventricle (e.g. posterior fossa mass)
CSF absorption, normal CSF volume = 100 - - Communicating (Non-Obstructive)
150 mL (50 in ventricles, 25 around brain, 75 Hydrocephalus
around spinal cord); CSF production is constant
- absorption is blocked at some part of
at 0.4-0.6 cc/hr
extraventricular pathway, such as
 MECHANISMS
arachnoid granulations.
- increased CSF production: e.g. choroid
- causes: meningitis, SAH, trauma
plexus papilloma (0.4-1% of intracranial
- CT findings: all ventricles dilated
tumors)
 MANAGEMENT
- Increased in venous sinus pressure –
- spinal taps (for transient, communicating
venous sinus thrombosis
hydrocephalus)
- obstruction to CSF flow
- remove obstruction (if possible)
UST FMS MEDICAL BOARD REVIEW 2019 12 | SURGERY
NEUROSURGERY
EDGARDO T. TAN, MD
MARIA LOURDES D. MAGLINAO, MD
RICARDO C. ENRILE, MD & GILBERT J. RAÑAO, MD

- excision of choroid plexus papilloma - Syndromic (Crouzon syndrome, Apert


- third ventriculostomy (for obstructive syndrome, Pfeiffer syndrome) or Non-
hydrocephalus) Syndromic.
- shunts:
 ventriculoperitoneal (VP) = ventricle to
peritoneum
 ventriculopleural = ventricle to pleura
 ventriculo-atrial (VA) = ventricle to
right atrium
 lumboperitoneal = lumbar spine to
peritoneum (for communicating
hydrocephalus and pseudotumour
cerebri)

 Shunt Complications

obstruction
• etiology: infection, obstruction by choroid
plexus, buildup of proteinaceous accretions,
blood, cells (inflammatory or tumour)
• signs and symptoms of acute hydrocephalus
or increased ICP
• radiographic evaluation: “shunt series” (plain
x-rays which only show disconnection of tube
system), CT, isotope shunt study (nuclear
medicine)

- infection (3-4%)
• etiology: S. epidermidis, S. aureus, gram-
negative bacilli
• presentation: fever, nausea and vomiting,
anorexia, irritability; signs and symptoms of
shunt obstruction; shunt nephritis (antibodies
generated against bacteria in shunt leads to
kidney damage)
• investigation: CBC, blood culture, shunt tap
(lumbar puncture (LP) usually NOT
recommended in obstructive hydrocephalus)

- overshunting
• slit ventricle syndrome (collapse of
ventricles leading to shunt catheter
occlusion by ependymal lining)
• subdural effusion, hygroma, hematoma
• secondary craniosynostosis (children) Indications for Surgery:
• low pressure headache  Signs of elevated ICP
- seizures  Rapid abnormal growth of skull
Craniosynostosis  Age vs clinical situation
• Craniosynostosis - premature fusion of one or  Availability of craniofacial team
more sutures of a child's skull with resultant - Best time to do surgery is in the late
restriction of skull growth in the affected area and infancy, i.e.
compensatory bulging at the other sutures. between 6-12 mos.
• Skull growth at the cranial sutures for the first 2  less risk of blood loss
years of life, at the end of which the skull has  bone more malleable & can be
achieved >90% of its eventual adult size remodeled
- Primary Craniosynostosis or Secondary  Reshaping & replacement of bone gaps
Craniosynostosis readily re-ossified before the age of
one year, but need refilling thereafter

UST FMS MEDICAL BOARD REVIEW 2019 13 | SURGERY


NEUROSURGERY
EDGARDO T. TAN, MD
MARIA LOURDES D. MAGLINAO, MD
RICARDO C. ENRILE, MD & GILBERT J. RAÑAO, MD

 Spinal Dysraphism: Spinal dysraphism refers Chiari Malformation


to incomplete midline closure of the neural
tube or one or more vertebral arches. ~
“Neural Tube Defect”
 A deficit in primary neurulation leads to
anencephaly or myelomeningocele, which fall
into the category of open spinal dysraphism;
failure in secondary neurulation or disjunction
leads to closed spinal dysraphic anomalies.
(Tethered
Cord Syndrome)
 Process of neurulation occurs during days 18-
 Chiari I malformation : Cerebellar tonsils
28 of gestation (Primary Neurulation)
descent into the foramen magnum (5mm).
 Closure of the rostral neuropore occurs during
Syringomyelia commonly associated & may
gestational days 22-24. Failure at this point
coexist in 60–70% of patients, resulting in
results in anencephaly.
progressive scoliosis in 30% of these patients.
 Closure of the caudal neuropore occurs during
 Chiari II malformation : herniation of the
days 24-26; disruption during this period
tonsils, vermis, medulla, and fourth ventricle
causes myelomeningocele.
into the spinal canal is present. Chiari II
 Beyond day 27, a disruption is unlikely to cause
malformation occurs in >90% of patients with
an NTD such as myelomeningocele.
myelomeningocele.
 Folate deficiency is associated with 70% of
 Chiari III malformation : Suboccipital
myelomeningocele; Folic Acid: taken 1 month
encephalocele w/ herniation of all brainstem
before conception and continuing through the
structures into the foramen magnum
first trimester  Chiari IV malformation : Cerebellar hypoplasia
or aplasia
 For prophylaxis: 400 ug (0.4 mg)/day
 Neurological signs: presence of progressive
 For those with family history or a member who
neurological deficits is an indication for surgery
has history of Spina Bifida: 4000 ug (4.0  Management:
mg)/day
- Surgical decompression of foramen magnum
 Surgery: Repair or closure with or without VPS;
and of cervical canal if treatment required
“Operative care aims to preserve all functional
- Confirm stability of craniocervical junction
neural tissue, to reconstitute the normal
- Insure shunt for hydrocephalus is functional
barriers and thus minimize retrograde
infections, and to control early progressive
 Tethered Cord Syndrome:
hydrocephalus”
 Abnormally low conus medullaris associated
 Key concepts:
with a short, thickened filum terminale
- Critical goals: free placode from dura, water-
 The spinal cord is fastened to an immovable
tight
structure, such as lipoma, vertebra, dura or
dural closure, skin closure
skin
- Timing goal: surgical closure with latex-free
 Clinical manifestations:
setup
 Leg weakness and atrophy
ideally </= 36 hrs after birth
 Gait disturbance
- Post-op CSF leak usually means a shunt is
 Sensory deficit
Required
 Neurogenic bowel/bladder
 Hyporeflexia
 Diagnosis: MRI scan, Ultrasonography, Sensory
Evoked Potentials, EMG NCV, Urodynamics
 Surgical indications:
 The indications for surgery are based
upon an understanding that the
specific lesion and tethered spinal cord
are producing symptomatology and
neurologic changes which is
unacceptable, and that the operation
has the potential to produce relief of
symptoms and neurologic improvement
 The surgery will interrupt the
progression of the signs and symptoms

UST FMS MEDICAL BOARD REVIEW 2019 14 | SURGERY


NEUROSURGERY
EDGARDO T. TAN, MD
MARIA LOURDES D. MAGLINAO, MD
RICARDO C. ENRILE, MD & GILBERT J. RAÑAO, MD

 PERIPHERAL NERVE: - Symptoms may be intermittent, are


usually worst at night, and may be
relieved by shaking the hand while
holding it in a dependent position
- Symptoms are often provoked by wrist
flexion

 Tumor:
 Most peripheral nerve tumors are benign
and slow growing
 Significant pain increases the likelihood of
a malignant tumor
 Diagnosis: History, PE, EMG-NCV, MRI
scan with contrast
 Most treatment is surgical resection in - Signs of wasting thenar eminence & weakness
order to establish the diagnosis and to of the abductor pollicis brevis and diminished or
evaluate for signs of malignant
altered sensation in the median nerve
degeneration
distribution
 Surgery: Total or subtotal resection, nerve
- (+) Tinel‟s sign, Phalen‟s test
sacrifice or preservation – will depend on
- Diagnosis by EMG-NCV
tumor histology and the function of the
parent nerve - Treatment maybe conservative, but if with
atrophy & severe weakness then the transverse
 Entrapment Neuropathy: carpal ligament can be divided
 A peripheral nerve injury resulting from
compression either by external forces or  Brachial Plexus
from nearby anatomic structures
 Certain nerves are vulnerable at specific
locations by virtue of being superficial,
fixed in position, traversing a confined
space, or in proximity to a joint
 Most common symptom is pain (frequently
at rest, more severe at night, often with
retrograde radiation causing more
proximal lesion to be suspected) with
tenderness at the point of entrapment;
paresthesia, loss of function
 Treatment of Nerve Entrapment
 Conservative Treatment: non-steroidal
anti-inflammatory drugs or splint
 Surgical Treatment:
 Upper Trunk Lesions (Erb-Duchenne
- Indications:
paralysis) – C5-6
 Failed conservative treatment
- (gleno-humeral dislocation) “bellhop‟s
 Typical clinical finding with electro-
tip” or “waiter‟s tip” position; affects deltoid,
diagnostic data
biceps, rhomboids, brachioradialis,
 Severe sensory loss, muscle
supraspinatus and infraspinatus
atrophy, weakness
- arm hanging to side, internally rotated
and extended at the elbow
 Carpal Tunnel Syndrome:
 Lower Trunk Lesions (Klumpke‟s
- Pain & numbness in the distribution of the
paralysis) – C8-T1 resulting to “claw
median nerve in the hand
hand” deformity with weakness of the
- More common in patients with diabetes,
intrinsic hand muscles
hypothyroidism, acromegaly and pregnancy

UST FMS MEDICAL BOARD REVIEW 2019 15 | SURGERY


NEUROSURGERY
EDGARDO T. TAN, MD
MARIA LOURDES D. MAGLINAO, MD
RICARDO C. ENRILE, MD & GILBERT J. RAÑAO, MD

 Thoracic Outlet Syndrome  Other ablative techniques include Dorsal


 Caused by bony, fascial, and muscular root entry zone lesions for brachial plexus
structures that interfere with the injury with deafferentation pain, Myelotomy
neurovascular bundle (Brachial plexus) used in the treatment of bilateral cancer
 A fibrous band with the scalenius anterior pain, Cordotomy in the treatment of
muscle, a cervical rib, or its remnant may unilateral cancer pain, Sympathectomy
result in angulation or compression of the primarily used in patients with hyperhidrosis,
lower trunk of the brachial plexus or C8/T1 sympathetically mediated pain, causalgia,
roots and subclavian vessels chronic regional pain syndrome, and
 Diagnosis: Raynaud disease, and Nerve block or
- X-ray (look for osseous abnormalities neurectomy usually reserved for short-term
EMG-NCV relief in patients with a poor prognosis and
- 3D CT scan, MRI, cervical myelography short life
(r/o foraminal narrowing or disc expectancy.
compression)
- Angiography (vascular anomaly or  Epilepsy Surgery:
narrowing compression)  Failure to respond to three anticonvulsant
- Exploration (stenosis or aneurysms….) medications prompt referral to a center
 Treatment: Conservative or surgical specializing in epilepsy evaluation &
treatment, and approximately 1.33% to
 FUNCTIONAL NEUROSURGERY 4.50% of those patients are candidates for
 Functional neurosurgery is concerned surgical intervention
with the anatomic or physiologic  The goal of the workup of the patient who is
alteration of the nervous system to a potential candidate for the surgical
achieve a desired effect. treatment of epilepsy is to identify the
 This can be done with focal electrical cortical area responsible for the onset
stimulation procedures, ablative of the seizure; the treatment is
procedures, or implantation of pumps to relatively straightforward and involves
deliver drugs, usually to the CSF but removal of the lesion.
possibly also to the parenchyma.  Other surgical techniques used in epilepsy
 It deals primarily with the treatment of surgery is anterior temporal lobectomy, focal
pain, movement disorders, epilepsy, and cortical resection, multiple subpial
some psychiatric disorders when they transection, hemispherectomy, and corpus
are refractory to conventional callosotomy.
treatments. These disorders all have in
common hyperfunction or deranged
function of some part of the CNS.

 Deep Brain Stimulation – Parkinson


Disease
 Electrical stimulation of the nervous system
is used in the treatment of movement
disorders esp. in Parkinson disease, the
most common movement disorder for which
patients have surgery
 Stereotactic stimulation of the Subthalamic
nucleus is most effective for the treatment
of rigidity and akinesia in Parkinson patients
who had become unresponsive to  Trigeminal Neuralgia characterized by
pharmacologic agents or intolerant of their brief episodes of severe, lancinating pain in
side effects. one or more of the three divisions of the
 Other stimulation procedures include Spinal trigeminal nerve, usually V2 and V3
cord stimulation used for the treatment of  Most patients respond to the oral
chronic pain, dystonia, and bladder administration of carbamazepine, Baclofen
dysfunction and vagal nerve stimulation for and Gabapentin
the treatment of intractable seizures and  The most common mechanism is presumed
severe depression. to be related to vascular compression of the
 Ablative lesioning of the CNS for the fifth cranial nerve as it enters the brainstem.
treatment of pain, movement disorders,  Surgical therapy is usually reserved for
epilepsy, and psychiatric diseases. patients who fail to respond to medical
UST FMS MEDICAL BOARD REVIEW 2019 16 | SURGERY
NEUROSURGERY
EDGARDO T. TAN, MD
MARIA LOURDES D. MAGLINAO, MD
RICARDO C. ENRILE, MD & GILBERT J. RAÑAO, MD

treatment; Microvascular decompression


involves a small suboccipital craniotomy for
microsurgical exploration of the dorsal root
entry zone of the trigeminal nerve on the
affected side.
 The offending vessel, usually the superior
cerebellar artery, is then dissected off the
nerve, and a barrier (Teflon or polyvinyl
alcohol sponge) is placed between the
vessel and nerve to prevent continued
pulsatile focal compression.

 Percutaneous trigeminal rhizotomy


techniques generally involve radiofrequency
heat lesioning of the trigeminal ganglion,
glycerol injection (Fig. 67-24) into the spinal
fluid of Meckel cave, as well as SRS has
been described for the treatment of
trigeminal neuralgia.

 Stereotactic Radiosurgery (SRS)


 The term stereotactic radiosurgery (SRS)
refers to techniques that allow delivery of
high-dose radiation that conforms to the
shape of the target and has rapid isodose
fall-off, minimizing damage to adjacent
neural structures.
 The two most common devices used for
conformal SRS for intracranial lesions are
the LINAC (linear accelerator) and the
gamma knife.
 Other type of SRS include Proton Beam
Radiation and Cyberknife.
 Diseases treated with these technologies
include AVM, Vestibular Schwannomas and
intracranial metastasis.
 SRS is not effective for lesions >3 cm in
AVM and <2.5 cm in Vestibular
Schwannoma. Effective obliteration and
elimination of the risk of hemorrhage in AVM
takes 2 to 3 years.

UST FMS MEDICAL BOARD REVIEW 2019 17 | SURGERY


REVIEW TEST
GILBERT RAÑOA, MD

NEUROSURGERY REVIEW TEST

CHOOSE THE BEST ANSWER:

_____ 1. A patient who after a head trauma _____ 7. In patients who are immunocompromised
demonstrate the following signs and symptoms or with HIV disease manifest low to moderate grade
(Hypertension, Bradycardia, Respiratory irregularity) fever, headache, vomiting and drowsiness. A cranial
is said to have this: CT scan with contrast revealed several 0.5 to 1 cm
A) Cushing Disease hypodense masses and some with concentric target
C) Cushing Syndrome sign, located in the cortical white matter and basal
B) Cushing’s Phenomenon or sign ganglia areas, is said to be harboring this infection:
D) Triad of head injury A) Toxoplasmosis
C) Staphylococcus brain abscesses
_____ 2. A patient was involved in a vehicular B) Gram negative/anaerobic microorganism
accident and when brought to the hospital is found D) CNS lymphomas
to be drowsy and responds to painful stimulus by
partial eye opening, moaning and in bilateral _____ 8. The most common levels of cervical disc
abnormal flexion of both UEs is said to have this syndrome are at this level:
Glasgow Coma Scale (GCS) score: A) C4-5 & C5-6
A) 5 C) C6-7 & C7-T1
C) 6 B) C5-6 & C6-7
B) 7 D) C7-T1 & T1-T2
D) 8
_____ 9. A woman who recently got married would
_____ 3. All of the following can be given in a like to receive a prophylactic dose of folic acid for
traumatic brain injured patient who manifested signs fear of developing a spina bifida baby. You would
and symptoms of headache, nausea, vomiting and recommend this dose:
beginning anisocoria EXCEPT: A) 4000 ug/day
A) Mannitol 20% C) 4.0 mg/day
C) Hypertonic Saline Solution B) 400 ug/day
B) Hyperventilation D) 0.04 mg/day
D) Corticosteroids
_____ 10. A 40 y/o pregnant woman with a history
_____ 4. A 40 y/o patient with a sudden onset of of hypothyroidism and diabetes complained of pins
severe headache, nausea and vomiting with nuchal and needles sensations in her both hands especially
rigidity, right ptosis and dilated pupil is said to be in at night. Said symptoms can be relieved by shaking
this Hunt and Hess Grading Scale: the hand while holding it in a dependent position.
A) Grade I She also has wasting of her both thenar eminence
C) Grade II and weakness of the abductor pollicis brevis. She
B) Grade III must be suffering from this disease:
D) Grade IV A) Upper Trunk Brachial Plexus Injury
C) Carpal Tunnel Syndrome
_____ 5. The most common site of Hypertensive B) Thoracic Outlet Syndrome
Intracerebral Hemorrhage is in this site: D) Radial Nerve Entrapment Neuropathy
A) Putamen
C) Cerebellum
B) Thalamus
D) Subcortical white matter

_____ 6. Complete excision of this tumor is said to


be least likely to recur:
A) Glioblastoma Multiforme
C) Gliosarcoma
B) Anaplastic astrocytoma
D) Juvenile Pilocytic astrocytoma

UST FMS MEDICAL BOARD REVIEW 2019 | SURGERY


PEDIATRIC SURGERY
HERMOGENES R. REGAL, MD

PEDIATRIC SURGERY sided duodenal junction on UGIS, while a smooth


blind pouch points to Duodenal Atresia; On Barium
INGUINO-SCROTAL ABNORMALITIES enema, a high-lying cecum in the right upper
quadrant or epigastric area is compatible with
Hernia and Hydrocoele Malrotation; Ultrasound studies can also identify
Malrotation with abnormal orientation of the
> Hernia and Hydrocoele have the same Superior Mesenteric vessels.
pathophysiology; patent processus vaginalis testis
> The pathology in Malrotation is the non-rotation of
> Hernia manifests as dramatic appearance of a the duodena-jejunal junction under the Superior
bulge at the inguinal area (sometimes including Mesenteric vessels and non-rotation of the ileo-cecal
enlargement of the scrotum) during straining junction over the Superior Mesenteric vessels
(crying, playing) and disappearance at rest. resulting in both junctions lying side-by-side in the
Hydrocoele, on the other hand, manifests as scrotal mid-epigastric area with a narrow mesenteric pedicle
enlargement with minimal change in size. Likewise, and not fixed retroperitoneally. This anatomy
Hydrocoele does not present with an inguinal bulge. predisposes to Midgut Torsion or Volvulus, and
together with the presence of “Ladd’s Bands”
> Hernia and Hydrocoele, having the same crossing over the 2nd portion of the duodenum,
pathophysiology, are treated similarly surgically; cause the obstruction at the level of the duodenum.
High ligation of the patent processes. An exception
is a Hydrocoele in an infant up to the first year of > That being the pathology, surgical correction
life because there’s a high chance of spontaneous entails detorsion of the midgut volvulus, lysis of the
resolution. Ladd’s Bands, adhesiolysis to widen the narrow
mesenteric pedicle thereby bringing the cecum to
Undescended and Retractile Testes the opposite (left) side of the abdomen (“Ladd’s
Procedure”).
> Absence of testis in the scrotal sac maybe
interpreted as Undescended or Retractile. In > In Duodenal Atresia, there’s discontinuity of the
either condition, the testis maybe palpated along the bowel at that level, hence, the ideal treatment is to
inguinal area. The presence of anastomose the proximal and distal ends of the
testis in the scrotum at birth is compatible with a atretic bowels.
Retractile Testis.
Hirschsprung’s Disease and Intestinal Atresias
> In either situation, if the testis, having been
appreciated in the inguinal area, can be manipulated > These two are the common conditions that
and brought down to the scrotal sac without undue manifest as Low Intestinal Obstruction (bilious
tension, it is a Retractile Testis. If it cannot be vomiting with frank abdominal distention) in the
brought down to the scrotal sac, it is an neonatal period. If extended into infancy,
Undescended Testis. Strangulated Inguinal Hernia should be considered.

> Differentiation is important between the two > The pathology of Hirschsprung’s Disease is
because treatment would be different. Retractile absence of ganglion cells in the recto sigmoid wall
Testis does not need any surgical intervention. layers but may extend proximally involving the
However, Undescended Testis has to be surgically entire colon and even the small bowel. Contrast
brought down and fixed inside the scrotal sac study (Barium enema) will demonstrate the
(Orchidopexy) ideally not later than 2 years of age, contracted aganglionic segment, and the dilated
the reason mainly, for fertility. proximal ganglionic segment with a Transition Zone
in between.
Testicular Torsion
> Testicular Torsion is an emergency situation > Small Intestinal Atresias, unlike Duodenal Atresia,
because continued ischemia of the testis secondary present with variations, from a simple membrane
to torsion results in poor fertility. Ideal detorsion inside the bowel lumen, to a frank discontinuity
should be done within 6-8 hours. It presents including defect in the mesentery, to severe
suddenly with testicular pain, scrotal enlargement malformations of both bowels and mesentery
with edema and erythema. (“Christmas Tree Deformity”, “Apple-Core
Deformity”, “Multiple Atresias”). This occurs more
GASTROINTESTINAL OBSTRUCTION commonly in the Small Bowel but may also affect
the Colon. Bowels distal to the atretic segment are
Malrotation and Duodenal Atresia unused, thereby, presenting as small-calibered
bowel (“Microcolon seen in Barium Enema”).
> These are the two common conditions presenting
as Upper Intestinal Obstruction (bilious vomiting and >Treatment for Hirschsprung’s Disease is Endorectal
minimal abdominal distention) in the neonatal Pull through (Transanal, or with Abdominal
period. Both will have radiologic appearance of a component) with or without preliminary creation of
“Double Bubble Sign”. Contrast studies will stoma. Atresias may be reconstructed primarily or -
differentiate one from the other: Malrotation will may be done in stages with preliminary stoma
demonstrate “Corkscrew Sign, Beak Sign, and right- creation.
UST FMS MEDICAL BOARD REVIEW 2019 1 | SURGERY
PEDIATRIC SURGERY
HERMOGENES R. REGAL, MD

High Imperforate anus would necessitate an initial


Instussusception colostomy (preferably Sigmoid) and a more complex
construction of a new anal opening a few months
> An infant presenting with “bloody mucoid stool” later (PSARP - Posterior Sagittal Anorectoplasty).
accompanied with signs of intestinal obstruction
(vomiting, irritability or “in pain”, and later ABDOMINAL WALL DEFECTS
abdominal distention) should be considered to have
Intussusception, unless proved otherwise. Hence, Omphalocoele and Gastroschisis
workups should be geared towards obstruction and
not on infectious conditions (“tissue-invasive > Omphalocoele is a central defect on the
gastroenteritis”). Contrast studies (Barium Enema) abdominal wall with an amniotic sac covering it and
may be used to document the Intussusception attached to the Umbilical cord. The defect is large
(“Coiled-Spring Appearance”). Ultrasound may also (>4 cm.) and may contain eviscerated bowels,
be used to demonstrate the “Target Sign” or “Donut stomach, and part of the liver or spleen. There are
Sign”. associated congenital anomalies and syndromes
such as Beckwith-Wiedemann, WAGR, Downs, and
> Treatment, if the condition is recognised early and cardiac anomalies, explaining the poorer prognosis
there is no contraindication (signs of bowel of Omphalocoele compared to its counterpart,
perforation and peritonitis), is preferably non- Gastroschisis.
operative. Saline or air may be used to reduce the
Intussusception and monitored by either > Gastroschisis is a defect on the abdominal wall, to
Fluoroscopy for Pneumatic Reduction and Ultrasound the right of the umbilical cord, smaller in diameter (4
for Saline Reduction. Failed non-operative reduction cm or less) and does not have a covering. Only the
ends up with manual reduction intra-operatively. small intestines eviscerate and are edematous and
erythematous. Associated anomalies are intestinal
Hypertrophic Pyloric Stenosis atresias.

> Manifestation is persistent non-bilious vomiting > Due to the abdominal defect, the neonate is prone
starting usually at the 2nd week of life. A to losing fluid (Dehydration) and body heat
pathognomonic physical examination finding is a (Hypothermia). Aside from these two, sepsis is
palpable “Olive-shaped mass” at the mid-epigastric another cause of morbidity and mortality. Fluid
area. Scout film of the abdomen will reveal a resuscitation often requires 2-3x the normal fluid
“Single-Bubble Sign” and ultrasound examination will requirement of a neonate (120-180 ml. IVF/kg).
show an elongated pyloric canal (16 mm long or Feeding is usually delayed especially in
more) and a thickened pyloric muscle (4 mm thick or Gastroschisis, necessitating parenteral nutrition.
more).
> Primary closure of the defect is the ideal
> Surgical treatment is the only effective treatment treatment, however, if it cannot be done, staged
— Incision of the sero-muscular layer of the pylorus closure using silo prosthesis or just skin closure with
(“Fredet-Ramstedt Pyloromyotomy”). a resultant ventral hernia is recommended.

ANORECTAL MALFORMATION RESPIRATORY DISTRESS

Imperforate Anus Congenital Diaphragmatic Hernia (CDH)

> Imperforate Anus are of the Low or High Type, > An extra-pulmonary condition that manifests as
depending on the level of the Blind Rectal Pouch, respiratory distress (tachypnea, intercostal,
whether it is within or above the “Complex Muscle of subcostal, and suprasternal retractions) after birth
Continence” (Levator Ani Muscles). Gross perineal within 24-28 hours. The physical appearance serves
findings may indicate the type of Imperforate Anus; as a hint to the neonate’s problem (“barrel-chest,
“Meconium Beads, Bucket-handle Deformity, Ano- scaphoid abdomen”). There is decreased to absent
perineal fistula” are findings compatible with Low breath sounds on the aftected side, predominantly
Imperforate Anus; “Rectovestibular fistula, the left, and the heart sound is appreciated on the
Meconium coming out from the male’s urethral opposite chest.
meatus and female’s vaginal introitus, Flat bottom”
are findings compatible with High Imperforate Anus. > Immediate surgical closure of the diaphragmatic
defect does not translate into better survival but
> Cross-table Lateral X-rays are are done in those may tilt the balance if done on an unstable neonate
undetermined type after 18-24 hours, to allow and results in higher morbidity and mortality
swallowed air to reach the most distal part of the secondary to Pulmonary Hypertension. Optimizing
bowel. In the neonatal period, if the distance Acid-Base and Hemodynamic status with
between the rectal pouch and the anal dimple is pharmaceutical agents (pulmonary vasodilators),
within 1 cm., it is of the Low type; If the distance is special ventilatory support (high-Frequency
greater than 1 cm., it is of the High Type. oscillation), and even extra-corporeal blood
oxygenation (ECMO) may be done prior to surgical
> Low Imperforate Anus are treated with outright correction.
creation of an anal opening (“Anoplasty”). While
UST FMS MEDICAL BOARD REVIEW 2019 2 | SURGERY
PEDIATRIC SURGERY
HERMOGENES R. REGAL, MD

Esophageal Atresia manifest as symptoms of Biliary Atresia. There are 5


types of Choledochal Cyst according to Todani
> Another extra-pulmonary condition that may Classification:
present as respiratory distress if not recognized 1. Fusiform; 2. Diverticular;
early. Failure to insert the nasogastric tube at the 8- 3. Choledochocoele; 4. Mixed intrahepatic and extra
10 cm level leads one to suspect the presence of hepatic bile duct dilatations;
Esophageal Atresia, thus, avoiding initiation of 5. Caroli’s.
feeding that will cause aspiration. Another cause of Surgical treatment is excision of the dilated
aspiration is reflux of gastric content through the extrahepatic bile duct and and reconstruction with
distal Tracheo-esophageal fistula. A reason why hepatico-jejunal anastomosis, Roux-en-Y. Drainage
these babies should always be placed in a semi- of the cyst into any segment of the intestinal tract
upright position to minimize gastro-esophageal results in higher rate of cholangitis and progressive
reflux. cirrhosis, aside from malignancy developing in the
retained cyst wall.
> There are several types of Esophageal Atresia.
The most common type (85%) is a proximal HEAD AND NECK LESIONS
oesophageal atresia with a distal tracheo-esophageal
fistula. A remote second is an isolated oesophageal Thyroglossal Duct Cyst
atresia without a fistula. Those neonates with a
tracheo-esophageal fistula will have globular > This is a midline neck mass appreciated starting at
abdomen, while those without fistula will have a 3 years of age when fat around the neck decreases.
scaphoid abdomen. It’s located at the area of the hyoid bone and moves
when the tongue is protruded. A common
> Treatment is primary anastomosis with ligation of complication is cyst infection. In such situation,
the fistula except in neonates with severe cardiac antibiotic treatment is given and drainage if abscess
anomalies. develops. The treatment is excision of the cyst,
together with its tract up to the base of the tongue
JAUNDICE including the medial portion of the hyoid bone
(Sistrunk’s Operation).
Biliary Atresia and Neonatal Hepatitis
Branchial Remnants
> Pathologic jaundice (very early or persistent
jaundice beyond physiologic duration) should be > These are congenital anomalies presenting either
entertained and recognised early because of as skin opening (Sinus or Fistula) or a cystic mass
prognostic implication. The two pathologic on the lateral aspect of the neck (anterior triangle).
conditions to entertain during this neonatal period is The most common is the 2nd Branchial Anomaly,
“Biliary Atresia” and “Neonatal Hepatitis”. either as a fistula or a cyst. The cyst presents later
Choledochal Cyst is possible but mainly presents at a in childhood while the fistula manifest early even in
later age. Jaundice, echoic stools, and highly infancy. The external opening of the fistula is seen
coloured urine are the main manifestations of Biliary just anterior to the mid portion of the
Atresia. Acholic stools may also be present in sternocleidomastoid muscle and ends up internally
Neonatal Hepatitis depending on the severity of the into the tonsillar fossa or pyriform sinus. A remote
parenchymal involvement. second most common is the 1st Branchial Anomaly
whose external opening is near the angle of the
> No liver profile nor blood examinations will mandible and ends up inside the external auditory
specifically identify Biliary Atresia or Neonatal canal. A common complication is development of
Hepatitis. Absent or rudimentary gall bladder, and infection. Excision is the treatment of choice.
“Triangular cord sign” in Ultrasound are findings in
Biliary Atresia. Percutaneous liver biopsy, if needed, Torticollis
will show histologic findings of periportal fibrosis and
neoductular proliferation in Biliary Atresia. It is not > An alarming hard mass on the lateral aspect of
uncommon for these infants to end up with biliary the neck starting at the 2nd week of life. It is a
exploration in the operating room. fibrotic sternocleidomastoid muscle resulting in
flexion of the neck on the ipsilateral side and
> Surgical correction of Biliary Atresia (Kasai rotation towards the contralateral side. Persistent
Hepato-Portoenterostomy) should be done within position of the baby’s head and neck will result in
the golden period of the first 2 months of age. Later facial (hemihypoplasia) and cranial (plagiocephaly)
correction results in poorer jaundice-free survival deformities. Passive movement (exercises) of the
rate. Those with clinical signs of liver cirrhosis and neck is done to prevent the above complications.
portal hypertension are candidates for liver The mass often spontaneously resolves over time
transplant. during infancy.
Cystic Hygroma
Choledochal Cyst
> A cystic mass commonly located in the posterior
> Choledochal Cyst presents as a triad of Abdominal triangle of the neck but may also be found in other
pain, jaundice, and palpable mass, in younger areas such as the axilla, the retroperitoneum, and
children. In the neonatal period or infancy, it will the extremities. It’s lymphatic in origin and it does
UST FMS MEDICAL BOARD REVIEW 2019 3 | SURGERY
PEDIATRIC SURGERY
HERMOGENES R. REGAL, MD

not regress spontaneously, unlike tumors of vascular PEDIATRIC TRAUMA


origin. Infection and intracystic bleeding are its Initial Assessment and Management
complications. Being a benign tumor, excision
without sacrificing vital structures of the neck is the > Injuries sustained by paediatric patients are
treatment. Unresectable, or problematic cysts may mostly blunt (falls, vehicular accident, etc.), in
be treated with sclerosing agents such as Bleomycin contrast with that of the adult, which is penetrating.
or OK 432. Due to the nature of mechanism of injury and the
child’s size, multiple organ injury is common.
INTRA-ABDOMINAL MASSES
> There are anatomic and physiologic peculiarities
Wilms’ Tumor of children that has to be considered in addressing a
paediatric trauma patient; larger head proportion,
> This is an embryonal tumor affecting the kidney shorter and narrower airway, less subcutaneous fat,
containing tissues from normally developing kidneys; pliable ribs and mediastinum, unprotected intra-
Blastemal, Stromal, Epithelial tubules. Great stride in abdominal solid organs (liver, kidney), hemodynamic
the improvement of prognosis and survival has been status, to name a few.
achieved in this type of tumor based on the series of
American continental and multi-institutional studies > Nevertheless, Assessment and Management still
conducted by the NWTSG for several decades. They follows the protocol: Primary Survey, Resuscitation,
were able to identify prognostic factors: Histology Secondary Survey, Monitoring and Reassessment up
(favourable and unfavourable); Stage; Loss of to Definitive treatment.
Heterozygosity (1p & 16q). Unfavourable histology,
advanced stage, and loss of heterozygosity spell Child Abuse
poorer prognosis. Optimum chemo and radio
adjuvant treatment were also identified for the > Approximately 80% of all abusers are known to
different stages of the disease. Across the seas, the the child, such as the parents or close relatives.
European continental SIOP were also able to achieve
the same good results, differing only in the routine > One should suspect abuse in the following
neoadjuvant chemotherapeutic treatment sans situations: The severity of injury does not conform
histological confirmation. Surgical excision is the with the cause, or even denial of any cause; The
main treatment for this tumor. history varies or is inconsistent from time to time or
from informant to informant; Apathy from parents or
> Syndromes may be associated with this tumor relatives; Significant delay in seeking treatment.
such as Beckwith-Weidemann Syndrome, WAGR
Syndrome, and Denys-Drash Syndrome. > Findings in the child should make one suspect of
abuse: Bite marks, old fractures in different stages
> In bilateral involvement of the kidneys, kidney of healing, cigarette burns, “shaken baby
preservation is the principle being followed; syndrome”, immersion scald burns in infants, bruises
Neoadjuvant chemotherapy to shrink the tumors, of various colours and resembling shape of objects
and partial nephrectomy when this is achievable. and hand, and many more.

Neuroblastoma > The above situations and findings should be


reported to appropriate authorities. A high index of
> This is a tumor of neural crest in origin, arising suspicion should always be present.
from the adrenal medulla and along the sympathetic
ganglion chain from the neck down to the pelvis.
Despite extensive studies and even mass screening,
the achieved survival rate was not successful as that
of Wilms’ tumor, except for infants younger than 1
year of age, across all stages.

> Likewise, prognostic factors were identified: Good


outcomes are associated with Stage I, II, and IV-S
patients who are younger than 18 months, have
hyper diploid DNA flow cytometry, favourable
histology and have <1 copy of MYCN, high Trk-A
expression, and absence of chromosomal 1p
abnormalities. In contrast, a poor prognosis is
predicted in children older than 18 months with-
advanced tumors (stages III and IV), more than 10
copies of MYCN , low Trk-A expression, diploid
DNA ploidy, allelic loss of 1p36, and unfavorable
histology.

> The mainstay in management is surgical excision.

UST FMS MEDICAL BOARD REVIEW 2019 4 | SURGERY


REVIEW TEST
HERMOGENES R. REGAL, MD

-its usual massive amount of bleeding, is


PEDIATRIC SURGERY REVIEW TEST asymptomatic.
d. No visible meconium
1. A 3-week old infant presents with persistent
bilious vomiting and a non-distended abdomen. 8. Which among these findings are consistent with
A surgical condition to consider would be: low imperforate anus?
a. Duodenal atresia a. Meconium beads
b. Malrotation b. Rectal gas above the pubo-coccygeal
c. Hypertrophic pyloric stenosis line
d. Ileal atresia c. Distance between the rectal gas and
anal dimple is greater than 1 cm.
2. A 2-day old neonate developed persistent d. "Flat bottom"
bilious vomiting without abdominal
distention. Barium enema performed 9. In what part of the bowel are ganglion cells
showed the cecum located in the right absent in Hirschsprung's disease?
lower quadrant. The most probable a. Contracted segment
diagnosis is: b. Proximal transition zone
a. Duodenal atresia c. Dilated segment
b. Malrotation d. Proximal to the dilated segment
c. Hypertrophic pyloric stenosis
d. Ileal atresia 10. Which among these findings are compatible with
Hirschsprung's disease affecting the entire colon?
3. A 2-day old neonate manifested bilious a. The appendix has ganglion cells
vomiting and abdominal distention. A b. Barium enema shows "microcolon"
possible diagnosis is: c. Paucity of abdominal gas on x-ray
a. Duodenal atresia d. Transition zone is at the sigmoid
b. Malrotation
c. Hypertrophic pyloric stenosis 11. Physical examination findings in Gastroschisis:
d. Ileal atresia a. Has a sac covering
b. Defect is 5 cm and larger
4. A 4-month old infant presents with irritability, c. May have liver evisceration
vomiting, and bloody stool. Which is d. Defect is to the right of the umbilicus
the appropriate procedure to be done?
a. Proctosigmoidoscopy 12. A 6-hour old neonate has respiratory distress
b. Stool culture and cyanosis. Breath sounds were absent over the
c. Barium enema left chest. The abdomen is unusually flat. The most
d. Technetium pertechnetate scan plausible condition is:
a. Pneumothorax
5. In a 6-month old infant with an enlarged b. Cystic lesions of the lung
fluctuant right scrotum, groins flat, and c. Congenital Diaphragmatic hernia
asymptomatic, the appropriate d. Esophageal atresia
treatment would be:
a. Observation 13. A 3-hour old neonate has an unusually flat
b. Aspiration abdomen and vomits immediately during milk
c. High ligation of the hernial sac feeding followed by respiratory distress. What type
d. Mesh repair of esophageal atresia does the infant have?
a. Proximal atresia with distal TEF
6. A 2-year old boy was brought for consultation b. Isolated esophageal atresia
because of an empty right scrotum. At birth, both c. Atresia with proximal and distal TEF
testes were palpable at the scrotum. On PE, the d. Isolated TEF
right testis was palpated at the lower groin near the
base of the scrotum and can be mobilized down into 14. Source/s of respiratory distress in esophageal
the scrotal sac. What is the recommended atresia:
treatment? a. Aspiration of saliva
a. Orchidopexy b. Aspiration of gastric fluid
b. Hormonal treatment c. Gastric distention
c. Orchiectomy d. All
d. Observation
15. Finding consistent with biliary atresia:
7. Which finding in imperforate anus is consistent a. Ultrasound reveals dilated intrahepatic
with a better prognosis with regard to continence? ducts
a. Meconium coming out from the vaginal b. HIDA scan detects isotopes in the small
orifice intestines
b. Meconium coming out from the urethral c. Liver biopsy shows giant cell
meatus transformation
c. Meconium coming out from the for d. Ultrasound shows absent or contracted
Meckel's diverticulum, which, aside from gall bladder
UST FMS MEDICAL BOARD REVIEW 2019 | SURGERY
REVIEW TEST
HERMOGENES R. REGAL, MD

16. A 3-year old boy presents with a midline neck


mass near the hyoid bone. The mass moves with
tongue protrusion. What is the appropriate
treatment?
a. Aspiration
b. Excision of the mass
c. Excision of the mass and its tract up to
the base of the tongue
d. Excision of the mass and its tract up to
the base of the tongue including the
midportion of the hyoid bone

17. A 1-month old baby has a 5x4x3 cm fluctuant


mass on the left lateral neck. Which of the following
statements regarding the mass is true?
a. A possible complication is intracystic
bleeding or cyst infection
b. It arose from the second branchial
remnants
c. It spontaneously resolves
d. Radical resection is needed because of
its malignant nature and abdominal
distention contributing to the respiratory
distress.

18. A mother noticed a palpable mass in the


abdomen while bathing her 1-year old child. HVA
and VMA examinations showed increased values.
Which among these examinations would indicate
better prognosis?
a. MYCN amplification
b. High mitotic karyorrhexis index
c. Shimada stroma-rich
d. DNA diploidy

19. Anatomic peculiarities of pediatric patients that


should be considered in dealing with trauma
a. Lesser body surface area
b. Larger proportion of head
c. Greater subcutaneous fat
d. Well-protected intra-abdominal
organs

20. We should suspect child abuse when:


a. Discrepancy between the history and
severity of the injury
b. The history given by the relatives are
inconsistent
c. There is a significant delay from the
time of the injury to the seeking of
medical attention
d. All of the above

UST FMS MEDICAL BOARD REVIEW 2019 | SURGERY


CARDIOPULMONARY BYPASS
CHRISTOPHER CHENG, MD

CARDIOPULMONARY COMPOSITION OF CARDIOPLEGIA SOLUTION


BYPASS Dextrose 50% KCl NaHCO3
Adult 25 ml 20 ml (40 meqs) 10 meqs
CARDIOPULMONARY BYPASS Pedia 25 ml 10 ml (20 meqs) 10 meqs
MODEL OF AN OXYGENATOR Infants 25 ml 5 ml (10 meqs) 10 meqs

CONDUCT OF PERFUSION CARDIOPLEGIA


I. Perfusion Circuits Intermittent 4:1 cold blood cardioplegia
 oxygenator membrane, hardshell Temperature cardioplegia 4° - 8°C
reservoir Antegrade infusion
 arterial filter Pressure about 150 mmHg
 3/8” tubing for arterial line Retrograde infusion
 ½” tubing for venous line Coronary sinus pressure = 30 – 50 mmHg
 ¼” tubing for yellow vent, green and blue
for cardiotomy reservoir Maintenance doses given every 20 - 30 mins.
 cardioplegia line
“Hot Shot” or warm BCS given before removal of
CONDUCT OF PERFUSION cross-clamp
 Circuit Prime
500 ml of Plain LR WEANING FROM BYPASS
5 ml of Heparin (1000 units / ml)
 Drugs in Prime 1 gm. of Ca++ gluconate given after removal of
50 meqs of NaHCO3 (50 ml) cross-clamp venous line slowly clamp upon
500 mgs Solu-Medrol reaching body temperature of 37°C full clamp
Mannitol of venous line on coming “OFF-BYPASS”

5 Color - Coded Pump Heads SYSTEMIC INFLAMMATORY


1. Red - Arterial pump RESPONSE SYNDROME
2. Pink - Cardioplegia delivery pump  Leukocytosis
3. Yellow - LV / RV vent pum  Fever
4. Green - Cardiac sucker pump  Pulmonary complications
5. Blue - Accessory sucker pump  Renal complications
 Bleeding
CONDUCT OF PERFUSION
II. Flow Rates

Cardiac Index
BSA x 2.4 Normothermia (37°C)
BSA x 2.0 - 2.2 Mild (31° - 32°C)
BSA x 1.8 Moderate (28°-30°C)

ARTERIAL CANNULA
FLOW CHART

CONDUCT OF PERFUSION
III. Anti-coagulation Management

ACT is maintained at greater than 480 secs


ACT checked every 20 - 30 minutes interval

CONDUCT OF PERFUSION
IV. Perfusion Management

Perfusion pressures
MAP - maintain between 50 - 70 mmHg during CPB
Urine output - 1 cc / kg / hr.
CLASSIFICATION OF HYPOTHERMIA
Classification Temperature

Mild 32° - 37°C


Moderate 28° - 31°C
Deep 18° - 28°C
Profound 0° - 18°C

UST FMS MEDICAL BOARD REVIEW 2019 1 | SURGERY


REVIEW TEST
CHRISTOPHER CHENG, MD

_____9. Lung cancer in a 65 y.o female with no previous


TCVS REVIEW TEST smoking history is most likely to be
A. Large cell carcinoma
CHOOSE THE BEST ANSWER: B. Squamous cell carcinoma
C. Bronchoalveolar Carcinoma
______1. The most common frequently encountered D. Adenocarcinoma
anterior mediastinal tumor in adults is:
A. Lymphoma ____10. Which of the following preoperative functional
B. Neurogenic tumors assessments would qualify for lobectomy in patients with
C. Thymoma lung cancer?
D. Germ cell tumor A. FEV1 of 1.5L
B. FEV1 of 2 L
______2. A patient with lung cancer who presents with C. FEV1 < 80% predicted
brain metastasis is most likely to have D. All of the above
A. Squamous cell carcinoma
B. Small cell carcinoma One or more completion
C. Bronchoalveolar Carcinoma A - if 1, 2 & 3 are correct
D. Adenocarcinoma B - if 1 & 3 are correct
C - if 2 & 4 are correct
______3. Myasthenic patients without Thymoma will D - if all are correct
experience improvement or resolution of muscle weakness
after thymic clean-up in: _____11. ASD closure is indicated for:
A. 95% A. Symptomatic young patients before 7 yrs.
B. 75% old
C. 50% B. Adult patients with clinical cyanosis
D. 25% C. Adult patients with Qp/Qs = 1.5:1
D. Adult patients with pulmonary vascular
______4. A 60 y.o. with 30 pack-yr smoking history resistance (PVR) of 10 wood units/m2
presented with a 2.5cm SPN at the lateral border of the
right lower lobe on Chest X-ray. The best initial diagnostic _____12. True statements regarding isolated VSD:
procedure is A. 80% diagnosed at one month of age will
A. Open thoracotomy for excisional biopsy close spontaneously with peri membranous
B. VATS lung biopsy and muscular VSDs
C. Fine needle biopsy B. Initial CHF from left to right shunt may
D. Repeat CT scan after 2 months anti-TB improve with increasing pulmonary vascular
treatment resistance
C. If untreated, reversal of shunt occurs
_____5. The primary treatment for oat(small) cell (Eisenmenger Syndrome)
carcinoma of the of the lung is D. Closure is contraindication in Eisenmenger
A. Surgery Syndrome
B. Chemotherapy
C. Chemotherapy and radiotherapy _____13. Palliative procedures for congenital heart
D. All of the above disease include:
A. Balloon atrial septostomy (Rashkind
_____6. Pancoast tumor in the early stages (without N2) procedure)
should be treated with B. SVC to RPA shunt (Glenn Shunt)
A. Surgical resection followed by radiation C. Subclavian artery to pulmonary artery shunt
therapy (Blalock-Taussig Shunt)
B. Surgical resection followed by chemotherapy D. Pulmonary artery banding (PAB)
and radiotherapy _____14. Blalock-Taussig Shunt palliation is to increase
C. Induction chemotherapy followed by surgical effective pulmonary flow for cyanotic (CHD), therefore
resection indicated for babies with:
D. Induction chemotherapy and radiotherapy A. Total anomalies pulmonary venous return
followed by surgical resection B. Tetralogy of Fallot
C. Truncus arteriosis
_____7. Commonly associated symptoms with Pancoast D. Tricuspid artesia
tumor are, except:
A. chest and associated shoulder pain _____15. Congenital heart defects that will lead to
B. Horner’s syndrome irreversible pulmonary hypertension (Eisenmenger
C. SVC syndrome Physiology) when left untreated:
D. Atrophy of hand muscles A. Large ASD-secumdum
B. Complete AV canal defect
_____8. Advantages of VATS over open thoracotomy C. VSD
is/are: D. PDA
A. Reduced mortality
B. More complete surgical resection _____16. Surgical procedure options for D-TGA
C. Faster recovery and return to regular (transposition of Great Arteries):
activities A. Blalock-Taussig Shunt
D. All of the above B. Arterial switch-Jatene
C. Ventricular switch
D. Atrial switch (baffle) – Mustard or Senning

UST FMS MEDICAL BOARD REVIEW 2019 | SURGERY


REVIEW TEST
CHRISTOPHER CHENG, MD

_____17. Characteristics of myocardial stunning: ______26. The anterior LV wall shown to be hypokinetic
A. Decreased myocardial contractility with on echocardiogram. This would indicate obstruction in
normal perfusion which of the following coronary arteries:
B. Presence of irreversible damage A. Left anterior descending coronary artery
C. Contractile abnormality reversible with time (ans.)
D. Inotropic support contraindicated B. Left circumflex coronary artery
C. Right coronary artery
_____18. Myocardial viability can be assured with: D. Left Obtuse Marginal coronary artery
A. Contrast angiogram and ventriculogram
B. PET SCAN A newborn was noted to be cyanotic especially when
C. Exercise ECG crying. Chest x-ray showed a small boot-shaped heart
D. Dobutamine echocardiograph with diminished pulmonary vascular markings.

_____19. Class 1 indications for CABG surgery alone: ______27. The following diagnosis should be considered:
A. Left main coronary artery (LMCA) stenosis > A. VSD
60% B. Tricuspid atresia
B. Acute myocardial infarction C. TOF (ans.)
C. Three vessel disease with LV dysfunction EF D. TGA
< 50%
D. Evolving STEMI > 12 hrs. without on going ______28. Blalock-Taussig shunting can relieve the
ischemia cyanosis in this baby by connecting the pulmonary artery
to the:
_____20. Independent risk factors for increased morbidity A. aorta
and mortality after CABG surgery: B. innominate artery
A. Acuity of operation C. subclavian artery(ans.)
B. Female gender D. carotid artery
C. Ejection fraction
D. Prior heart operation ______29. However definitive correction of this congenital
heart defect would require the ff: at around one year of
A 35 y.o. male was brought to the emergency room age.
because of cough and fever of one-week duration. The A. closure of VSD
night before admission, he complained of dyspnea and 2- B. widening of the right ventricular outflow
pillow orthopnea. P.E. PR-88, RR-28, BP-110/70, T-39 tract
deg.c. Lungs: decreased breath sounds and tactile C. switching of the great arteries
fremitus on the right lower lung field. D. only a & b (ans.)

______21. Which of the following would you request to


establish the diagnosis?
A. Chest CT scan
B. Chest Ultrasound
C. Chest x- ray (ans.)
D. Thoracentesis

______22. The right chest was shown to be filled with


significant amount purulent fluid causing compression
atelectasis of the right lung. This is best treated with:
A. Thoracentesis
B. Closed tube thoracostomy (ans.)
C. Thoracotomy and decortication
D. VATS

______23. Pleural fluid was sent for C & S. The most likely
pathogen is
A. E. coli
B. M. tuberculosis
C. S. viridans
D. S. pneumonia (ans.)

A 48. y.o. male patient was noted to have a holosystolic


murmur radiating to the left axilla, following an episode of
severe constricting chest pain, an hour ago.
______24. Which of the ff. should be highly considered?
A. RHD with severe mitral regurgitation
B. AMI with papillary muscle dysfunction (ans.)
C. Mitral valve prolapse
D. Acute Aortic valve insufficiency

______25. The diagnosis could be easily established by


which of the ff:
A. 12 lead ECG
B. Cardiac enzymes
C. 2-D echo
D. all of the above (ans.)
UST FMS MEDICAL BOARD REVIEW 2019 | SURGERY

You might also like